You are on page 1of 181

2017, 2018, 2019, 2020,

2021

MRCS Part A
September
Recalls
September 2017 Recalls
EMQ Rib # management
1- young athlete with single rib #, CXR free after 3 hrs- analgesic, discharge
2- old age with COPD with single rib # , CXR free after half an houradmit,analgesic & observe
3- middle age with multiple rib # in severe pain, CXR pneumothorax-chestdrain

EMQ Femoral #
4- 1-80 years SELF DEPENDANT female with # neck of femur ?
Hemiarthoplasty (there wasn’t THR in options )
5- 2- known case of ca breast with subtrochantric #
Internal fixation with intramedullary nail
6- 3-40years old with displaced subcapital #
Internal fixation

EMQ about pulse oximetry/spo2/level of co2


7- 1-shows saturation of hemoglobin - Pulse oximetry
8- 2-affected by low perfusion -Pulse oximetry

9- 12 y old boy with osteoid forming tumor in lower femur?


osteosarcoma??ewing??osteoblastoma??benign bone cyst

10- RTA pt intubated and mechanical ventilation while examination and turningpatient on his
back he had sudden dyspnea??
tube displacement??neurogenic shock??pneumothorax??sudden increasein ICP

11- Young age return from Bangladesh With bloody diarrhea and lowerabdominal pain.
Mucosa severely distorted on sigmoidoscopy??
Entameba, UC, Camphylobacter (any pt return from asia ascaris. not present then entameaba)

12- Man ate barbecued chicken then had bloody diarrhea?? Campylobacter, C.difiicile

13- Inf epigastric vein drains into- external iliac vein

14- Black toe with aortic aneurysm? Embolus

15- Black toe with DM and positive pedal pulse? Small vessels disease

16- Patient Post sigmoid colectomy then discovered he has antithrombin 3 def ??
give warfarin for 3 months?? give warfarin for life?? give heparin ??

17- Acute intestinal ischemia by emboli

1
EMQ vitamins
18- -Alcohol patient with peripheral neuropathy: B6
19- Post gastrectomy anemia: B12
20- Dry eye and skin: Vit A
21- Easy bruising and delayed wound healing: Vit C

22- Stat q of paired t test

23- Stat q of man whitney test

24- Some audit q cant remember


Pregnant woman with DVT and family history of similar attack ??
antiphospholipid ab a.k.a anticardiolipin

25- A patient with RTA and multiple injuries.Developed dyspnea .. X ray show edema
bilaterally….Defect in ARDs-DIFFUSION DEFECT

26- What is the JVP in AF--- absent "a" wave AF

27- Signs and symptoms of Hyperthyroidism with eye sign? Graves

28- Long history given about hypothyroidism with TSH low/T3 low? 2ry hypothyroidism

29- Patient with Acromegaly how to investigate


PET scan, AP skull , Lateral skull, Cephalometry, CT (there wasn’t mri in options)

30- Clear q about CPP calculation


BP 110/70 and ICP 18 was given
0.333 (systolic-dystolic)+dystolic=MAP hence, 0.333 (110-70)+70
MAP-ICP=CCP (83-18)=65

31- Transpyloric plane at what level- L1

32- Origin of inf parathyroid??


3rd pharyngeal pouch or 3rd pharyngeal cleft or3rd pharyngeal arch

32- Question about planter flexion of ankle…asking root value-S1

33- History of trauma to wrist…lunate dislocationAsking nerve damage. Median n lesion ?

34- Q about young boy with trauma wrist.Xray normal and discharged. Came back after one
week with severepain at anatomical snuff box….DX? Scaphoid fracture

2
35- Q of defect in distribution of both ulnar and median nerve ?? klumpkey’spalsy lower trunk
brac plexus(C8,T1) , median trunk of plexus or median cord

36- What is true about Right coronary artery


A- orign after the aortic valve
B- run in interventicular groove
C- give rise to anterior descending
D-arise from ant ascending aorta

37- Where does great cardiac v run? Anterior septal groove

38- Patient for elective hernia repair under general anaesthesia, Temperatures35.8 how to
proceed?x
A.Change the room temperature forcefully and proceed UGA
B. Give IV fluid worm UGA
C. Proceed without changes
D. Switch to local??
E. post pone until at least 36??

39- SA node supplied by? Vagus & symphatetic


40- Lacrimal gland….Pterygopalatine ganglion
41- Patient with injury to finger while gardening.. Came with swelling of pulpspace??
A.Streptococcus pyogenes
B. Klebsiella,
C. staph aureus
42- thorn prick something injury??strept pyogenes
sporothrix most common if not present then strept pyogenes

43- Prosthetic valve bacteria ?


A. S. epidermidis
B. s.viridans
44- Anasthesia in hemodynamics unstable patient: Ketamine
45- Patient with RTA, Developed bradycardia and hypertension why?
Cushing triad/Intracerebral hge

46- RTA with breast trauma and swelling lesion: Fat necrosis

Knee injury
47- 1-twisted knee while playing next day with swelling and positive tap Unable to fully
extend? Medial meniscus
48- 2- Trauma to knee from side with valgus injury … MCL

49- Recurrent chest infection with meconum ilius in infancy? Which test will be abnormal?
.Sweat chloride a/w cystic fibrosis

3
50- Gene in familial polyposis coli ? APC-a/k/a beta catenin (chromosome 5)

51- Breast micro calcification in xray/mamo no mass do what?


A. Mri ?
B. Fnac
C. Core biopsy
52- Hypotension in RTA patient? Give fluid challenge and cross match?? ct??
53- Best prognosis in Melanoma?? Breslow thicken

54- Best prognosis in BCC?? completeness of excision

55- Post splenectomy defect seen in blood??


Increase platelets, HJ,papenheimer, poikilocytosis

56- Post phlebitic syndrome treatment?

57- Neonate with rectovaginal fistula defect in ? Cloacal membrane

58- Unilateral cleft lip defect?


A. Defect in fusion of max with lat nasalprocess(my answer)
B. maxilla with median nasal process
C. mandibular defect
59- There is another Qs about RTA and lucid interval: Extradural hge

60- Why dilated pupil in extra dural left side ?


compression on parasympathetic fibershence unopposed sympathetic
Patient can't smile symmetrical after trauma to facial nerve during surgery?
Buccal or marginal mandibular br of facial nerve (acc. To medscape it’s the second one)

61- Patient cannot look downward and outward? Trochlear Nerve

62- Patient with papillary carcinoma with no evidence of LN on ULS ?


A. total thyroidectomy alone
B. total thyroidectomy with lymph node excision
One more Q about thyroid nodule, palpable thyroid mass on swallowing FNAC negative
Further plan of action? Hemithyroidectomy

63- Q Post renal transplant 18 months with generalised LAP? EBV


Another one
64- Generalized LAP for 2 years I think?
Lymphoma or reactive lymphadenopathy

65- Q Woman with renal ca and bone metastasis pain??

4
Radio(did it) \, NSAID, Paracetamol
Start with analgesic ladder and if that fails go on to radiotherapy

66- Rectal bleeding in child? Juvenile polyp

67- 7 yr old boy …pain in knee and hip limping Q about perthe's disease

68- 11 yr old obese boy with hip pain SUFE

69- A nurse developed contact allergy to gloves? type 4 or 1 hypersensitivity


70- Child with his father working in garden develop rhinorrhea and itching? IgE

EMQ oral lesions


71- One with precancerous red velvety lesion in the mucosa under the tongue
lesion? Erythroplakia
72- One with blue transparent lesion under tongue? Ranula
73- Patchy lesion and Bleeding from tongue Smoker and alcoholic? Scc

74- Patient with history of pancreatitis Presented with yellow stool difficult to flush bulky
What deficiency ? lipase

75- Patient with ca rectum From which artery blood supply derived from? IMA

76- Perforated gastric ulcer at the anterior wall of greater curvature?


A. Left gastroepiploic artery or
B. Rt gastroepiploic
C. Splenic artery,
D. Gastroduodenal

77- Patient received warfarin and developed bleeding Deficiency of ?? vit k

78- Q on scrotal swelling after long standing and has small testes. Disappeared on supine
position? Varicocele??

79- Q First metabolic response of adrenaline?


Glycolysis (not tachycardia like fawzia)

80- Q patient fasting blood glucose > 7 mmol and post prandial > 11? Dx? (pre:
5.5 </ post: 7.8<) Diabetes mellitus

81- Patient developed recurrent attack of dizziness and fainting attack? Hypogycemia.
What is elevated in blood?
A. Insulin level
B. C- peptide level

5
C. Increase insulin + Increase C peptide

82- Which cell depends on Coricycle for its metabolism?

83- Q Beta1 found in? Ventricle

84- Q Patient return from Bangladesh develop painless jaundice? Hepatitis A

85- Patient with UC presented with abdominal pain… max in LIF and High grade fever,
X ray shows distended transverse colon? Toxic megacolon

86- Localised Annular ca sigmoid colon, no secondaries Type of surgery?


A. left hemicolectomy and anastomosis
B. Hartman (did it and mostly wrong)

87- Q Patient with difficult swallowing after long time of reflex? Cancer oesophagus
Barrett leading to adenocarcinoma

88- Acidosis in diarrhea, due to defect in? Loss of HCO3

89- Patient smoker with continuous cough wash out shows Squamous cellwhat changes
happened?
Irreversible or reversible changes??

90- Q What goes with absolute acidosis ?something Cant remember options but mostly was
urinary ph by exclusion

91- Patient with multiple # Develop sign of PE Diagnosis? CTPA

92- Cardiac output at rest in athletic heart?


A. 5 to 6 lit per min
B. 8 to 10 lit per min

93- Temporal tenderness high ESR? Temporal Arteritis a.k.a giant cell arteritis
94- Non caseating necrosis found in
A. Tb
B. Sarcoidosis
C. Crohn disease
D. Celiac

95- Scenario of DIC

96- 2 q about shock types can't remember


97-

6
98- ECG in Repolarization of ventricle- T wave

99- External urethra sphincter Supply by what root S2,3,4

100- Incontinence after delivery with normal sphincter anatomy ? Pudendal nerve

101- A male patient was examined PR. Posterolateral structure in PR exam4 cm from anal
verge ?
A. Puborectalis,
B. seminal vesicles,
C. prostate

102- A doctor examine female patient…PV examination --ant structure palpable is


A. base of urinary bladder,
B. fundus of uterus,

103- Site of ectopic testes ?


A. Root/base of penis,
B. superficial inguinal pouch

EMQ Incisions and structure divided

104- One pfennestine? Anterior Rectus sheath and fascia transversalis


105- Upper midline ? linea alba
106- Inguinal surgery ? ext oblique aponeurosis

107- Lumbar puncture surface marking…Level of ilac crest ? L4

108- Q of male vs female anatomy of pelvic bones. Difference in male and female pelvis

109- Renal artery posterior to renal vein (VAU)

110- 2nd cervical vertebra feature ?? bifid spinous process, odontoidprocess ( No odontoid in
options )

111- Q about hydrocephalus and Dilated lateral and third ventricle? Obstruction of Silvius
aqueduct

112- Q about structures at popliteal fossa..Closest to knee joint ? popliteal artery

113- Q tumour marker of melanoma ? S100

114- Q marker of breast cancer??

7
CA125?? CEA?? NON OFABOVE?? it is CA15 3

2q similar
115- 15 patient with loss of lumbar lordosis and leg numbness
116- 20s weight lifter with loss of groin sensation and leg numbness
A. Central prolapse
B. Post lat prolapse
C. Spondylolisthesis

Stupid EMQ resuscitate


117- One Post appendicectomy and low urine out put: fluid challenge
118- One with RTA severe hypotension: fluid and cross matching
119- One Severe asthmatic and in distress: CPP ventilation

120- Old age with history of cervical OA presented with numbness in upper limb how to
investigate? MRI
121- Complex supracondylar Investigation # ? CT

122- Q Pregnant lady with suspected perforated DU? Asking for investigation?
A. CT,
B. USG,
C. Chest x ray

123- Patient post cholecystectomy 3 day with abdominal pain tachycardia investigation?
A. CT,
B. USG,
C. Coeliac angio

124- Same scenario With PR 100, BP 90/60 Cool peripheries Afebrile. Cause?
HYPOVOLUMEA- Intra abdominal bleed

125- Q chronic COPD ABG shows? High co2 high hco3

126- Patient post op. Diabetic and on insulin. No oral intake. On IV fluidsNS and Hartmann
developed confusion and convulsions. Reason?
Hypoglycemia

127- Q of peri op management of diabetic pt? did glucose monitoring can't remember others

128- Q rapidly increases thyroid mass with fixation to other tissue? Anaplastic

129- Medial border of femoral hernia- Lacunar lig

130- Q relationship of brachial artery and median nerve at cubital fossa- LAM

8
131- Multiple myeloma on Xray finding- Multiple lytic lesion

132- Q Patient with general fatigue and weight loss with positive Benz jones protein in urine
and passing pink color particles- Multiple Myeloma

133- Obstuctive jaundice with pale stool and yellow urine what is Increased urine ?
A. conjugated bilirubin
B. Urobilinogen
Q initiation of abduction of shoulder? Supraspinatous

134- Q renin act on what? Angiotensinogen

135- Q cerebropontine angle tumour involve which cranial nerves? CN 7 8

136- Q patient presented with hyper pigmentation ? addisson crisis

137- Nerve supply to angle of mandible?


A. Great auricular.
B. Auriculotemporal

138- Fluid maintenance post op ? 2 L dextrose and 1 L saline

139- There is Q about patient with history of carcinoid Diagnosis?


A. Ct
B. barium
C. urine 5-HIAA

140- Increase ADH lead to?


-water retention (low Na, other normal)??
Increase fluid in ECF or ICF or trasnscelluar

141- One young age male with neck pain and uvula shifted to another side? quinsy

142- Child with high grade fever and neck spasm to one side with noisy breathing?-
A. diptheria?
B. Retropharyngeal abscess
C. Ludwig angina

143- 39 years female with family history of ca breast, asymptomatic. Screening by?
A. Mammo
B. us??

144- Q Multiple small gall stone.. due to Increased what?-

9
cholesterol
(pigmented GS)
Unconj bil

145- Q Patient with Pancolitis, which extra intestinal manifestation is least likely to occur in
patient with UC? Polyarteritis nodosa

146- Q Patient can not do opposition of thumb


A. Recurrent br of median nerve (specific for opponens)
B. Deep branch of Ulnar

147- There is a case RTA with shock Sign of acidosis?


In the choices decrease bicarbonate

148- The process which Enhance bacterial ingestion? Opsonization

149- Q Multiple LN What to find on biopsy? Multiple abnormal mitotic figures

150- History of appendicectomy. With yellow granules from RIF wound - Actinomyces
151- Case post op received large amount of blood transfusion developed SOB?
A. Fluid overload
B. hemolytic

152- Q post TURP Hypotension and confusion electrolyte abnormality- HYPONATREMIA

153- Most common sign of hyponatremia?


Confusion ???

154- late complications of blood transfusion? Jaundice (GVHD)

155- Bleeding while passing chest tube Source of Bleeding from? intercostal A

156- Hx of pelvic surgery With Post op erectile dysfunction? Pelvic splanchnic n

157- PE what ECG finding direct: t wave inversion in ll, lll and avF

158- Patient with long standing celiac disease. What type of lymphoma developed?
A. B cell
B. T cell ( t acc to medscape)

159- Which one of the following increase gastric acid secretion?


A. Histamine,
B. gastrin,
C. vagus

10
160- Prosthetic metallic valve behind the sternum at the level 3rd ICS?
A. Aortic,
B. Mitral
C. Pulmonary
161- During tracheostomy Structure in front 2nd, 3rd and 4th tracheal cartilage?
Thyroid isthmus

162- Q Epithelial lining of the urinary bladder? Transitional cell

163- Patient post op sigmodectomy and stoma formation Which of the following is part of
recovery enhance program?

164- Weird q of frozen section? did Slide before staining

165- Hyponatremia post op asking the cause? Administration of Na free fluid

166- Q lose of sensation in dorsum of foot and dorsflexion? Common peroneal

167- Q Recurret UTI and pneumoturia? Diverticular

168- Q Hypoxia and hypo ventilation? insensitivity of central chemoreceptors to PCO2

169- One of the border of anatomical snuff box EPL

170- Which of the following increase water and bicarbonate secretion from pancreases?
Secretin

171- Lose of foot eversion The sensory area affected is? CPN (dorsum of the foot)

172- Why to choice inulin to measure GFR? filtered and not reabsorbed

173- Which one used to measure GFR? Inulin

174- Protein C deficiency Defect in what?


A. Factor 5
B. Factor 8

175- Noradrenaline act on which receptor? alpha1

176- Scenario of parotid swelling and facial nerve involvement? Adenoid cystic Ca

177- Numbness of cheek and upper frontal teeth after trauma to face? Infraorbital N

178- Which of the following is correct regarding meneitriere disease? gastric ca

11
DVT prophylaxis 2 q emq
179- Patient undergoing sigmoid colectomy?? LMWH + stocking
180- Patient Dm undergoing above knee amputation?? LMWH

Patient with duke B classification 5 y survival ? 70%

181- Patient with fall from height or burn with highly urea and creatinine with dark urine
contain myoglobin? Rhabdomyolysis

Emq from lymphaadenopathy


Options
Anal cancer
Melanoma
Cervical cancer
Papillary thyroid cancer
Qs
182- Patient return from Australia to UK has unilateral inguinal LN
183- Patient has underwear bleeding and bilateral inguinal LN
184- Patient has no neck mass and post triangle LN

EMQ 2 Q
185- Patient with # neck femur before one year, treated by internal fixation now presented
with pain and stiffness of hip? AVN

186- Patient with metatarsal dislocation # , wire fixation After ten years with pain and
stiffness ? post traumatic OA

187- Patient with left side leg weakness What circulation? Right anterior cerebral artery

188- Defect in pleuroperitoneal membrane result in? bochadelek hernia

189- From sheet fawzia, Absent femoral pulse and radial pulse intact
Inturrupted aortic arch syndrome

190- Hypothermia first response ? Vasocontriction

191- Blood supply to cecal tumor? ILEOCECAL

192- Procesus vaginalis derived from? Peritoneum

193- Patient with miosis and hyperhydrosis with blinking defect?

Emq 2 q

12
194- Patient planning for elective inguinal surgery Hb 9 How to correct?
I chose give haeme oral iron preparation
195- Cant remember other one

196- Patient with renal failure Serum k 7 what is the first action? Calcium gluconate

197- About anatomy of spleen.which of the following is true? I chose


contain macrophage

198- Patient works in vinyl chloride factory Develop liver mass Angiosarcoma

199- Patient with HIV and thyroid mass what are the action?
A. Repeated investigations
B. Post pone surgery
C. Inform all the staff in the theater
D. Continue with prophylaxis

200. Patient with anal ca associated with? HPV 16,18

201- Patient on oral antibiotics developed dysphagia? Candida

202- Patient with multiple radio opaque stone analysis confirm ca oxalate stone. What is the
cause?
A. hyperparathyroidism
B. Familial hyperoxaluria (right answer)

203- Q Patient on warfarin with INR 5. He is about to undergo surgery for strangulated
femoral hernia
A. IV vitamin K
B. FFP
C. Aspirin
D. prothrombin complex

204- Patient post gastrectomy developed B12 deficiency Blood picture will show??
Megaloblastic (MCV increase)

205- Correct about right ventricle ?


A. Forms the apex of heart
B. contact with diaphragm

206- Q Most of pain stimulus transmitted by? Spinothalamic tract

207- Temp tract ? spinothalamic too

13
208- Q Pt with trauma presented with Marked clawing of hand injury to?
A. Ulnar nerve injury at wrist
B. Ulnar nerve at elbow

209- Sensation at first web space by? Deep peroneal nerve

210- Surgery to short saphenous vein, post op loss of sensation over lateral foot? Sural n

211- Abduction of hip impaired Injury to? Sup gluteal nerve

212- Q Patient with torticollis? Injury to spinal accessory nerve

213- Patient with numbness of hand and tingling sensation with dripping objects, Awake him
at night?
Diagnosis as Carpal tunnel syndrome? Treatment Decompression

214- Posterior leg compartment syndrome diagnosis by


A. Hallux flexion,
B. Hallux extension
C. Ankle planter flex

215- Q Injury to ring finger distal phalanx  fdp

216- Patient with ca breast treated by wide local and radiotherapy now presented with back
pain. How to investigate ?
A. Ct,
B. Xray
C. MRI
D. PET

Emq Management of head trauma


217- Patient with GCS 11 and depressed skull #: CT in an hour
218- Patient old age with fall and repeated vomiting: CT
219- Patient alcoholic and fall: Observe

220- Patient with thyroid nodules FNA shows benign cell? Hemithyroidectomy

221- Q Erythematous tender swelling of the breast How to differentiate it from inflammatory
carcinomatous?
A. No response to Antibiotics
B. nipple inversion
C. Associated with mass

14
222- Q 55 year old with ca pancreas Obstructed jaundice with single liver metastasis ?
Treament ERCP stent

223- Q Patient with thyroid nodule and pulsatile forehead mass? Follicular Ca

224- Q testicular torsion How to treat ? Fixation orchidopexy within 6 hours

225- Q 25 yr old male ..appendicectomy done 1 yr ago for carcinoid of tip of appendix.
Presented with facial flush and diarrhea? Investigation?
A. Ct,
B. small bowel enema,
C. large bowel enema
Q After passing central line Patient collapses? Tension pneumothorax

226- Herpes zoster on T8 T10 site?? Costal margin

227- Disease resulted from failure of neural crest cells to migrate? Hirschprung

228- Alcoholic drunk patient with urgent desire to pass urine, Run and fall with trauma from
wall His urgency disappeared. Next day presented to ER with lower abdominal pain and
complaining of no urine out put Bladder empty by exam. Just tender lower abdomen?
BLADDER RUPTURE

229- Q Reason to give steroid to brain tumor before surgery?


A. Decrease peri-tumour edema
B. Dec tumour vuscularity
C. Dec tumour size

230- Q Osteomyelitis in SCD? Salmonella

231- Patient post op for anal fistula. Not healing With RIF pain? CD

232- Botulinum toxin act by? BLOCK ACH

233- There's is Q About Old age with claustrophobia patient and metallic valve With trauma
and fall on shoulder. How to investigate?
A. CT,
B. MRI,
C. Xray,
D. PET scan,
E. USG ??

234- There was that q of finger swelling History of trauma not tender not attached to skin no
foreign body inside?

15
A. Abscess,
B. Fb granuloma,
C. Aneurysm digital a,
D. Epidermoid
235- A man came with left loin pain, USG shows left hydronephrosis and a bladder mass- TCC

237- A patient with painful shoulder. Painful arc with pain between 60 to 120 degree
shoulder movement. Tendon involved?
Supraspinatous M -impingement syndrome/tendinitis/painful arc

238- A question on Baroreceptors… which mechanism acts first in the body in case of
hypovolemia and hypotension.

239- A patient with problem of finger extension and mild wrist extension difficulty.. no
sensory loss… nerve injury? PIN

240- In cushing disease due to pituitary tumor abnormality seen in adrenal gland is?
adrenal hyperplasia

240- A patient developed hypotension immediately after passing foleys catheter….. type 1,
latex allergy

242- A patient with pharyngeal diverticulum. Asking the insertion site of diverticulum..
A. Through inferior constrictor muscles
B. Between medial and lateral constrictor
C. Above the inferior constrictor
D. Between crico & thyropharyngeus

243- A 8 yr child with swelling in neck. Biopsy shows pleomorphic malignant cells with
osteoid formation…Dx?
A. Gaint cell tumor,
B. Enchondroma,
C. Lymphoma

244- A patient with tingling and numbness in the thumb and index finger…more at night.
Positive Tinel's sign. What is the most appropriate management?
A. Analgesia and rest
B. Division of flexor sheaths
C. Brachial plexus exploration
D. Division of flexor retinaculum

245- Prolonged QT with pancreatitis. Asking the electrolyte abnormality?


A. Hypocalcemia -fat saponification
B. Hypercalcemia

16
C. Hypokalaemia
D. Hypo or hypernatremia

246-A patient received blood transfusion. 30 min later he developed hypotension, tachypnea
and back pain.?
A. Acute haemolytic reaction
B. Non haemolytic reaction

247- which one of the following blood transfusion reactions occur late?
A. Fever,
B. malaise,
C. jaundice,
D. haemotylic reaction

248- 50 yr old diabetic male, admitted for THR. After surgery, he had anuria. Vitals signs
normal. No signs of dehydration.
A. Underactive bladder
B. Renal stone
C. Bladder rupture
249- 35 yr old male patient. Admitted 4 days ago for perianal abscess drainage. Discharged.
He presented to ER with diffuse painful gluteal skin irritation redness and fever?
A. Erysipela
B. Cellulitis
C. Necrotizing fascitis

250- A 45 yr old female patient admitted 4 days ago for elective LAP chole and discharged
without any problems. Patient came to ER with severe abdominal pain. Vitals: HR 96, BP
100/90. Temp: 36. RR 25, WBC 3.5 and Hb 10. Dx?
A. Biliary trauma
B. Post biliary syndrome
C. Gram negative sepsis-c difficile (SIRS FEATURES)
D. Post op bleeding

251- Similar question, same scenario. Asking for investigation?


CT, USG, coeliac artery angiogram,

252- EMQ ear discharge


A patient with 40db hearing loss and painless discharge- cholestetoma
A child with ear discharge, pain relieved after ear discharge- ASOM

253- long scenario patient with post op diarrhoea due to antibiotic use. Dx as Cl. diffiile
diarrhoea. Asking for treatment?
A. Oral metro
B. IV metronidazole

17
C. Oral vanco

254- a young patient presented with slow urine stream and increased frequency of urine. Had
fever , urethral discharge and dysuria 6 months before…..
A. Urethral stricture
B. UTI

18
September 2018 Recalls
1. A 70 old diabetic man presented with multiple profuse urination. He describes that every
time he finishes his toilet, he has the same desire to urinate with a large amount. What is the
most likely cause?
A. Cystitis
B. Detrusor overactivity
C. Bladder diverticulum
D. Underactive bladder
E. Bladder outlet obstruction

2. A 49 years old man reported that he neglected his desire to urinate due to stressful
condition. After the situation passed, he felt no desire to urinate. On examination, he has
abdominal tenderness and distension with an empty foley catheter. What is the most
accepted cause?
A. Blocked catheter
B. Hypovolemia
C. Bladder diverticulum
D. Ureteric stone
E. Bladder rupture

3. A patient suffered sensory deficit over the anterior thigh with inability to extend the knee
after an open hernia repair. What is the affected nerve?
A. Obturator nerve
B. Sciatic nerve
C. Genitofemoral nerve
D. Femoral nerve
E. Ilioinguinal nerve

4. A 45-year-old man presents with fever and pain in his right loin and groin. A soft swelling
was noted in his femoral triangle. Diagnosis of a psoas abscess was made. Which of the
following statements is most accurate regarding psoas major?
A. It arises from the lateral borders of the bodies of T12 to L5
B. It extends the hip
C. It inserts into the greater trochanter of the femur
D. It is innervated from T12 and L1
E. It passes posterior to the capsule of the hip joint

5. A 42 years old man developed a painful scrotal swelling after vasectomy. The most
probable diagnosis is:
A. Spermatocele
B. Sperm granuloma
C. Hydrocele
D. Teratoma

19
E. Epidydimoorchitis

6. A 6 years old child presented with his mother with a tender left hemiscrotum. On
examination, there is a blue dot over the upper pole of the left testis. What is the most likely
cause?
A. Torsion testicular appendages
B. Hematocele
C. Torsion testis
D. Varicocele
E. None of the above

7. A 42 years old man noticed a testicular mass after history of trauma 3 weeks. He has
enlarged supraclavicular lymph node. What is the most likely diagnosis?
A. Teratoma
B. Seminoma
C. Lymphoma
D. Hydrocele
E. Sperm granuloma

8. A 7-year-old baby was brought by his mother after she noticed that he has evident limping
with pain in his left hip. She denies any history of trauma. What is the most accurate
diagnosis?
A. Perthes disease
B. Slipped upper femoral epiphysis
C. Developmental dysplasia of the hip
D. Septic arthritis
E. Osteoarthritis

9. A 12 years old overweight child presented with obvious limping and limited internal
rotation. He denies any history of trauma. What is the most accurate diagnosis?
A. Perthes disease
B. Slipped upper femoral epiphysis
C. Developmental dysplasia of the hip
D. Septic arthritis
E. Osteoarthritis

10. A 2 months old neonate was brought by his mother after a breech delivery. She is
concerned that there is a prominent skin crease on the left hip. What is the most accurate
diagnosis?
A. Perthes disease
B. Slipped upper femoral epiphysis
C. Developmental dysplasia of the hip
D. Septic arthritis
E. Osteoarthritis

20
11. A 21 years old soldier presented with metatarsal pain. Xray revealed no bony
abnormality. 2 weeks later, Xray revealed callus formation. What is the most likely cause?
A. March fracture
B. Freiburg disease
C. Morton’s neuroma
D. Osteoarthritis
E. None of the above

12. A female patient presented with sharp pain over the dorsum of the foot radiating to the
toes. On examination, there is tender swelling in the cleft between 3rd and 4th metatarsal
bones. What is the most likely cause?
A. March fracture
B. Freiburg disease
C. Morton’s neuroma
D. Osteoarthritis
E. None of the above

13. A 30 years old athlete sustained trauma to his right knee which became rigidly flexed at
30 degrees. Trials to flex or extend knee were unsuccessful and he mentioned that his knee
gives away. What is the most likely cause?
A. Anterior cruciate ligament injury
B. Loose body
C. Dislocated patella
D. Meniscal tear
E. Posterior cruciate ligament injury

14. A 32 years old athlete injured his left knee. There is a positive valgus stress test with
minimal knee effusion. What is the most likely diagnosis?
A. Anterior cruciate ligament injury
B. Lateral collateral ligament injury
C. Medial collateral ligament injury
D. Meniscal tear
E. Posterior cruciate ligament injury

15. A 33 years old man suffered an ankle injury. Xray was normal and surgeon suspects
ligamentous injury. What is the most appropriate investigation?
A. Repeat Xray
B. MRI
C. CT scan
D. Bone scan
E. Ultrasound

21
16. A man fell on his hand during a rugby game. Examination revealed a suspected
intraarticular fracture of elbow joint. What is the most appropriate investigation?
A. Repeat Xray
B. MRI
C. CT scan
D. Bone scan
E. Ultrasound
17. A 41 years old female presented to ortho clinic complaining of pain in her left shoulder.
Examination revealed a painful arc between 60-120 degrees. What is the most likely cause?
A. Frozen shoulder
B. Anterior shoulder dislocation
C. Posterior shoulder dislocation
D. Supraspinatus tendinitis
E. Calcific tendinitis

18. A patient cannot elevate his arm above 90 degrees and cannot shrug his shoulders after a
posterior neck triangle maneuver. What the nerve affected?
A. Glossopharyngeal nerve
B. Vagus nerve
C. Long thoracic nerve
D. Axillary nerve
E. Accessory nerve

19. A patient underwent ORIF for a bimalleolar fracture, with a below knee cast. Then he
developed inability to dorsiflex his ankle and paranesthesia along the dorsum of the foot.
What is the affected nerve?
A. Common peroneal nerve
B. Sural nerve injury
C. Superficial peroneal nerve
D. Deep peroneal nerve
E. Sciatic nerve

20. A man sustained a deep laceration between Achille’s tendon and lateral malleolus. Which
structure may be injured?
A. Peroneus tertius muscle
B. Saphenous nerve
C. Superficial peroneal nerve
D. Tibialis posterior muscle
E. Peroneus longus muscle

21. A 53 years old female presented with loss of sensation over the lateral aspect of the leg
after a varicose vein surgery. What is the affected nerve?
A. Sural nerve
B. Saphenous nerve

22
C. Superficial peroneal nerve
D. Deep peroneal nerve
E. Tibial nerve

22. A man suffered a fracture neck of femur. ORIF was done through a posterior hip
approach. Postoperatively, he complained weakness in his left leg. What is the likely cause?
A. Tourniquet paralysis
B. Sciatic nerve injury
C. Femoral nerve injury
D. Compartmental syndrome
E. None of the above

23. A 33 years old man suffered an anterior ankle laceration. On examination, there is loss of
toes dorsiflexion. Where do you expect an associated sensory loss?
A. Dorsum of the foot
B. 1st web space
C. Dorsum of the foot except 1st web space
D. Lateral aspect of the foot
E. Medial aspect of the foot

24. A 26-year-old man presents to the Emergency Department with extensive bleeding from
his arm after sustaining a glass injury. On examination there is a 7 cm transverse laceration
across the anterior aspect of his elbow. On exploring the cubital fossa, you would expect the
brachial artery to be:
A. anterior to the median nerve
B. lateral to the biceps tendon
C. lateral to the median nerve
D. medial to the median nerve
E. superficial to the bicipital aponeurosis

25. Where is the level of ulnar nerve injury in a hand with complete clawing at little and ring,
interossei wasting and hypothesia of the little and ring fingers?
A. At the axilla
B. At the elbow
C. At the upper arm
D. At the wrist
E. Deep ulnar nerve injury

26. A patient has a weak grip; when she tries to hold a knife, her hand flexes with lost
sensation over badge area. What is the affected nerve?
A. Radial nerve
B. Median nerve
C. Ulnar nerve
D. Lateral cord

23
E. Posterior cord

27. A 24 years old female patient presented with a fixed flexed little and ring. He can flex it
normally but cannot extend it due to pain. What is the appropriate treatment?
A. Fasciotomy
B. Release trigger finger
C. Carpal tunnel release
D. NSAIDS
E. None of the above

28. A 40 years old patient presented within 3 hours after suffering a closed subcapsular
fracture. The most appropriate treatment is:
A. Hemiarthroplasty
B. External fixation
C. Internal fixation
D. Casting
E. Total hip replacement

29. What is the anterior relation of femoral hernial neck?


A. Inguinal ligament
B. Pectineal ligament
C. Lacunar ligament
D. Femoral vein
E. Fascia Lata

30. A female patient presented with a dark black gangrene affecting her perineum with
extension of necrotic tissue to the groin. What is the causative organism?
A. Streptococcus pyogenes
B. Staphylococcus aureus
C. Mixed aerobe and anaerobe
D. Pseudomonas
E. MRSA

31. What is the relation of ulnar nerve to the ulnar artery in wrist?
A. Ulnar nerve is ulnar to ulnar artery
B. Ulnar artery is ulnar to the ulnar nerve
C. Ulnar nerve is radial to ulnar artery
D. Ulnar nerve is superficial to ulnar artery
E. None of the above

32. A patient presented with painful ankle joint after sudden trauma to his ankle. Squeezing
his calf does not produce plantar flexion. What is the test to be positive?
A. Retropulsion test
B. Thomas test

24
C. Simmonds Thompson test
D. Forment test
E. None of the above

33. A woman presented with a 24-hour history of colicky pain and bloody diarrhea after
eating barbecued chicken. What is the causative organism?
A. Campylobacter
B. Salmonella
C. Staphylococcus aureus
D. Clostridium perfringens
E. Streptococcus pyogenes

34. What is the causative organism of septic arthritis in a patient with sickle cell disease?
A. Staphylococcus aureus
B. Streptococcus pyogenes
C. Pseudomonas
D. Salmonella
E. None of the above

35. A patient with Gouty arthritis undergone joint aspiration. What is the expected finding
under the microscope?
A. Needle shaped crystals and negative birefringence
B. Rhomboidal shaped crystals and negative birefringence
C. Needle shaped crystals and positive birefringence
D. Rhomboidal shaped crystals and positive birefringence
E. None of the above

36. What is the Least reliable sign of compartment syndrome?


A. Increasing pain out of proportion
B. Altered sensation
C. Increased compartment pressure
D. Pain on passive stretch
E. Loss of pedal pulses

37. A patient recently come from Bangladesh complains of abdominal pain and vomitting. He
gives no history of similar attacks. What is the most likely cause?
A. Typhoid fever
B. Hepatitis A
C. Campylobacter
D. Staphylococcus aureus
E. Clostridium perfringens

38. A 53 years old female presented with a painless dark black eschar affecting her hand.
What is the likely cause?

25
A. Streptococcus pyogenes
B. Anthrax
C. Campylobacter
D. Staphylococcus aureus
E. Clostridium perfringens

39. A patient presented with a wound with dirty odour and crepitation. What is empirical
antibiotic to be used till the result of swap appear?
A. Metronidazole
B. High dose penicillin
C. Low dose penicillin
D. Rifampicin
E. Tetracycline

40. A nasal MRSA carrier patient came to the hospital for surgery. What is the most
appropriate treatment?
A. Mupirocin2%
B. Chlorhexidine gluconate
C. IV vancomycin
D. IV Teicoplanin
E. None of the above

41. A 23 years old man suffered a distal femoral Fracture during a sport injury. Which nerve is
at greatest risk?
A. Tibial nerve
B. Common peroneal nerve
C. Sciatic nerve
D. Superficial peroneal nerve
E. Sural nerve

42. A man suffered a complex tibial fracture during a car accident. There is a dirty overlying
wound. What is the definitive management?
A. Open reduction and internal fixation by nail
B. Open reduction and internal fixation by plate
C. Open reduction and external fixation
D. Closed reduction and cast
E. None of the above

43. A 42 years old weightlifter presented with unexplained loss of weight, pain in both lower
limbs and loss of ability to urinate. What is the most likely cause?
A. Metastatic lesion
B. Stress vertebral body fracture
C. Axial subluxation
D. Spondylolisthesis

26
E. None of the above

44. A 25 years old athlete sustained an ankle injury. Xray was normal before 2 months. He is
now presenting with persistent ankle pain. What is the appropriate next step?
A. X-ray
B. MRI
C. CT
D. Bone scan
E. None of the above

45. A 62 years old man is known to have a prostatic cancer. Investigations revealed a
metastatic vertebral lesion. What is the most appropriate management?
A. Radical prostatectomy
B. PCA
C. Radiotherapy
D. Watchful waiting
E. None of the above

46. A man has trauma to his face affecting orbit with periorbital hematoma, sub-conjunctival
hemorrhage and epistaxis. What is the possible fracture site?
A. Nasal bone
B. Naso-ethmoidal
C. Maxillary
D. Maxilloorbital
E. None of the above

47. A 50-year-old women presents with a diffuse swelling in the region of her right parotid
together with facial pain. On examination she has a right sided facial nerve palsy what is the
cause?
A. Pleomorphic adenoma
B. Adenoid cystic carcinoma
C. Sarcoid
D. Sjogren's syndrome
E. Sialolithiasis

48. 35 yrs. old female on the routine mamo has intermediate microcalcification and the
mamo was negative 3 years ago with no lymph node
A. Ductal carcinoma in situ
B. Lobular carcinoma in situ
C. Invasive duct carcinoma
D. Lobular carcinoma
E. Paget's disease

49. Female patient on warfarin and must make further investigation. What is the next step?

27
A. PET CT
B. Core Biopsy
C. MRI
D. Excisional biopsy
E. None of the above

50. A 29-year-old patient brought to ER after falling from the floor A patient nonresponsive to
any stimuli there is no eye opening what is his GCS?
A. 3
B. 0
C. 5
D. 6
E. 11

51. A 33-year-old male is complaining of a headache and a sensation of pressure between the
eyes and pain in his teeth. On examination she is febrile ad he reported that he had had
dental extraction 5 days ago. What is the likely diagnosis?
A. Ethmoid sinus cancer
B. Maxillary sinus cancer
C. Nasal polyps
D. Ethmoidal sinusitis
E. Maxillary sinusitis

52. A 28 years old patient stabbed on right side posterior to axillary line, Xray showing
pneumothorax with fluid level. What is the best management?
A. Chest drains with suction
B. Needle Decompression
C. thoracentesis
D. Chest drain
E. None of the above

53. Which of the following is a tumor marker of melanoma?


A. VMA
B. CA 15-3
C. S100
D. CEA
E. None of the above

54. Male patient 45-year-old presented with red Velvety like lesion in his tongue
What is the most likely diagnosis?
A. Leukoplakia
B. Erythroplakia
C. Squamous cell carcinoma
D. AIDS

28
E. None of the above

55. A 55-year-old female presented with Neck rubbery mass with axillary and inguinal masses
what is the most likely diagnosis?
A. HIV
B. lymphoma
C. Leukemia
D. Squamous cell carcinoma
E. None of the above

56. A 24-year-old patient presented with trauma to the chest his CXR shown widening of the
mediastinum. what of the structure being rupture?
A. ascending aorta
B. descending aorta
C. arch of the aorta
D. esophageal rupture
E. SVC

57. A 25-year-old male pedestrian is hit by a van on a busy road. He is brought to the
Emergency Department by ambulance. On examination he is dyspneic, and hypoxic despite
administration of high flow 100% oxygen. His blood pressure is 110/70 and pulse rate is 115
bpm. The right side of his chest is hyper-resonant on percussion and has decreased breath
sounds. The trachea is deviated to the left. What is your management?
A. Needle in 5th intercostal space at the mid axillary line
B. Between Xiphoidal process and left sternocostal margin
C. Between Xiphoidal process and right sternocostal margin
D. 3rd intercostal space
E. None of the above

58. A 25-year-old man is admitted having been stabbed in the interior chest. On examination
he is alert and coherent. He has congested neck veins. His pulse is 140 beats/minute and his
blood pressure 90/60 mmHg. He has normal breath sounds. What is the most likely cause of
his cardiac arrest 15 minutes later?
A. Cardiac tamponade
B. Congestive cardiac failure
C. Hypovolemia
D. Tension pneumothorax
E. Ventricular arrhythmia

59. A 85-year-old man presents with a cough and hemoptysis. He has a modest smoking
history of 15 pack years. He is found to have a tumor located in the right main bronchus, with
no evidence of metastatic disease his pathology showing keratin between flat cells. He
decides to undergo any treatment what is your diagnosis?
A. Adenocarcinoma

29
B. Small cell lung cancer
C. Large cell lung cancer
D. Squamous cell carcinoma
E. None of the above

60. A 45-year-old homeless man presents with a cough and weight loss over three months.
On examination his BMI is 19 and he has reduced breath sounds in the right upper zone. His
chest X-ray shows a caveating lesion in the right upper lobe. He undergoes a bronchoscopy
and a bronchial biopsy. The biopsy shows featureless necrosis surrounded by epithelioid
macrophages and giant cells. Which of the following is the most likely diagnosis?
A. Actinomycosis
B. Bronchiectasis
C. Sarcoidosis
D. Squamous cell carcinoma
E. Tuberculosis

61. A 55-year-old smoker patient presented with hemoptysis and mucous phlegm.
pathological examination of his phlegm showing squamous cell in his sputum what is the
most likely diagnosis?
A. Squamous cell carcinoma
B. Small cell carcinoma
C. Tuberculosis
D. reversible metaplasia
E. None of the above
62. A 55-year-old female presented with epigastric pain and vomiting with inability to leaning
forward his investigations reveled elevated amylase and lipase. what is the type of necrosis in
this condition?
A. Liquefactive
B. Fibrinoid
C. Fat
D. Gangrenous
E. Coagulative

63. A 23-year-old female presented with right iliac fossa pain his U/S showing acute
appendicitis. What is the most predominant cell after 24 hours?
A. Neutrophils
B. Basophils
C. Eosinophils
D. Macrophages
E. T lymphocytes

64. A female patient 45 years old referred to surgeon with solitary thyroid swelling of thyroid
of less the pathology of the nodule showing papillary thyroid carcinoma 2cm cm with no
lymph nodes enlarged what is your treatment?

30
A. total thyroidectomy with nodal clearance
B. total thyroidectomy
C. hemithyroidectomy
D. subtotal thyroidectomy
E. None of the above

65. Hassall s corpuscles present in which gland of the following?


A. Thyroid
B. Para thyroid gland
C. Thymus glad
D. Pituitary gland
E. Pancreas

66. Young girl complaining from enlarged cervical lymph node. Diagnosed on Histopathology
as Medullary thyroid carcinoma. what is appropriate management?
A. total thyroidectomy with nodal clearance
B. total thyroidectomy only
C. hemithyroidectomy
D. subtotal thyroidectomy
E. None of the above

67. A tall thin, 25-year-old woman presents to the surgical clinic with a complaint of swelling
in the front of her neck for the last four months. On clinical examination she has a swelling in
the left lobe of the thyroid and multiple neuromas lesions within the oral cavity. Her blood
pressure is 220/120 mmHg. Laboratory investigations show that he calcium and electrolytes
are normal and serum calcitonin and urinary VMA levels are elevated, which of the following
is the most likely cause of the hypertension?
A. Conn's syndrome
B. Cushing's disease
C. Essential hypertension
D. Pheochromocytoma
E. Renal artery stenosis

68. Female patient 50 years old has history of Hashimoto thyroiditis 8 years ago now she
noticed the gland rapidly enlarged and have difficulty of swallowing with pathology bland
cells and CD 20 +ve phenotype what's your diagnosis?
A. Lymphoma
B. Medullary thyroid carcinoma
C. Follicular thyroid carcinoma
D. Papillary thyroid carcinoma
E. None of the above

31
69. A 40-year-old women presented with a mass on her forehead. On examination, she had a
fluctuant pulsatile mass on her head. Examination of her neck revealed a mass inferior to the
hyoid with a positive Berry's sign. What is the most likely underlying diagnosis?
A. Follicular thyroid cancer
B. Medullary thyroid cancer
C. Papillary thyroid cancer
D. Anaplastic thyroid cancer
E. Parathyroid gland cancer

70. A Chinese lady has conductive hearing loss, palatine ulcer and cervical lymphadenopathy,
what is the most likely diagnoses?
A. Nasopharyngeal carcinoma
B. Oropharyngeal carcinoma
C. TB
D. Infectious mononucleosis
E. Actinomycosis

71. You review 42-year-old woman six weeks following a renal transplant for focal segmental
glomerulosclerosis. Following the procedure, she was discharged on a combination of
tacrolimus, mycophenolate, and prednisolone. She has now presented with a five-day history
of feeling generally unwell with anorexia, fatigue and arthralgia. On examination, she has a
temperature of 37.9 and has widespread lymphadenopathy. What is the most likely
diagnosis?
A. Hepatitis C
B. Coxsackie virus
C. HIV
D. Hepatitis B
E. Cytomegalovirus

72. A 64-year-old man presents to the clinic with right upper quadrant discomfort. He has
never attended the hospital previously and is usually well. He has just retired from full time
employment as a machinist in a PVC factory. CT scanning shows a large irregular tumour in
the right lobe of his liver. Which of the following lesions is the most likely?
A. Liposarcoma
B. Angiosarcoma
C. Hamartoma
D. Hydatid liver disease
E. Benign angioma

73. A patient was diagnosed with papillary thyroid carcinoma thyroidectomy was done. what
will be his follow up plan after operation?
A. Calcitonin
B. Thyroglobulin
C. Alpha fetoprotein

32
D. CEA (carcinoembryonic antigen)
E. None of the above

74. A 65-year-old woman presents with a 2.5 cm diameter mass in the upper outer quadrant
of the left breast with associated axillary lymphadenopathy. A core biopsy is taken which
confirms the presence of carcinoma. Which of the following types of carcinoma is this most
likely to be?
A. Invasive ductal carcinoma
B. Invasive lobular carcinoma
C. Medullary carcinoma
D. Mucinous carcinoma
E. Tubular carcinoma

75. A biopsy specimen using Prussian blue stain to detect which of the following?
A. Amyloid
B. Sarcoid
C. IRON
D. Asteroid bodies
E. None of the above

76. A female patient 45 years old referred to surgeon with solitary thyroid swelling of thyroid
of less the pathology of the nodule showing papillary thyroid carcinoma 2 cm with no lymph
nodes enlarged
A. total thyroidectomy with nodal clearance
B. total thyroidectomy only
C. hemithyroidectomy
D. subtotal thyroidectomy
E. None of the above

77. Young girl complaining from enlarged cervical lymph node. Diagnosed on Histopathology
as Medullary thyroid carcinoma. What is appropriate management?
A. total thyroidectomy with nodal clearance
B. total thyroidectomy only
C. hemithyroidectomy
D. subtotal thyroidectomy
E. none of the above

78. A thyroidectomy specimen from a 43-year-old lady shows a mass with prominent oxyphil
cells and scanty thyroid colloid and complaining from pulsatile mass in her skull what is your
diagnosis?
A. Follicular carcinoma
B. Anaplastic carcinoma
C. Medullary carcinoma
D. Papillary carcinoma

33
E. Lymphoma

79. A tall, thin, 25-year-old woman presents to the surgical clinic with a complaint of swelling
in the front of her neck for the last four months. On clinical examination she has a swelling in
the left lobe of the thyroid and multiple neuromas lesions within the oral cavity. Her blood
pressure is 220/120 mmHg. Laboratory investigations show that he calcium and electrolytes
are normal and serum calcitonin and urinary VMA levels are elevated, which of the following
is the most likely cause of the hypertension?
A. Conn's syndrome
B. Cushing's disease
C. Essential hypertension
D. Pheochromocytoma
E. Renal artery stenosis

80. A 65-year-old woman presents with a 2.5 cm diameter mass in the upper outer quadrant
of the left breast with associated axillary lymphadenopathy. A core biopsy is taken which
confirms the presence of carcinoma. Which of the following types of carcinoma is this most
likely to be?
A. Invasive ductal carcinoma
B. Invasive lobular carcinoma
C. Medullary carcinoma
D. Mucinous carcinoma
E. Tubular carcinoma

81. Female patient had mammogram 2 years ago which was normal result , now
mammogram showed microcalcification, her examination and u/s revealed 2 cm mass in her
right breast......??
A. Breast Conserving surgery
B. Modified radical mastectomy
C. Radiotherapy
D. Chemotherapy
E. None of the above

82. 9-year-old female presented with smooth fluctuant mass showing halo sign in
mammography aspiration was done there is no blood, no fluid refilling what is your
management?
A. discharge no follow-up required
B. surgical excision
C. true cut biopsy
D. FANC
E. None of the above

83. A 50-year-old Patient came to ER assessment of consciousness level was done according
to GCS patient can localizes pain, open eye to pain and sounds what the GCS?

34
A. 9
B. 7
C. 12
D. 11
E. 13

84. RTA patient admitted after observation then discharged by giving treatment. 10 days later
came with abdominal discomfort especially after eating, it became worse. What is the cause?
A. Diaphragm rupture
B. diaphragmatic hernia
C. curling ulcer
D. acute gastric dilatation
E. esophageal perforation

85. A 30 years old patient brought to ER with rib fracture patient is intubated, then becomes
hypotensive and hypoxic what is your diagnosis?
A. Tension pneumothorax
B. Hemothorax
C. Hemopericardium
D. Hemopneumothorax
E. None of the above

86. A patient came after RTA, his GCS was less than 10, after log rolling, he suddenly become
hypoxic and rapid decrease in po2 what is your explanation?
A. Respiratory center depression
B. Spinal cord injury
C. tracheal tube displacement
D. pneumothorax
E. large amount of hemorrhage

87. A 27-year-old man is involved in a road traffic accident. He is seen in the emergency
department with chest pain. Clinical examination is essentially unremarkable, and he is
discharged. He subsequently is found dead at home. What is the most likely underlying
injury?
A. Tracheobronchial tree injury
B. Traumatic aortic disruption
C. Cardiac laceration
D. Diaphragmatic rupture
E. Rupture of the esophagus

88. A 19- year-old man fell down a flight of stairs. On presentation to the Emergency
Department his Glasgow Coma Score (GCS) was 14/15 with no local neurological signs. An
hour later he Vomited, and his GCS deteriorated to 10. He developed a fixed and dilated left
pupil. Which of the following is likely to seen on the scan?

35
A. Left-sided acute subdural hematoma
B. Left-sided extradural hematoma
C. Left-sided midbrain hematoma
D. Right-sided acute subdural hematoma
E. Right-sided extradural hematoma

89. A 45-year-old homeless man presents with a cough and weight loss over 3 months. On
examination his BMI is 19 and he has reduced breath sounds in the right upper zone. His
chest X-ray shows a caveating lesion in the right upper lobe. He undergoes a bronchoscopy
and a bronchial biopsy which shows featureless necrosis surrounded by epithelioid
macrophages and giant cells. Which of the following is the most likely diagnosis?
A. Actinomycosis
B. Bronchiectasis
C. Sarcoidosis
D. Squamous cell carcinoma
E. Tuberculosis

90. A 6 years old child brought with his mother with a groin swelling increase with playing
and decrease with laying down. What is the possible diagnosis?
A. Indirect inguinal hernia
B. Varicocele
C. Undescended testis
D. Ectopic testis
E. None of the above

91. In tempo mandibular joint dislocation, the patient has severe pain around the joint. what
is the affected nerve?
A. Auriculotemporal
B. Infra-alveolar
C. Great auricular
D. Great occipital
E. None of the above

92. Patient with serum potassium 6. What should be immediate step?


A. IV fluid
B. Insulin and glucose infusion
C. B agonist
D. Pamidronate
E. IV ca gluconate

93. 30years-old alcoholic man presents with acute severe upper abdominal pain and
vomiting. He is admitted to the intensive therapy unit with a diagnosis of severe acute
pancreatitis 48 hours later he develops peripheral paresthesia and carpopedal spasms. The
most likely underlying metabolic abnormality is :

36
A. hypercalcemia
B. hyponatremia
C. hypocalcemia
D. hypokalemia
E. hypernatremia

94. A woman had a routine gynecological check. what is palpated anterior to the cervix on
PV?
A. base of bladder
B. dome of bladder
C. ischiorectal fossa
D. puborectalis muscle
E. none of the above

95. A 28-year-old man presents with an ischiorectal abscess. Where is this abscess cavity
sited?
A. Above the levator ani
B. Between external and internal anal sphincter
C. Lateral to the obturator internus
D. Medial to the internal anal sphincter
E. Medial to the pudendal canal

96. The maximum point of resistance while you put urinary catheter is:
A. Bulbar urethra
B. membranous urethra
C. preprostatic urethra
D. prostatic urethra
E. none of the above

97. The male pelvis differs from the female one by all of the following except:
A. the female pelvis is wider and shallower than male
B. the female inlet is rounded or oval in shape while male is apple in shape
C. suprapubic angle is acute while male is large
D. the bone is lighter in female than in male
E. coccyx is straight in female while in male is curves

98. A patient is noticed that he is confused, agitated after TUR operation. Diagnosis is:
A. Hypercalcemia
B. Hyperkalemia
C. Hypocalcemia
D. Hypokalemia
E. Hyponatremia

37
99. A surgeon makes a Pfannenstiel incision for access to the pelvic organs. He incises the
abdominal wall down to and through the rectus sheath. He retracts the rectus abdominis
muscles laterally from the midline to expose the:
A. linea alba
B. peritoneum
C. posterior rectus sheath
D. transversalis fascia
E. transversus abdominis muscle

100. Groin mass in a child appears during the day time and disappear during the night what is
the pathology?
A. Dartos muscle
B. Patent processes vaginalis
C. Testicular capsule
D. Tunica albuginea
E. Tunica vasculosa

101. A patient diagnosed that he has a herpes zoster complaining of pain in the tip of the
nose. Which of the following at the same Dermatome of tip of nose?
A. Ear pinna
B. angle of mouth
C. jaw angle
D. cornea
E. None of the above

102. Posterior relation of the kidney is:


A. psoas muscle
B. peritoneum
C. liver
D. hepatic flexure
E. gonadal vessels

103. Anterior relation of head of pancreas include:


A. pylorus of the stomach
B. common bile duct
C. portal vein
D. IVC
E. Aorta

104. Bleeding is come from Anterior gastric ulcer between the antrum and the body of
stomach , what is the source of bleeding?
A. short gastric
B. left gastroepiploic
C. right gastroepiploic

38
D. left gastric
E. none of the above

105. A 63-year-old man who smokes heavily presents with dyspepsia. He is tested and found
to be positive for helicobacter pylori infection. One evening he has an episode of
hematemesis and collapses. What is the most likely vessel to be responsible?
A. Portal vein
B. Superior mesenteric artery
C. Gastroduodenal artery
D. short gastric
E. None of the above

106. Which of the following is most susceptible to colonic ischemia?


A. Right colon
B. Transverse colon
C. Rectosigmoid colon
D. left colon
E. splenic flexure

107. 55 years old complained of a stroke affecting left lower extremity paralysis , what is the
arterial supply responsible for that paralysis
A. right anterior cerebral
B. left anterior cerebral
C. right middle cerebral
D. PICA.
E. anterior spinal artery

108. 31 years old female suffered from dysarthria and wheelbarrow effect of tongue on one
side. which nerve is affected?
A. Lingual nerve
B. Hypoglossal nerve
C. Chorda tympani
D. Glossopharyngeal
E. Vagus

109. 55 years old suffered from Vitamin deficiency and the patient is alcoholic and has
manifestations of Wernicke's ataxia with cardiomyopathy
A. thiamine deficiency
B. pellagra
C. folic acid
D. iron
E. vitamin B12

110. Vitamin deficiency causing night blindness is:

39
A. Vitamin A
B. Vitamin B12
C. Vitamin B2
D. Vitamin C
E. Vitamin D

111. Which hormone increases pancreatic secretion rich in bicarbonate


A. Secretin
B. CCK
C. Gastrin
D. Histamine
E. Serotonin

112. Zinker’s diverticulum in pharynx occur between:


A. Between sup and middle constrictors
B. Between middle and inf constrictor
C. Between inferior constrictor and cricopharyngeal muscle
D. Between false and true vocal cords
E. None of the above

113. Patient with congenital anti thrombin 3 came to do an operation, what will you
prescribe for her postoperative
A. warfarin 6 month
B. warfarin for life
C. heparin for 6 months
D. heparin for life
E. None of the above

114. Target INR for warfarin is:


A. 1:2
B. 2:3
C. 3:4
D. 4:5
E. 5:6

115. Electrolyte disturbance that causes flat p wave and inverted T wave is:
A. Hypokalemia
B. Hyperkalemia
C. Hypocalcemia
D. Hypercalcemia
E. Hyponatremia

116. Which wave is coinciding with Ventricular repolarization?


A. P wave

40
B. QRS wave
C. T wave
D. ST segment
E. None of the above

117. Which diuretic acts on Na/k channel in collecting duct and causes hyperkalemia
A. Furosemide
B. Thiazide
C. Mannitol
D. spironolactone
E. None of the above

118. Creatinine clearance used to measure what?


A. GFR
B. RBF
C. Plasma blood flow
D. Tubular secretion
E. Tubular reabsorption

119. Renin acts on which one of the following


A. ADH
B. angiotensin 1
C. angiotensin 2
D. Angiotensinogen
E. None of the above

120. Diarrhea causing metabolic acidosis is due to loss of what?


A. HCO3
B. HCL
C. K
D. Ca
E. Na

121. If you do DRE, what will you palpate 4 cm from anal verge posterolateral
A. ischiospinous ligament
B. puborectalis muscle
C. sacrotuberous ligament
D. seminal vesicles
E. bladder

122. Patient complained from homonymous hemianopia, what is the most affected site in
brain?
A. optic nerve
B. optic chiasma

41
C. Lateral geniculate body
D. temporal lobe
E. parietal lobe

123. In tempo mandibular joint dislocation, the patient has severe pain around the joint what
is the nerve?
A. Auriculotemporal
B. Infra-alveolar
C. Great auricular
D. Great occipital
E. None of the above

124. Scenario of mastoid process infection with pus can cause destruction of which of the
following
A. inner ear
B. sigmoid sinus
C. occipital-mandibular
D. internal carotid artery
E. superior sagittal sinus

125. 35 years old woman presented with recurrent peptic ulcer. She is on proton pump
inhibitor and previously perceived h pylori eradication treatment. Which of the following is
likely to raise in her venous blood?
A. CCK
B. Secretin
C. Gastrin
D. Histamine
E. Pancreozymin

126. Defect in left pleuro-peritoneal membrane:


A. Morgagni hernia
B. Bochdalek hernia
C. Hiatus hernia
D. Ectopia cardia
E. None of the above

127. 4 Month child with failure to thrive with recurrent chest infection and dyspnea increase
with lying flat in the night and awake him from sleep?
A. TOF
B. Congenital diaphragmatic hernia
C. Asthma
D. GERD
E. None of the above

42
128. 1-day neonate presented by bilious vomiting and scaphoid abdomen
A. Esophageal atresia
B. CHPS
C. Duodenal atresia
D. HSD
E. Volvulus

129. 4-week girl presented by non-bilious vomiting and failure to thrive


A. Esophageal atresia
B. CHPS
C. Duodenal atresia
D. HSD
E. Volvulus

130.10 months boy is screaming with passing dark blood, sausage shaped abdominal mass in
examination with free DRE
A. Intussusception
B. Meckel's diverticulum
C. juvenile polyp
D. anal fissure
E. anal hemorrhoid

131. Patient has vertebral mass can cause respiratory compromise on which level?
A. mandible
B. cricoid
C. thyroid
D. hyoid
E. odontoid

132. A patient with Malignant melanoma. what is the best prognostic factor
A. .5 mm Breslow thickness
B. Free margins
C. No lymph nodes involved
D. No distant spread
E. None of the above

133. A patient had a velvety red lesion in the mouth. Biopsy revealed keratin. What is the
cause?
A. SCC
B. adenocarcinoma
C. Basal cell carcinoma
D. Keratoacanthoma
E. Seborrhic keratosis

43
134. Right coronary artery is a branch from?
A. Anterior of Ascending aorta
B. Arch of aorta
C. Posterior and left from ascending aorta
D. Posterior and right from ascending aorta
E. None of the above

135. In describing the sino-atrial node, which one of the following statements is correct?
A. it is part of the somatic nervous system
B. It is usually supplied by the left coronary artery
C. If lies in the wall of the left atrium
D. It lies in the wall of the right ventricle
E. It receives fibers derived from the vagus nerves

136. A 58-year-old man, admitted for inguinal hernia surgery, is found to have an enlarged
heart shadow on his chest radiograph. Which one of the/following statements is true of the
right ventricle?
A. Has a muscular wall that is normally thicker than that of the Left ventricle
B. Forms the apex of the heart
C. Forms the right border of the heart
D. Is directly related to the diaphragm
E. Gives rise to the aorta

137. A 32-year-old man presents to the acute surgical unit with acute pancreatitis. He
suddenly becomes dyspneic and his saturations are 89% on air. A CXR shows bilateral
pulmonary infiltrates. His CVP pressure is 16mmHg. What is the most likely diagnosis?
A. Pulmonary edema
B. Pneumococcal pneumonia
C. Staphylococcal pneumonia
D. Pneumocystis carinii
E. Adult respiratory distress syndrome

138. Posterior right rib fracture at the angle of t8 inward cause injury of?
A. Liver
B. rt lower lung
C. IVC
D. Pulmonary trunk
E. Hemiazygos vein

139. A 25-year-old man is admitted having been stabbed in the interior chest. On
examination he is alert and coherent. He has congested neck veins. His pulse is 140
beats/minute and his blood pressure 90/60 mmHg. He has normal breath sounds. What is the
most likely cause of his cardiac arrest 15 minutes later?
A. Cardiac tamponade

44
B. Congestive cardiac failure
C. Hypovolemia
D. Tension pneumothorax
E. Ventricular arrhythmia

140. Clinical scenario about esophageal varices, the anastomosis in lower esophagus is made
between which esophageal veins and …………
A. Left gastric vein
B. short gastric vein
C. Right gastric vein
D. Left gastroepiploic vein
E. Right gastroepiploic vein

141. All are content of the posterior mediastinum except:


A. Descending aorta
B. Thoracic duct
C. Azygous vein
D. body of vertebrae
E. splanchnic nerves to heart

142. A case of burn in 70 kg patient with 50% surface area affection what is the fluid
requirements during the 24 hours?
A. 16 liters
B. 14 liters
C. 11 liters
D. 9 liters
E. 7 liters

143. 72-year-old man is due to undergo an esophagectomy for malignancy. His BMI is 17.5.
What is the best feeding regime immediately following surgery?
A. Total parenteral nutrition.
B. Feeding jejunostomy.
C. Feeding duodenostomy.
D. Liquid diet orally.
E. PEG

144. A patient will do hemorrhoids and has HB of 9, he is put in a list after 6 weeks how
would you give him supplement?
A. blood transfusion
B. IV IRON
C. oral iron
D. vitamin B
E. folic acid

45
145. A child has got a Water burn (scald) covering whole dorsal forearm with erythema with
blisters, what is your management?
A. occlusive dressing
B. keep it open
C. split thickness skin graft
D. full thickness skin graft
E. flap

146. Male 35 years old has blood pressure is 110/80 and ICP is 18, what is cerebral perfusion
pressure
A. 52
B. 72
C. 92
D. 112
E. 132

147. Which center is stimulated in response to hypothermia?


A. Thalamus
B. hypothalamus
C. subthalamus
D. reticular formation
E. inferior olivary nucleus

148. First compensatory measures in the body toward shock?


A. baroreceptor stimulation
B. sympathetic stimulation
C. adrenaline activation
D. parasympathetic inhibition
E. renin activation

149. A drug used in induction anesthesia in shock for a patient came in trauma
A. ketamine
B. thiopentone
C. propofol
D. etomidate
E. All of the above

150. Isoprenaline has an inotropic effect through


A. alpha1
B. d1
C. d2
D. alpha2
E. beta1

46
151. Local anesthesia is given for biers block to a colle's fracture?
A. prilocaine
B. lidocaine
C. lidocaine with adrenaline
D. bupivacaine
E. xylocaine

152. Lignocaine common side effect?


A. Cardiotoxicity
B. Hepatotoxicity
C. Malignant hyperthermia
D. Histamine release
E. Renal damage

153. Depolarizing agent for anesthesia?


A. Succinyl choline
B. Vecuronium
C. Atracium
D. Pancuronium
E. Neostigmine

154. Which cell is dependent on Cori cycle for energy:


A. RBC
B. Muscle cell
C. Hepatocyte
D. Fat cells
E. All of the above
155. What is the cause of hemorrhagic pancreatitis?
A. Amylase
B. Lipase
C. trypsin
D. gastrin
E. glucagon

156. Which of the following is not considered a measure to enhance rapid recovery after a hip
surgery?
A. A carbohydrate rich solution 2 hours preoperatively
B. Preoperative anti emetic
C. No nasogastric tube
D. Limit IV fluids
E. Good analgesia

157. 35 years old patient after eating canned food presented by flaccid paralysis and
diagnosed as botulinum toxin affection. What is the mode of action of botulin toxin

47
A. Block release of acetyl choline from its vesicles
B. Block nicotinic cholinergic receptors
C. Block muscarinic cholinergic receptors
D. close ca release from sarcoplasmic reticulum
E. none of the above

158. Ampulla of Vater opening in?


A. In the posteromedial part of Proximal jejunum
B. In the posteromedial part of Third part of duodenum
C. In the posteromedial part of 2nd part of duodenum
D. In the posteromedial part of First part of duodenum
E. None of the above
159. Low pa O2 and High Pa CO2 occur in :
A. Type 1 respiratory failure
B. Type 2 respiratory failure
C. Respiratory alkalosis
D. Metabolic acidosis
E. Metabolic alkalosis

160. Skin incision for evacuation of an abscess and after healing scar overgrowth and a
painful mass rapidly growing and the Skin is immobile over lesion
A. epidermoid cyst
B. dermatofibroma
C. lipoma
D. dermoid cyst
E. pyogenic granuloma

161. Woman has a gunshot to her back, which transect cord at Lt side presents with features
suggestive of a Brown-Sequard syndrome. The clinical findings will include which of the
following ?
A. Left sided weakness, left sided proprioception and vibration loss, right sided loss of pain
and temperature
B. Right sided weakness, right sided proprioception and vibration loss, left sided loss loss
of pain and temperature
C. Right sided weakness, right sided proprioception and vibration loss, right sided loss loss
of pain and temperature
D. Right sided weakness, left sided proprioception and vibration loss,right sided of loss of
pain and temperature
E. None of the above

162. Inferior parathyroid gland develops from?


A. 1st pharyngeal pouch
B. 2nd pharyngeal pouch
C. 3rd pharyngeal pouch

48
D. 4th pharyngeal pouch
E. 6th pharyngeal arch

163. Vertebral artery true is enters the foramen transversum at which level?
A. C3
B. C4
C. C5
D. C6
E. C7

164. A man complains of being thirsty and getting up in the middle of the night to go to the
toilet. His weight is 85.5 kg, height 1.65 in and blood pressure 167/94 mmHg. An oral glucose
tolerance test was performed and produced the following results: Fasting plasma glucose
11.3 mmol/L The most likely diagnosis:
A. Diabetes insipidus
B. Diabetes mellitus
C. Impaired fasting blood sugar
D. sepsis
E. None of the above

165. The higher control of autonomic nervous system is


A. thalamus
B. hypothalamus
C. subthalamus
D. cerebellum
E. medulla

166. A 37-year-old man presents with severe headache, photophobia and neck stiffness.
Kernig's sign is positive and a midline lumbar puncture is performed immediately to
determine if a cerebrospinal fluid (CSF) pathogen is involved. In performing this procedure, at
which level we do this procedure?
A. At iliac crest
B. At anterior iliac spine
C. Transpyloric plane
D. Subcostal plane
E. None of the above

167. Inferior epigastric vein drains in:


A. Internal iliac vein
B. External iliac vein
C. Superior mesenteric vein
D. Inferior mesenteric vein
E. None of the above

49
168. 59 years old come with a spinal trauma and after neurological examination the physician
said that he has spasticity with hyperreflexia in his limb, what is the site of lesion
A. corticospinal tract lesion
B. lower motor neuron lesion
C. Cerebellar lesion
D. Basal ganglia lesion
E. Hypothalamus lesion

169. A 33-year-old female attends the day unit for elective varicose vein surgery. She has
previously had recurrent pulmonary embolic events. After the procedure she is persistently
bleeding. Her APTT is 52 (increased).
A. Factor V Leiden
B. Protein C deficiency
C. Disseminated intravascular coagulation
D. Antiphospholipid
E. None of the above
170. 34 years old pregnant woman develop swollen leg. her mother and her maternal aunt
have the same problem during pregnancies. Which of the following test will be positive in the
blood?
A. Antinuclear antibodies
B. Antiphospholipid antibodies
C. Anti-thyroglobulin antibodies
D. Antimitochondrial antibodies
E. None of the above

171. Caucasian girl with splenomegaly, jaundice and increase bilirubin level. What is the
diagnosis?
A. Spherocytosis
B. Sickle cell anemia
C. Intermittent porphyria
D. ITP
E. none of the above

172. A 71-year-old lady trips over and falls landing on her left shin. She sustains a large
pretibial laceration of her leg, wound dirty and rugged flap edges
A. Immediate split thickness skin graft
B. Delayed split thickness skin graft
C. Primary closure
D. Delayed primary closure
E. Random free flap

173. 6 cm Laceration on the scalp with dirty edges, treatment:


A. Flap
B. primary closure

50
C. healing by secondary intention
D. simple suture
E. Delayed closure with sutures

174. In cases of Unilateral cleft lip there is failure of fusion of?


A. Maxillary process and medial nasal process
B. Maxillary process and lateral nasal process
C. Frontonasal process
D. Palatine processes
E. None of the above

175. What is the embryological cause of vagino-rectal fistula?


A. Vittelointestinal tract
B. Urachus
C. Cloaca
D. Mesonephric duct
E. Metanephric duct

176. After clamping the umbilical cord what will not give oxygenated blood to the embryo
any more
A. Ductus venosus
B. Umbilical artery
C. Ductus arteriosus
D. Pulmonary artery
E. Pulmonary vein

177. A 32-years old man presents with a painful torticollis there is no past medical history and
his only complaint is that he has been feeling rather tired over the last 4 nights. On
examination, he has large rubbery mass in the lateral aspect of his neck as well as a few
smaller masses along his internal jugular vein. You correctly assume that the muscular neck
spasm and the large mass are connected. The most likely cause of his torticollis is due to
pressure on?
A. Ansa cervicalis
B. Cervical plexus branch
C. Cranial accessory nerve
D. Spinal accessory nerve
E. Vagus nerve

178. A case of steatorrhea due to gastrinoma what is the cause?


A. decrease lipase
B. whipple disease
C. gluten sensitivity
D. decrease bile secretion
E. increase output of juice in the duodenum and stomach

51
179. A 42-year-old multiparous woman is admitted to the Emergency Department due to
pelvic discomfort. The duty gynecologist diagnoses uterine prolapse. Which anatomical
structure gives significant direct support to the uterus?
A. Cervical ligaments
B. broad ligament
C. Round ligaments
D. Transverse perineal Muscles
E. None of the above

180. Rheumatoid lady day1 postoperative with hypotension and low sodium and high
potassium with long term steroids
A. Adrenal insufficiency
B. Cushing syndrome
C. Conn's disease
D. SIADH
E. Diabetes insipidus

181. Mechanism of Conserving water after hypovolemic shock


A. Dilation of afferent
B. Contraction of afferent
C. dilation of efferent
D. increase bowman's capsule permeability
E. None of the above

182. One takes ibuprofen for longstanding osteoarthritis. Twenty-four hours after emergency
embolectomy his blood pressure is 90/60 mmHg and he pass very dark brown urine. A
dipstick shows myoglobin and traces of blood. His blood glucose is 15 mmol/L (normal-4.0-
6.0), urea 12 mmol/L (normal 3.2-7.5) and creatinine 180 mmol/L (normal 35-110). What is
the most likely cause of his abnormal renal function?
A. Drug induced nephropathy
B. Hypovolemia
C. Methemoglobinemia
D. Rhabdomyolysis
E. Acute renal failure

183. A 65 years old man presented with an inguinoscrotal swelling in the right groin which is
non tender. A cough impulse is elicited, at operation, an indirect inguinal hernia is repaired.
The cremasteric muscle is derived from which of the following
A. External oblique aponeurosis
B. Internal oblique muscle
C. Rectus abdominal muscle
D. Rectus sheath
E. Transversalis fascia

52
184. Tourniquet used in operation to control blood loss and give bloodless field Is based on:
A. after exsanguination Inflation of cuff above diastolic
B. Inflation of cuff above mean before exsanguination
C. Inflation of cuff above mean before exsanguination
D. Inflation of cuff above systolic before exsanguination
E. Inflation of cuff above systolic after exsanguination

185. What is structure open during herniorrhaphy


A. external oblique Aponeurosis
B. internal oblique muscle
C. conjoint tendon
D. fascia transversalis
E. peritoneum

186. 35 years male presented by back pain and weakness of lower limbs due to central disc
prolapse with lower motor neuron lesion manifestation
A. cauda equina
B. syringomyelia
C. brown Sequard syndrome
D. posterior cord syndrome
E. anterior cord syndrome

187. During operation of superficial parotidectomy, facial nerve injury occurred inside the
operation and postoperative the patient suffered from all except
A. unable to close his eye
B. loss of corneal reflex
C. unable to elevate his eyebrow
D. numbness of the cheek
E. drop the angle of the mouth

188. A Patient presented with fracture of the face involving maxilla and numbness over the
cheek. Which of the following nerve is most likely involved?
A. Mental
B. Infra-orbital
C. Superior alveolar
D. Auriculotemporal
E. Supratrochlear

189. A 30 years old patient brought to ER with history of RTA his Echo showing cardiac
Tamponade for which pericardiocentesis was done what is the proper site of insertion?
A. Between Xephoid process and left sternocostal margin
B. Between Xephoid process and right sternocostal margin
C. Safety triangle in the chest

53
D. 2nd intercostal space
E. None of the above

190. A Young male come to the ER with crushed limb, by examination is tachycardic on 120 b-
min with Bp 110-70. By examining the limb, you notice that both bones of the leg are crushed
with completely avulsed popliteal artery, nerves are completely crushed with massive tissue
loss. What is the best choice for the patient?
A. Wait the tissues to gain viability
B. Interposition saphenous graft for popliteal artery repair
C. Interposition synthetic graft for popliteal artery repair
D. Amputation
E. None of the above

191. A 65-year-old male underwent a loop colostomy for emergency obstructing rectal tumor.
He came nowadays to do revision and anterior resection, what is the best investigation to
discover any leakage?
A. Barium enema
B. Unenhanced CT
C. Unenhanced MRI
D. Water Soluble contrast
E. Colonoscope

192. What is the most common biliary complications after lap cholecystectomy?
A. Bile leak from cystic duct stump
B. Common bile duct injury
C. Hepatic duct injury
D. Cholangitis
E. Duodenal injury

193. Male patient 58 years old with diabetes and chronic AF and come with sudden loss left
lower limb pain, by examination he has no distal pulsation below knee, what is the most
accepted cause of ischemia?
A. Chronic thrombosis
B. Embolic cause
C. Peripheral hypoperfusion
D. Normal Sequalae in old age
E. None of the above

194. A 6-year-old-boy with deteriorating health needs a blood transfusion, the parents
refused for religious reasons. What will you do?
A. Consent by proxy
B. Cannot proceed
C. Proceed without consent
D. Apply for a judge to a court

54
E. None of the above

195. A 26-year-old patient with down’s syndrome in need for appendicectomy, patient
understand all aspects of Surgery, from whom you will obtain the consent?
A. Consent the patient
B. Consent the parents
C. Cannot proceed
D. Proceed without consent
E. None of the above

196. A 63 years old patient complaining from Calf claudication on walking a short distance, by
examination there is loss of tibial arteries pulses, what is the most likley arterial occlusion
site?
A. External iliac artery
B. Common femoral artery
C. Superficial femoral artery
D. Popliteal artery
E. Anterior tibial artery

197. Male patient 39-year-old presented with sudden chest pain and hematemesis which
occurs after aggressive vomiting. By examination he has subcutaneous crepitus sensation
over the upper chest, what is the most accepted diagnosis?
A. Mallory Weiss syndrome
B. Borehave syndrome
C. Rupture trachea
D. Rupture Zenker diverticulum
E. Rupture pulmonary plebs

198. A 72-year-old man presents with dysphagia. He is investigated and found to have an
adenocarcinoma of the distal esophagus. His staging investigations have revealed a solitary
metastatic lesion in the right lobe of his liver.
A. Oesophagectomy
B. Endoscopic sub mucosal dissection
C. Photodynamic therapy
D. Insertion of oesophageal stent
E. None of the above

199. A 67-year-old lady presents with jaundice and abdominal pain. Her investigations show a
dilated common bile duct, a carcinoma of the pancreatic head compressing the pancreatic
duct. Her liver contains bi-lobar metastasis.
A. Gastrojejunostomy
B. Pancreatoduodenectomy
C. Conservative management
D. MRI guided pancreatic stent

55
E. Endoscopic pancreatic stent

200. A 54y male Patient with recurrent fainting attacks that improve by taking sugar drink, CT
abdomen show a pancreatic lesion of 3 cm size what is the best laboratory finding?
A. High CEA
B. High CA 19-9
C. High c-peptide
D. High Glucose
E. None of the above

201. A 70-year-old male resented with intestinal obstruction and diagnosed as Malignant
obstruction of the right colon. Surgical resection done and the pathology report show that
the malignancy involves all the submucosa with no invasion of the muscularis propria and
there are 2 nodes out of 24 is positive. PET scan showed that there is no Mets. What is the
TNM classification of the condition?
A. T1N0M0
B. T1N1M1
C. T1N1M0
D. T2N1Mo
E. T3N1M0

202. A 78-year-old man presents with symptoms of headaches and deteriorating vision. He
notices that there is marked pain on the right hand side of his face when he combs his hair.
What is the most likely diagnosis?
A. Giant cell arteritis
B. Wegener’s granulomatosis
C. Polyarteritis nodosa
D. Takayasu's arteritis
E. Buerger’s disease

203. A case of gangrene of the toe, with absent distal pulsation on examination using Doppler
USG which type of wave do you expect to find?
A. monophasic with low flow velocity
B. triphasic doppler signal
C. biphasic with low velocity
D. monophasic with high velocity
E. biphasic with high velocity

204. Right coronary artery is coming from:


A. post left part of ascending aorta above valve
B. post right part of ascending aorta above valve
C. anterior part of ascending aorta above valve
D. ascending aorta below valve
E. None of the above

56
205. A patient has anal incontinence after childbirth what is the best management step?
A. defecation defecography
B. anal manometry
C. colon transit time
D. CT with contrast
E. Colonoscope

206. Manifestation of Rt cerebellar lesion include all of the following except


A. Ataxia
B. Intention tremors
C. Rigidity
D. Unsteady gait
E. Dysmetria

207. A 34 years old presented by abdominal pain after open cholecystectomy. What is proper
management?
A. NSAID
B. Paracetamol
C. Epidural
D. Local lidocaine in the wound
E. None of the above

208. A 55 years old presented by ulcer on lateral border of tongue with cervical
lymphadenopathy, what do you think about the cause?
A. Adenocarcinoma
B. Basal cell cancer
C. Squamous cell cancer
D. Melanoma
E. Kaposi sarcoma

209. A 55 years old patient presented later with asymmetric smile after an operation in the
face. Which nerve is affected?
A. Trigeminal
B. Facial
C. Glossopharyngeal
D. Vagus
E. Hypoglossal

210. SVR high /CO low /CVP high


A. septic shock
B. neurogenic shock
C. cardiogenic shock
D. hemorrhagic shock

57
E. None of the above

211. SVR high /CO low /CVP low


A. septic shock
B. neurogenic shock
C. cardiogenic shock
D. hemorrhagic shock
E. None of the above

212. A patient developed extradural hematoma following a road traffic accident, on


examination he was found to have lateral squint and mydriasis, what nerve is most likely
affected?
A. Abducens
B. Oculomotor
C. Optic
D. Facial
E. Trochlear

213.2 years old child presented with Arnold chiari syndrome due to cerebellar tonsil
herniation and associated syringomyelia with affection of which tract
A. Dorsal column
B. Spinothalamic tract
C. Corticospinal tract
D. Extrapyramidal tract
E. None of the above

214. A patient has a pulsatile aneurysm related to a vessel at the level of greater trochanter.
What is that artery?
A. External iliac artery
B. common iliac artery
C. Inferior mesenteric artery
D. Femoral artery
E. Popliteal artery

58
September 2019 Recalls
1. A Child with kwashiorkor developed hepatomegaly. What is the deposit in liver?
A. Sarcoid
B. Melanin
C. Lipofuscin
D. Amyloid
E. None of the above

2. Patient had left iliac fossa pain and rectal bleeding. He was subsequently diagnosed with
sigmoid cancer. A decision was taken to embolize the responsible vessel. At which level does
it branch from aorta?
A. L1
B. L3
C. L5
D. L4
E. S1

3. TM joint is considered:
A. synovial bicondylar hinge joint
B. synovial ball and socket
C. synovial plane s
D. synovial pivot joint
E. none of the above

4. A 71-year-old lady trips over and falls landing on her left shin. She sustains a large pretibial
laceration of her leg leaving a dirty wound with rugged flap edges. What is the appropriate
wound management?
A. Immediate split thickness skin graft
B. Delayed split thickness skin graft
C. Primary closure
D. Delayed primary closure
E. Randomized free flap

5. Leg burn with deep wound ulcer 7 X 9 cm with tibia exposed. What is the best
management?
A. secondary intention
B. simple suture
C. immediate split thickness graft
D. delayed split thickness graft
E. flap
6. Caucasian girl with splenomegaly, jaundice and increase bilirubin level
A. Spherocytosis
B. Sickle cell anemia

59
C. Intermittent porphyria
D. G6PD
E. Hemophilia

7. Child with hydrocele, fluid accumulates in which layer


A. tunica vaginalis
B. tunica albuginea
C. superficial inguinal pouch
D. deep inguinal ring
E. none of the above

8. Recurrent pulmonary embolism despite of heparin use


A. LMWH
B. LMWH and stocking
C. Stocking
D. vena cava filter
E. None of the above

9. Bluish colored fluctuant swelling under tongue:


A. Ranula due to sublingual gland
B. small salivary gland tumor
C. submandibular gland tumor
D. thyroglossal cyst
E. branchial cyst

10. Weak sensation of lateral aspect of tongue after molar surgery.


A. Lingual n.
B. Glossopharyngeal nerve
C. Chorda tympani of facial nerve
D. Hypoglossal nerve
E. None of the above

11. What is the Cerebral perfusion pressure calculation if the BP 110/80 and ICP 18.
A. 72
B. 86
C. 92.
D. 96
E. None of the above

12. The following cells depends on Cori cycle for 02 completely.


A. Hepatocyte
B. Erythrocyte
C. Muscle cell
D. Cardiac muscle

60
E. None above the above

13. A patient had a dirty scalp laceration with no other skin loss. Management is:
A. wound excision and primary closure
B. simple suture
C. wound debridement with secondary intention healing
D. flap
E. none of the above

14. A man develops septicemia following surgery for perforated acute appendicitis He is
hypotensive. Arterial blood gases reveal: PH= (7.26) PaCo2= (7.2KPA) Pa02= (75 kPa).
Bicarbonate= (17 mmol/L) What is the most likely explanation for these readings ?
A. Compensated metabolic acidosis
B. Compensated respiratory acidosis
C. Mixed metabolic and respiratory acidosis
D. Uncompensated metabolic acidosis
E. Uncompensated respiratory acidosis

15. Hypercalcemia is firstly treated by


A. Iv saline chloride 0.9%
B. IV ca chelating
C. K supplementation
D. Pamidronate
E. Zoledronate

16. A case of suspected PE, what is the best investigation.


A. CTPA
B. VQ perfusion scan
C. Xray
D. ECG
E. D-Dimer

17. A 3 years child presented by hydrocele. Processes vaginalis comes from what structure
A. Peritoneum
B. Fascia transversalis
C. Internal oblique muscle
D. Cremasteric muscle
E. External oblique muscle

18. Which structure is opened during herniorrhaphy?


A. external oblique Aponeurosis
B. internal oblique muscle
C. conjoint tendon
D. fascia transversalis

61
E. peritoneum.

19. Anterior relation of head of pancreas


A. Pylorus of stomach.
B. CBD
C. Portal vein
D. Aorta
E. IVC

20. Right testicular vein drains into:


A. IVC
B. Rt renal vein
C. Right suprarenal
D. Right iiac vein
E. None of the above

21. Torticollis which nerve responsible for muscle which causing this Or Nerve supply of
sternocleidomastoid muscle?
A. Glossopharyngeal
B. Trigeminal
C. Vagus nerve
D. Spinal accessory nerve.
E. None of the above

22. What is the parasympathetic ganglion that supply the lacrimal gland?
A. geniculate ganglion
B. Pterygopalatine ganglion
C. Otic ganglion
D. Submandibular ganglion
E. None of the above

23. A man with prostatism, On PR, posterolateral ridge on palpation. What structure?
A. Sacrospinous ligament
B. Puborectalis muscle
C. Ischiorectal fossa
D. Obturator internus muscle
E. None of the above

24. On PV examination, you feel something in front at the level of Cervix


A. base of bladder
B. dome of bladder
C. Rectum
D. Uterus
E. None of the above

62
25. A man with BP 200/100 and potassium 2.1. What you expect to be found in his
investigation?
A. High aldosterone
B. high cortisol
C. low ACTH
D. High urinary catecholamines.
E. High calcitonin level

26. A man with RTA had 10/15 GCS and also taken unit blood transfusion now has low
platelet, normal APTT, low fibrinogen and high PT? What is the diagnosis?
A. DIC
B. Deficiency of anti-thrombin III
C. Thrombin deficiency
D. Factor 5 leyden
E. None of the above

27. A man underwent esophagogastrostomy for cancer esophagus. Post-operative period he


lost significant weight . what should you do?
A. TPN
B. Nasogastric tube feeding
C. Intraoperative jejunostomy
D. Gastrostomy
E. None of the above

28. A patient with cervical lymphadenopathy was investigated and FNAC suggested squamous
Cell carcinoma. Clinical examination and upper tract endoscopy were normal. What should be
the next line of investigation?
A. CT
B. MRI
C. FDG Pet CT (PET CT scan)
D. MRI of whole body
E. Radio-isotope investigation

29. What is the parasympathetic ganglion that supply the lacrimal gland?
A. geniculate ganglion
B. Pterygopalatine ganglion
C. Otic ganglion
D. Submandibular ganglion
E. Satellite ganglion

30. A young boy with sudden mildly tender swelling in dorsum of wrist
A. ganglion cyst
B. lipoma

63
C. aneurism
D. neurofibroma
E. sebaceous cyst

31. A woman noticed blood spot in the pants with revealed inguinal lymph nodes. what is the
Origin?
A. Vulva
B. Cervix
C. Body of Uterus
D. Fundus of uterus
E. None of the above

32. ECG changes in pulmonary embolism


A. ST elevation in V1V2V3
B. Wide QRS
C. Inverted T in V1V2V3
D. Hyperacute T wave
E. Left ventricular hypertrophy

33. Level of lumbar cistern CSF fluid?


A. L1
B. L2
C. S1
D. S2
E. T12

34. RTA patient admitted after observation then discharged by giving treatment. 10 days later
came with abdominal discomfort especially after eating, it became worse. What is the cause?
A. Diaphragm rupture
B. diaphragmatic hernia
C. curling ulcer
D. acute gastric dilatation
E. esophageal perforation

35. Recurrent DVT despite of heparin. What is the best prophylaxis?


A. LMWH
B. LMWH and stocking
C. Vena cava filter
D. Warfarin long term
E. None of the above

36. Patient with IV fluid having warming device attached during surgery which hormone
secretion decrease?
A. TSH

64
B. ACTH
C. ADH
D. Cortisol
E. Catecholamine

37. The "fight or flight", response produces a release of epinephrine (adrenaline). What is the
primary metabolic effect of epinephrine?
A. Alanine shunt activation
B. Cortisol release
C. Glycolysis
D. Tachycardia
E. Vasoconstriction

38. All are true about Bell's palsy except:


A. Dropping of the eye lid
B. Dryness of the eye
C. Dropping of the mouth
D. Pain in the ear
E. Numbness of cheek

39. 4 Month child with failure to thrive with recurrent chest infection and dyspnea increase
with lying flat in the night and awake him from sleep
A. TOF
B. Congenital diaphragmatic hernia
C. Asthma
D. GERD
E. None of the above

40. 1-day neonate presented by bilious vomiting and scaphoid abdomen


A. Esophageal atresia
B. CHPS
C. Duodenal atresia
D. HSD
E. Volvulus

41. Position of CBD papilla opening in duodenum in ERCP?


A. anterior & medial,
B. anterior & lateral
C. posterior and medial
D. posterior & lateral
E. none of the above

42. A patient with ureteric colic, pain radiates into loin to groin-Why? Which dermatome?
A. T11 to L1

65
B. T12
C. L2
D. L2-3-4
E. S2-3-4

43. What is not included in enhanced post op program:


A. Carbohydrate drink 2 hours before.
B. Intra operative antiemetics.
C. Preoperative diazepam
D. Early postoperative ambulation
E. Early removal of drains

44. In obstetric surgery, Pfannenstiel's incision was made. which structure is divided?
A. Rectus abdominis
B. arcuate ligament
C. external oblique
D. posterior rectus sheath
E. none of the above

45. Irritability, muscle twitching in postoperative period of thyroidectomy


A. hypocalcemia
B. hypokalemia
C. hyponatremia
D. hypercalcemia
E. hyperkalemia

46. A child with distended abdomen, pain, vomiting, groin swelling...


A. Indirect Inguinal hernia
B. Direct inguinal hernia
C. Hydrocele
D. Undescended testis
E. None of the above

47. Deep inguinal ring found in:


A. Fascia transversalis
B. Internal oblique muscle
C. Cremasteric muscle
D. External oblique muscle
E. None of the above

48. Action on which of the following receptor is responsible for the depressant action in an
old man who underwent laparotomy and is on Morphine with RR of 6/min?
A. Mu receptor
B. Delta

66
C. Dopamine
D. Serotonin
E. None of the above

49. A patient with bee sting suddenly developed shock. Which first?
A. IM adrenaline
B. IV hydrocortisone
C. IV antihistamine
D. Prednisolone
E. IV fluids

50. A 42-year-old multiparous woman is admitted to the Emergency Department due to


pelvic discomfort. The duty gynecologist diagnoses uterine prolapse. Which anatomical
structure gives significant direct support to the uterus?
A. Cervical ligaments
B. broad ligament
C. Round ligaments
D. Transverse perineal Muscles
E. None of the above

51. Origin of Rt Coronary artery from:


A. anterior aspect of ascending aorta above valve
B. right posterior aspect of ascending aorta above valve
C. inferiorly below cusp of aortic valve
D. aortic arch
E. none of the above

52. A girl had mastoid abscess. Spread of infection to


A. sagittal sinus
B. sigmoid sinus
C. superior sagittal sinus
D. internal ear
E. none of the above

53. Patient with HB 9gm%. plan for surgery after 6wk what is preferred?
A. blood transfusion
B. IV iron
C. oral iron
D. Folic acid
E. none of the above

54. Closure of the tricuspid valve occurs at the onset of which phase of the cardiac cycle?
A. Isovolumic relaxation
B. Rapid and reduced filling

67
C. Rapid and reduced ejection
D. Isovolumetric contraction
E. None of the above

55. A 26-year-old man is admitted to the Emergency Department with multiple peripheral
fractures. He is clinically shocked. Which is the structure responsible for the first hemostatic
response to a fall in systemic arterial blood pressure?
A. Adenohypophysis
B. Baroreceptor
C. Chemoreceptor
D. Renin
E. Neurohypophysis

56. Gastrin release is increased by:


A. Adrenaline
B. Glucagon
C. Calcitonin
D. Acetylcholine
E. None of the above

57. A previously healthy young male wakes up in the morning with sudden SOB with fall in
spo2, low BP ,tachycardia and absent breath sounds on the left side what is the probable
diagnosis
A. spontaneous pneumothorax
B. Pneumonia
C. Pleural effusion
D. Hemopneumothorax
E. pericardial effusion

58. A man with left loin colicky pain. Imaging show stone impaction at the Vesicoureteral
junction. Where it is visible?
A. Ischia tuberosity
B. Ischial spine
C. Sacroiliac joint
D. L2-5 transverse process
E. None of the above

59. Male patient admitted in ICU after RTA. He was intubated as his GCS 7 with admission,
monitor showing rapid decrease in po2 after leg movement what is the cause of this
condition
A. Pneumothorax
B. Spinal cord injury
C. Slipped endotracheal tube
D. Respiratory center depression

68
E. Cardiac tamponade

60. Facial injury cause numbness in cheek and upper 3 teeth. Which nerve is affected?
A. Supratrochlear
B. Supraorbital
C. Infratrochlear
D. Infraorbital
E. None of the above

61. Unique features of axis vertebrae.


A. bifid spinous process
B. carotid tubercle(C6)
C. no transverse process
D. no spinous process
E. none of the above

62. What is true about the ureter?


A. Ureter passes below aortic bifurcation
B. Ureter passes above uterine artery
C. Ureter is medial to gonadal vessels
D. Ureter is lateral to gonadal vessels
E. All of the above

63. 55 years old male will go for total knee arthroplasty and you need a Tourniquet
application to have a bloodless field to which level you will adjust the Tourniquet pressure?
A. Above the mean arterial pressure
B. Above systolic blood pressure
C. Equal to the mean arterial pressure
D. Above diastolic arterial pressure
E. None of the above

64. Which type of lymphoma is associated with coeliac disease?


A. T-cell lymphoma
B. B- cell Lymphoma
C. Burkitt's lymphoma
D. Mantle cell lymphoma
E. None of the above

65. Which statement relating to the peri operative management of patients with diabetes
mellitus is false?
A. They should be placed first on the operating list
B. An intravenous sliding scale should be used in all cases
C. Potassium supplementation is likely to be required in diabetics on a sliding scale
D. Electrolyte abnormalities are more common after major visceral resections

69
E. None of the above

66. A female 30 years old, developed pain between 3rd and 4th toes with no history of
trauma. It is elicited by vertical compression on metatarsal heads. What is the most likely
diagnosis?
A. Stress fracture
B. Freiburg disease
C. Morton neuroma
D. Osteoarthritis
E. None of the above

67. A patient developed acromioclavicular joint dislocation after trauma during rugby game.
Xray showed no fracture. What is the expected ligamentous injury?
A. Coracoacromial ligament injury
B. Coracoclavicular ligament injury
C. Deltoid ligament injury
D. Ligamentum teres injury
E. None of the above

68. What is the causative organism of septic arthritis in patients with sickle cell anemia?
A. Staph. Aureus
B. Strept. pyrogens
C. Salmonella
D. Neisseria
E. Pseudomonas

69. Which dermatome that supplies thumb and index fingers?


A. C6
B. C8
C. T1
D. C5
E. C7

70. A 36 years old heavy lifter complained of back pain radiating to his leg. MRI was ordered
and result is as shown in the picture. What is the level of lesion?

70
A. L3-4
B. L4-5
C. L5-S1
D. L2-3
E. S1-1
71. A 53 years old female complained of stitches and pain in her hands especially at rest.
Histology of the lesion would probably show:
A. Needle shaped crystals
B. Rhomboidal crystals
C. Granulomatous necrobiosis
D. Apple green perfringens
E. Asteroid bodies

72. A 56 years old male presented with painless hematuria, back pain with Xray showing
multiple osteolytic lesions in vertebrae alongside with Lung lesions. What is the most likely
cause?
A. Small cell lung cancer
B. Prostatic cancer
C. Renal cell carcinoma
D. Neuroblastoma

71
E. Adenocarcinoma of the lung

73. A 45 years old man complained of pain in his knee. His lab investigation revealed Ca of 8.6
and uric acid of 26 (N: 3.5-7). Histology of the knee aspirate would probably show:
A. Needle shaped crystals
B. Rhomboidal crystals
C. Granulomatous necrobiosis
D. Apple green perfringens
E. Asteroid bodies

74. A 53 years old man is due to undergo Cholecystectomy. His swap revealed that he is a
nasal carrier of MRSA. What is the most appropriate treatment?
A. Chlorhexidine Daily bath
B. IV vancomycin
C. Mupirocin nasal spray
D. IV rifampicin
E. Oral teicoplanin

75. A 12 years old boy was brought by his mother with a sudden onset scrotal pain, visible
black spot above testes. What is the most appropriate diagnosis?
A. Testicular torsion
B. Hematocele
C. Morgagni cyst torsion
D. Epidydimoorchitis
E. Varicocele

76. A patient is undergoing an inguinal hernia repair. Postoperatively, the patient has
weakness in his leg with lost extension of knee. Which nerve is affected?
A. Ilioinguinal nerve
B. Genitofemoral nerve
C. Femoral nerve
D. Obturator nerve
E. Sciatic nerve

77. A 6 years old child presented with short left leg associated with knee pain for the last 3
months. What is the most likely diagnosis?
A. Perthes disease
B. Slipped femoral cap
C. Developmental dysplasia of the hip
D. Septic arthritis
E. Osteogenesis imperfecta

78. A 13-year-old obese child is brought by his mother with limited internal rotation. External
rotation to limb causes pain. What is the most likely diagnosis?

72
A. Perthes disease
B. Slipped femoral capitulum
C. Developmental dysplasia of the hip
D. Septic arthritis
E. Osteogenesis imperfecta

79. A 32 years old man sustained an elbow trauma during his work. His examination revealed
severe pain on extremes of pronation and supination. What is the most likely injured nerve?
A. Radial nerve
B. Anterior interosseus nerve
C. Posterior interosseus nerve
D. Median nerve
E. Ulnar nerve

80. A 35 years old male undergone open reduction and internal fixation for a complex tibial
fracture. Postoperatively, he became dyspneic, tachycardic with skin rash. What is the most
likely diagnosis?
A. Fat embolism
B. Pulmonary embolism
C. ARDS
D. Pulmonary infarction
E. Pneumonia

81. A 56-year-old man with chronic emphysema is on the high dependency unit (HDU), ten
days after anterior resection. He has developed acute shortness of breath' and hypotension):
Which of the following landmarks would be the most appropriate to use to obtain a femoral
arterial blood gas sample?
A. A point midway between the anterior superior iliac spine and the pubic tubercle
B. A point midway between the anterior superior iliac-spine and the pubic symphysis
C. A point midway between the greater trochanter and the pubic symphysis
D. A point 1 cm inferior and 4 cm lateral to the pubic tubercle
E. A point 2 cm lateral to the midway point between the anterior superior iliac spine and
pubic symphysis

82. A 41 years old man was playing football when he felt sudden pain in the calf, he could not
continue the game. Squeezing on calf muscles produced no planter flexion. What is the most
likely diagnosis?
A. Biceps muscle rupture
B. Posterior cruciate ligament injury
C. Achilles tendon rupture
D. Gastrocnemius tear
E. Ruptured baker cyst.

73
83. A 32 years old man was playing football when he fell on his knee. The joint got swollen
with pain, so he was prescribed rest for 4 weeks. After 4 weeks of rest he returned to play
again. He felt the same pain and the joint got swollen. What is the most likely diagnosis?
A. Medial meniscus tear
B. Anterior cruciate ligament injury
C. Medial collateral ligament injury
D. Posterior cruciate ligament injury
E. Loose body ligament injury

84. What is the most accurate relation of median nerve with brachial artery in the arm?
A. Medial to it
B. Medial then anterior then lateral
C. Lateral then anterior then medial
D. Posterior to it
E. Deep to it

85. A man presented after painting of ceiling with shoulder pain. The pain is mainly during
movements of shoulder between 60-120 degrees. What is most likely diagnosis?
A. Subacromial bursitis
B. Supraspinatus tendinitis
C. Frozen shoulder
D. Shoulder dislocation
E. None of the above.

86. A 35 years old female with pruritus ani. Sellotape examination shows egg in stool. What is
the most likely causative organism?
A. Enterovirus vermicularis
B. Ascariasis
C. Entamoeba histolytica
D. Giardia lambia
E. Cryptosporidium

87. During a pelvic surgery, A nerve was injured emerging from the lateral aspect of psoas
major causing lost sensation over the lateral aspect of the thigh. What is the corresponding
nerve root of the affected nerve?
A. L1-2
B. L2-4
C. L2-3
D. L3-4
E. L1-3

88. A 32 years old woman presented with perianal itching. Stool analysis showed cyst and
worms. What is the most appropriate treatment?
A. Mebendazole

74
B. Tinidazole
C. Metronidazole
D. Bithionol
E. None of the above

89. A middle age man with sub capital femoral neck fracture with minimal displacement.
What is the most appropriate management?
A. Internal fixation with nail
B. Internal fixation with plate
C. External fixation
D. Closed reduction and cast
E. Total hip replacement

90. A 32 years old man suffered RTA. On examination, there is a grossly open dirty
comminuted fracture of tibia. What is the most appropriate management?
A. Wound debridement and immediate external fixation
B. Wound debridement and internal fixation
C. Wound debridement and delayed external fixation
D. Closed reduction and cast

91. A 53 years old patient was admitted to ICU and received antibiotics. He subsequently
developed severe diarrhea with crampy abdominal pain. What is the appropriate treatment?
A. Oral vancomycin
B. Oral metronidazole
C. IV metronidazole
D. Oral mebendazole
E. None of the above

92. A patient suffered a trauma to his thigh leaving a dirty wound with lacerations. Several
days later the wound becomes infected with crepitations. Which Organism?
A. Clostridium deficile
B. Staphylococcus aureus
C. Streptococcus pyogenes
D. Clostridium perfringens
E. Bacillus anthracis

93. A patient undergone perianal surgery in a Lloyd Davis position. After operation, She
complained foot drop which subsequently improved. What is the most likely cause?
A. Neuropraxia of common peroneal nerve.
B. Neuropraxia of tibial nerve.
C. Neurotmesis of common peroneal nerve.
D. Neurotmesis of tibial nerve.
E. Axonotmesis of common peroneal nerve

75
94. After varicose vein surgery, lateral part of foot became numb in a 40 years old female.
What is the most likely affected nerve?
A. Saphenous nerve
B. Tibial nerve
C. Sural nerve
D. Lateral cutaneous nerve of the thigh
E. None of the above

95. A 32 years old female came to clinic with a gradual onset swelling at the lateral aspect of
knee. She reports no history of trauma or family history. What is the most likely cause?
A. Meniscal cyst
B. Loose body
C. Medial meniscus tear
D. Chondromalacia patellae
E. Lateral collateral ligament injury

96. A 53 years old farmer presented with blood tinged dark spot with eschar on his hand
dorsum after injury. He is feverish and severely toxic. What is the most likely causative
organism?
A. Streptococcus pyogenes
B. Bacillus Anthracis
C. Staphylococcus aureus
D. Clostridium perfringens
E. Bacteroides

97. A 32 years old man reported tingling sensation over the lateral 2/3 of the palm with weak
hand flexion. What is the most accepted causative trauma?
A. Scaphoid fracture
B. Hamate dislocation
C. Lunate dislocation
D. Distal radial fracture
E. None of the above

98. A 20-year-old male, with history of trauma to his groin 6 weeks ago, now presented with
irregular hard testicular swelling with fluid around it. The swelling trans-illuminated. He has
enlarged left supraclavicular lymph node. What is the most likely diagnosis?
A. Hydrocele
B. Hematoma
C. Teratoma
D. Torsion testis
E. Seminoma

99. A 22-year-old female patient presented with bloody nipple discharge with no palpable
masses in the breast. What is the most probable diagnosis?

76
A. Intraductal papilloma
B. Duct ectasia
C. Fibro adenosis
D. Breast cancer
E. Paget disease

100. A 45-year-old lady with persistent creamy discharge from breast. What is the diagnosis?
A. Intraductal papilloma
B. Duct ectasia
C. Fibro adenosis
D. Breast cancer
E. Paget disease

101. A 53 years old Lady noticed an anterior neck swelling. She has a pulsatile mass located in
her forehead. What is the most likely cause?
A. Lymphoma
B. Medullary thyroid cancer
C. Follicular thyroid carcinoma
D. Papillary thyroid carcinoma
E. Thyroglossal cyst

102. A 53 years old man taking carbimazole for hyperthyroidism suddenly developed
agitation, tachycardia and elevated blood pressure upon emotional stress. What is the likely
cause?
A. Carbimazole overdose
B. Hypothyroidism
C. Hyperthyroidism
D. Thyrotoxic crisis
E. None of the above

103. Female patient has Mediastinal mass biopsy taken from the mass which showing- Hassall
corpuscles what the most probably diagnosis?
A. Lymphoma
B. Follicular carcinoma
C. Schwannoma
D. thymoma
E. none of the above

104. During parathyroidectomy, the surgeon wants to identify the parathyroid gland, Frozen
section was done. What is the best answer describing the Frozen section??
A. Put the specimen on paraffin
B. But stain firstly then put the specimen
C. Using labeled fluorescence
D. Put cells on slide then stain with hematoxylin and eosin

77
E. Put the specimen in formaldehyde

105. A young woman presented with breast cyst aspirated was done until become dry and
cytology showing NAD what is your management
A. Discharge No follow up.
B. culture of aspirated fluid
C. excision of cyst
D. benzidine test
E. take biopsy

106. A young woman with 2 cm size right thyroid swelling. Biopsy was taken and showing
papillary carcinoma with no metastasis. What is the best Treatment for this case?
A. Right hemithyroidectomy.
B. Subtotal thyroidectomy.
C. Total thyroidectomy
D. Total thyroidectomy with lymph node clearance
E. Radioactive iodine

107. A pregnant woman 6 weeks with presented with palpitation, heat intolerance and
swelling in her neck what is the best treatment for here?
A. Propranolol
B. Thyroidectomy
C. Propylthiouracil
D. Carbimazole
E. Radioactive iodine

108. A 33-year-old lady presented with thyroid swelling in her neck and high calcitonin what
is your possible diagnosis?
A. medullary carcinoma
B. follicular carcinoma
C. papillary carcinoma
D. lymphoma
E. anaplastic carcinoma

109. A 33-year-old female patient has forearm wound. what is the most important
component in 2nd phase healing?
A. Vitamin A
B. Vitamin B12
C. Hyaluronidase enzyme
D. Vitamin D
E. Vitamin C

110. A 23-year-old patient present to E/R department with Head trauma after examination he
has depressed fracture. What is your management?

78
A. Refer to neurosurgery
B. CT head within 2 hours
C. CT head within 8 hours
D. MRI brain
E. None of the above

111. An 28 years old patient stabbed on right side posterior to axillary line, x ray showing
pneumothorax with fluid level. What is the best management?
A. Chest drain with suction
B. Needle Decompression
C. thoracentesis
D. Chest drain without suction
E. None of the above

112. A 53 years old woman presented with cervical LN, on examination, supraclavicular,
axillary and inguinal LN all enlarged. What is your possible Diagnosis?
A. Lymphoma
B. TB
C. Cancer breast
D. Adenocarcinoma in the lung
E. None of the above

113. A 16-year-old child presented headache and discharge from medial canthus of left eye
what is your possible diagnosis?
A. Ethmoidal sinusitis
B. Maxillary sinusitis
C. Carcinoma in ethmoid sinus
D. Carcinoma in maxillary sinus
E. One of the above

114. A man with Ca prostate. His mother and sister have Cancer breast, father healthy, what
is the genetic association of this condition?
A. P53
B. K-Ras
C. BRCA1 and 2
D. MYC
E. Sis

115. A 40-year-old man presented to hospital after RTA. His GCS was 10 and his C.T showing
crescent shaped lesion. What is the cause of this condition?
A. Chronic subdural hematoma
B. Subarachnoid hemorrhage
C. Intra ventricular hemorrhage
D. Brain concussion

79
E. acute subdural hematoma

116. A Man hits his head against door, then loses consciousness for 15 sec. Examined, found
to be ok. Management?
A. Discharge home with instructions on head injury,
B. Call neurosurgeon
C. Hospitalize only if old age
D. Brain CT
E. None of the above

117. A female patient presented with thyroid swelling after examination her BP was 200/100
mmHg what is your Diagnosis?-
A. MEN IIA
B. Pheochromocytoma
C. Graves diseases
D. Medullary thyroid carcinoma
E. MEN IIB

118. A 55-year-old woman with increased TSH with Normal T4 and low T3 what is your
diagnosis?
A. primary hypothyroidism
B. Grave's disease
C. Secondary hypothyroidism
D. Sick euothyroid
E. Hyperthyroidism

119. Woman with increased TSH with Normal T4 . What is your diagnosis?
A. primary hypothyroidism
B. Grave's disease
C. Secondary hypothyroidism
D. Subclinical hypothyroidism
E. None of the above

120. Female patient has a 3cm mass in her breast what is the Best biopsy method for this
mass?
A. FNAC
B. Incisional biopsy
C. Excisional biopsy
D. Punch biopsy
E. Core biopsy

121. A 23-year-old female presented with solitary thyroid nodule what is the Best way to
biopsy for thyroid?
A. FNAC

80
B. Incisional biopsy
C. Excisional biopsy
D. Punch biopsy
E. Core biopsy

122. Male patient has in his investigations Urinary Bence Jones proteins (24h urine sample)
what is the most probably diagnosis?
A. Osteoblastoma
B. Myelofibrosis
C. Lymphoma
D. Leukemia
E. Multiple Myeloma

123. A man has cervical, inguinal and axillary lymph node enlargement his pathology showing
Hodgkin lymphoma. Which cell in Lymphoma originate?
A. B cells
B. T cells
C. NK cells
D. Eosinophils
E. Neutrophils

124. A man has a wound in his leg What is the early response after injury.
A. mast cells migration
B. eosinophils accumulation
C. cytokines release
D. N Killer cell release
E. Neutrophils

125. Which of the following is considered a positive test in cases of DVT?


A. Homan test
B. Hallux extension test
C. Babinski sign
D. Painful calf muscle on inversion or version
E. None of the above

126. DVT prophylaxis for patient with IHD and peripheral vascular disease:
A. Stocking
B. Unfractionated heparin 1000-unit hourly
C. LMWH
D. Stocking and LMWH
E. LMWH with pneumatic compression

127. DVT prophylaxis regarding patient with antithrombin 3 deficiency?


A. warfarin lifelong,

81
B. warfarin 3 or 6 months
C. IVC filter
D. LMWH
E. Stocking

128. A 46-year-old lady underwent Lap Cholecystectomy for chronic calcular cholecystitis, 1
months later she presented by progressive unremitting jaundice with pale stool, dark urine.
What is the most probable diagnosis?
A. Bile duct injury
B. Collection on the surgical bed of the gallbladder
C. Residual CBD stone
D. Acute pancreatitis
E. None of the above

129. You are conducting a cross sectional study measuring BP in certain population, what is
the type of the data that you will obtain?
A. Ordinal
B. Nominal
C. Continuous
D. Discrete
E. None of the above

130. Male Patient 54 years has no past medical history complained from severe abdominal
pain and sudden shocked, he is diagnosed as ruptured abdominal aortic aneurysm, what is
the best action?
A. CT abdomen with contrast
B. Urgent laparotomy
C. Duplex ultrasound
D. Best supportive care without intervention
E. None of the above

131. Female patient 45 years old complained from ischemic manifestation in her lower limb
with distal gangrene in the toes, theses event occurs following AAA repair, what is the most
common cause of the gangrene?
A. Arterial embolism
B. Atherosclerosis
C. Propagated Thrombus
D. Foreign body aneurysm
E. Vasospasm following surgery

132. A case of gangrene of the toe, with absent distal pulsation on examination using Doppler
USG which type of wave do you expect to find?
A. monophasic with low flow velocity
B. triphasic doppler signal

82
C. biphasic with low velocity
D. monophasic with high velocity
E. biphasic with high velocity

133. Elderly patient is diagnosed as having abdominal aortic aneurysm, which is considered
the most common cause of this condition?
A. Atherosclerosis
B. Marfan’s syndrome
C. Thrombosis
D. Autoimmune disease
E. None of the above

134. A case of intermittent claudication in the calf muscles with loss of pulse below knee only,
where is the most probable site of the arterial lesion?
A. SFA occlusion at the adductor canal
B. CFA before origin of profunda artery
C. External iliac artery
D. Anterior tibial artery
E. Internal iliac artery

135. An alcoholic man admitted with severe chest pain and hematemesis that occurs after
vigorous attack of vomiting. By examination he has crepitations in the anterior chest wall.
What is the most probable diagnosis?
A. Complete esophageal rupture
B. Mallory Weiss tear
C. Pneumothorax
D. Fracture ribs
E. None of the above

136. A patient presented with calf pain on walking 50 meter. Diagnosed as Claudication. Block
of which level is responsible?
A. Superficial femoral
B. popliteal
C. deep femoral
D. external iliac
E. profunda femoris

137. Male patient with left iliac fossa pain for the past 6 wks. with increased frequency of
defecation with attacks of fresh rectal bleeding. No significant weight loss. What is the likely
diagnosis?
A. Cancer sigmoid colon
B. IBD
C. Diverticulitis
D. Irritable bowel disorder

83
E. None of the above

138. A 56 patient after colorectal surgery. What investigation is done to rule out anastomotic
leak?
A. CT with contrast
B. MRI
C. Barium Enema
D. water soluble double contrast enema
E. Colonoscopy

139. A 10 years boy presented in RTA obviously internal hemorrhage mostly due to rupture
spleen and he is hemodynamically unstable, and you decide to procced for surgery, From
whom you will obtain your consent?
A. Consent the patient him self
B. Consent the parents
C. Proceed without consent
D. Two consultant consent
E. Apply to the court

140. A 27-year-old has had pancolitis for the last five years. Which one of these complications
is the least likely to develop?
A. Arthritis
B. Cholangitis
C. Iritis
D. Polyarteritis
E. Toxic megacolon

141. A 26-year-old patient with Down’s syndrome, in need for appendicectomy, patient
understand all aspects of Surgery?
A. Consent the patient
B. Consent the parents
C. Cannot proceed
D. Proceed without consent
E. Apply to court

142. A 22-year-old male with motorcycle injury he is unstable, unconscious, and needs urgent
surgical operation, from whom you take the consent?
A. Consent the patient
B. Consent the parents
C. Cannot proceed
D. Proceed without consent
E. Apply for a judge to a court

84
September 2020 Recalls
1. A patient developed severe watery diarrhea while he is in at ICU after a surgery for IBD,
colonoscopy was done and revealed a yellowish plaque affecting the colon. What is the
causative organism?
A. Streptococcus Bovis
B. Streptococcus Pyrogens
C. Staphylococcus aureus
D. Clostridium deficile
E. Campylobacter

2. A male patient in Africa presented with a swollen scrotum associated with chyluria. There
is no other symptoms. What is the most likely cause?
A. Schistosoma
B. E.coli
C. Staphylococcus aureus
D. Wuchereria Bancrofti
E. Bacteroides

3. A 49-year-old Egyptian man on holiday in the United Kingdom presents with haematuria.
He gives a history of bladder irritative symptoms for seven months. He has lived in a rural
community for his whole life. What is the possible diagnosis?
A. Cystitis cystica
B. Interstitial cystitis
C. Multiple superficial transitional cell carcinomas
D. Muscle invasive transitional cell bladder cancer
E. Squamous cell carcinoma of the bladder

4. A man developed severe gangrene and color change affecting perineum with extension of
necrotic tissue to the groin. Which bacteria causing this condition?
A. Staphylococcus aureus
B. E coli
C. Mixed aerobe and anaerobe
D. Streptococcus pyogenes
E. Clostridium perfringens

5. A 45 years old man came with a 24-hours history of severe knee pain, cannot move the leg
in any direction, he is apyrexial, WBC count normal, what is the most definitive investigations
to know the cause:
A. Arthroscopy
B. knee aspiration
C. ESR
D. Xray
E. USG

85
6. A middle-aged man was involved in a motorboat accident, he sustained a separation of
lumbar spine from sacral spine, with associated lumbar nerve root injury. Which of the
followingmovements will be affected?
A. Hip flexion
B. Knee extension
C. foot inversion
D. Planter flexion
E. Hip adduction

7. A 25 years old athlete came with a single rib fracture. He is in pain, but he is vitally stable.
What is the most appropriate management?
A. Chest drain
B. Admission, observation and analgesia
C. Analgesia and discharge
D. Infiltration of wound with 5% bupivacaine
E. None of the above

8. A 59 years old COPD patient came with a rib fracture. X-ray was done with no hemo or
pneumothorax detected, but the patient is in pain. What is the most appropriate
management?
A. Chest drain
B. Admission, observation and analgesia
C. Analgesia and discharge
D. Infiltration of wound with 5% bupivacaine
E. None of the above

9. A 34 years old patient presented after chest trauma. The trauma affected the ribs from 5th
to 10th which were fractured. The patient is cyanosed and tachycardic. What is the most
appropriate management?
A. Chest drain
B. Admission, observation and analgesia
C. Analgesia and discharge
D. Infiltration of wound with 5% bupivacaine
E. None of the above

10. A 33 years old man presented with shoulder pain. On passive shoulder abduction, he is
unable to maintain his shoulder abducted. Which of the following is the most accurate
diagnosis?
A. Supraspinatus tendinitis
B. Rotator cuff tear
C. Volkman ischemic contracture
D. Frozen shoulder
E. None of the above

86
11. A 53 years old female patient presented with a winged scapula after breast surgery. What
is the affected muscle?
A. Serratus anterior
B. Latissmus dorsi
C. Petctoralis major
D. Teres minor
E. Teres major

12. A 60-year-old man complains of aching, heaviness numbness and tingling in his thighs and
legs. These symptoms are precipitated by standing or by walking for 10 minutes. They are
relieved by sitting down. He has noticed that walking downhill is more troublesome than
going uphill. What is the most likely cause?
A. Occlusion of an external iliac artery
B. Occlusion of both common iliac arteries
C. Osteoarthritis of the hip
D. Saddle embolus
E. Spinal stenosis

13. A 32 years old man suffered numbness over the inferomedial aspect of the leg just below
the knee. Which nerve is affected?
A. Sural nerve
B. Femoral nerve
C. Infrapatellar nerve
D. Tibial nerve
E. Superficial peroneal nerve

14. A woman sustained needle brick to her thigh. Several days later she noticed red streaks
around site of the brick. What is the most likely cause of this condition?
A. Streptococcus viridians
B. Streptococcus Pyrogens
C. Staphylococcus aureus
D. Streptococcus pneumonia
E. None of the above

15. A 42 years old female presented with an ankle pain. Clenching the calf muscles does not
produce plantar flexion. What is the most likely cause?
A. Ruptured soleus muscle
B. Ruptured gastrocnemius muscle
C. Rupture tendoachile’s
D. Ankle fracture
E. Anterior talofibular ligament injury

87
16. A patient undergone colectomy for a colon cancer. Post operatively he was admitted to
ICU and ventilated. Several days later his chest Xray showed bilateral patches of pneumonia.
What is the causative organism of this case?
A. Streptococcus pneumonia
B. Staphylococcus aureus
C. Pseudomonas.
D. E coli
E. Hemophilus Influenza

17. A slightly overweight 13-year-old boy presents with a three-week history of right hip pain
and limping. On examination the hip moves into external rotation when flexed. Which of the
following is the most likely diagnosis?
A. Developmental dysplasia (congenital dislocation) of hip
B. Juvenile rheumatoid arthritis
C. Perthes disease
D. Slipped upper femoral epiphysis
E. Traumatic dislocation of the hip

18. A 70 years old female after total hip replacement for osteoporotic joint. The removed
head of femur was examined. What would you expect regarding pathological changes in the
specimen?
A. Normal mineralization and reduced bone density
B. Reduced mineralization and reduced bone density
C. Normal mineralization and normal bone density
D. Increased mineralization and increased bone density
E. None of the above

19. A man presented with right leg fracture. Onion peel appearance revealed in x-ray. what is
your diagnosis?
A. Osteosarcoma
B. Osteoblastoma
C. Multiple myeloma
D. Ewing sarcoma
E. Osteoclastoma

20. A 6-year-old baby was brought by his mother after she noticed that he has evident
limping with pain in his left hip. She denies any history of trauma. What is the most accurate
diagnosis?
A. Perthes disease
B. Slipped upper femoral epiphysis
C. Developmental dysplasia of the hip
D. Septic arthritis
E. Osteoarthritis

88
21. A patient developed ptosis and loss of sensation over the medial side of the arm after a
shoulder injury. What is the responsible nerve root?
A. C6
B. C7
C. C8
D. C5
E. T1

22. A 54 years old man was admitted for a cholecystectomy. Investigations revealed that he is
a MRSA nasal carrier. What is the most appropriate treatment?
A. IV teicoplanin
B. Oral vancomycin
C. Mupirocin spray
D. IV vancomycin
E. None of the above

23. A 24 years old man suffered an injury to his upper limb. On examination, his ring and little
fingers are partially flexed with intact sensation over the dorsal aspect of these fingers. What
is the most accepted lesion?
A. Ulnar nerve at the wrist
B. Ulnar nerve at the elbow
C. Ulnar nerve in the forearm
D. Median nerve
E. Radial nerve

24. A 20 years old man with history of a pinned midtarsal fracture 7 years ago is now
presenting with tarsal pain at the site of fracture. Xray would likely show:
A. Osteoarthritis
B. Rheumatoid arthritis
C. Malunion
D. Nonunion
E. Avascular necrosis

25. A patient has tibial shaft fracture, which was managed conservatively. Now he shows
signs of compartment syndrome. The Least reliable sign of compartment syndrome:
A. Loss of sensation
B. Absent distal pulse.
C. Pain on passive movement.
D. Tense swollen leg
E. None of the above

26. What is the dermatomal supply to the elbow, lateral forearm and thumb?
A. C6
B. C8

89
C. T2
D. T1
E. C5

27. During Posterior approach to the distal femur. Which nerve is at risk?
A. Common peroneal nerve
B. Sciatic nerve
C. Tibial nerve
D. Femoral nerve
E. None of the above

28. A 23 years old patient was admitted after falling on his outstretched hand. His
examination revealed that he has lost sensation over the index and middle finger. What is the
most accepted cause of this injury?
A. Hook of hamate fracture
B. Distal radial fracture
C. lunate dislocation.
D. Scaphoid fracture
E. Montagea fracture

29. Treatment of 1cm ureteric stone with no hydronephrosis:


A. ESWL
B. PCN
C. Percutaneous nephrolithotomy
D. Medical treatment
E. Ureteroscopy

30. A 26 years old man presents to the ED with extensive bleeding from his arm after
sustaining a glass injury. On examination there’s a 7 cm transverse laceration across the
anterior aspect of his elbow. On exploring the cubital fossa, you expect the brachial artery to
be:
A. Anterior to the median nerve.
B. Lateral to the biceps tendon
C. Lateral to the median nerve.
D. Medial to the median nerve.
E. Superficial to the bicipital aponeurosis.

31. What is the normal epithelial lining of the urinary bladder?


A. Stratified Squamous epithelium non keratinized
B. Columnar epithelium with goblet cell
C. Columnar epithelium without goblet cell
D. Transitional epithelium
E. Stratified Squamous epithelium keratinized

90
32. A 38 years old man sustained a tibial fracture for which he undergone closed reduction
and casting. Several hours later, he developed severe pain and numbness in his leg. What is
the most appropriate management?
A. Analgesia and follow up
B. Assess pulsation
C. Compartmental pressure management
D. 4 compartment decompression
E. None of the above

33. A surgeon made an incision from below the clavicle to the deltopectoral groove. Which of
the following structures will not be encountered?
A. Medial cord of brachial plexus
B. Axillary vein
C. Thoracoacromial artery
D. Pectoralis minor insertion
E. Pectoralis major

34. A surgeon noticed an artery branching at the superior aspect of pyriformis. What is that
artery?
A. Pudendal artery
B. Superior gluteal artery
C. Inferior
D. Superficial epigastric artery
E. None of the above

35. A 54 years old man undergone hip replacement for fractured neck femur. During follow
up, the man walks with the Trendelenburg gait. What is the most likely cause?
A. Sciatic nerve injury
B. femoral nerve injury
C. Superior gluteal nerve injury
D. Gluteus maximus muscle paralysis
E. Inferior gluteal nerve injury

36. A 49-year-old Egyptian man on holiday in the United Kingdom presents with haematuria.
He gives a history of bladder irritative symptoms for seven months. He has lived in a rural
community for his whole life. What is the causative organism?
A. Fasciola
B. Schistosoma mansoni
C. Schistosoma hematobium
D. Ascaris
E. Hepatitis A

91
37. A 23 years old athlete sustained multiple twisting knee injuries during a game. 24 hours
later, his knee is swollen and painful with restricted movements of the joint. MRI would
show:
A. Meniscal tear
B. Medial collateral ligament injury
C. Lateral collateral ligament injury
D. Anterior cruciate ligament injury
E. Posterior cruciate ligament injury

38. A 23 years old Patient underwent ORIF for a bimalleolar fracture. Below knee cast was
applied. The patient then developed inability to dorsiflex his ankle and paranesthesia along
the dorsum of the foot. Which nerve injury would be the cause of this case?
A. Common peroneal nerve injury
B. Sural nerve injury
C. Superficial peroneal n injury
D. Deep peroneal nerve injury
E. Tibial nerve injury

39. A child has Pruritis ani, stool analysis showed ova and cysts in stool. What is the most
appropriate treatment option?
A. Metronidazole
B. Mebendazole
C. Estradiol
D. Tinidazole
E. None of the above

40. A female patient presented with sharp pain over the dorsum of the foot radiating to the
toes. On examination, there is tender swelling in the cleft between 3rd and 4th metatarsal
bones. What is the most likely cause?
A. March fracture
B. Freiburg disease
C. Morton’s neuroma
D. Osteoarthritis
E. None of the above

41. A 25-year-old female with a displaced fracture to her tibia and fibula about 2 c.m from the
ankle joint, what is the best management to be done?
A. External fixation
B. Cast from groin to foot
C. Intramedullary nailing
D. Fixation with plates
E. None of the above

92
42. A 56-year-old man with chronic emphysema is on the high dependency unit (HDU), ten
days after anterior resection. He has developed acute shortness of breath' and hypotension):
Which of the following landmarks would be the most appropriate to use to obtain a femoral
arterial blood gas sample?
A. A point midway between the anterior superior iliac spine and the pubic tubercle
B. A point midway between the anterior superior iliac spine and the pubic symphysis
C. A point midway between the greater trochanter and the pubic symphysis
D. A point 1 cm inferior and 4 cm lateral to the pubic tubercle
E. A point 2 cm lateral to the midway point between the anterior superior iliac spine and
pubic symphysis

43. A 28 years old man developed loss of sensation below the knee after a posterior hip
dislocation. This is associated with weak movements of the leg. What is the most likely
cause?
A. Tourniquet palsy.
B. Sciatic nerve injury.
C. Compartment syndrome.
D. Femoral nerve injury.
E. None of the above

44. A 70-year-old man presents to his General Practitioner with back pain that has been
worsening over the last six weeks. He also complains of slow urinary flow. Digital rectal
examination reveals an irregular hard prostate. Prostate specific antigen is 300 ng/ml
(normal<4). What should the first line of treatment be:
A. Bladder neck incision
B. Hormonal manipulation
C. Radical prostatectomy
D. Transurethral resection of prostate
E. Watchful waiting

45. A patient developed sensory deficit over the anterior thigh with inability to extend the
knee after surgical hernia repair. What is the nerve injured?
A. Obturator nerve
B. Lateral cutaneous nerve of the thigh
C. Femoral nerve
D. Femoral branch of genitofemoral nerve
E. Sciatic nerve

46. A man developed acute scrotum after trauma with edematous tender left hemiscrotum.
On examination there’s a tender testis which is higher than the other one. The most likely
underlying cause is:
A. Testicular torsion
B. Testicular hematocele
C. Hydrocele

93
D. Teratoma
E. Spermatocele

47. A 50-year-old woman sustains a displaced distal radial fracture. This is manipulated under
anaesthetic and treated in a cast for six weeks. After three months she returns to the fracture
clinic with painful and limited pronation and supination. which of the following is the most
diagnosis?
A. Complex regional pain syndrome Type 1 (reflex sympathetic dystrophy)
B. Malunion
C. Non-union
D. Posterior interosseous nerve palsy
E. Radial nerve compression

48. A 62 years old man developed hematuria and weight loss over the last 2 months.
Abdominal U/S showed a 10 cm mass in the right kidney. What is the most appropriate
investigation?
A. MRI
B. Biopsy
C. CT scan
D. Ascending urethrogram
E. None of the above

49. A 35 years old man noticed a painful swelling above the testis after vasectomy. What is
the most likely underlying diagnosis?
A. Teratoma
B. Hematoma
C. Hydrocele
D. Spermatocele
E. Sperm granuloma

50. Which of the following infections is associated with staghorn stones?


A. Pseudomonas
B. E.coli
C. Acinitobacter
D. Staphylococcus aureus
E. Proteus

51. Most common organism for osteomyelitis is:


A. Streptococcus pneumonia
B. Staphylococcus aureus
C. Pseudomonas
D. Salmonella
E. H. influenza

94
52. A 36-year-old butcher injures his left index finger with a knife, suffering a deep laceration
on the palmar aspect of the middle phalanx. On examination he’s unable to flex DIP, which
tendon is likely to have been injured?
A. Flexor carpi facialis
B. Flexor carpi radialis
C. Flexor digitorum profundus
D. Flexor digitorum superficialis
E. Flexor indicis

53. A 52 years old man suffered an open fracture of tibia. What is the best management?
A. External fixation
B. Cast from groin to foot
C. Intramedullary nailing
D. Fixation with plates
E. None of the above

54. A 53 years old female presented with pain after completing her hand work as a typist. She
reported dropping of objects from her hand. What is the most appropriate treatment?
A. Steroids
B. Physiotherapy
C. Carpal tunnel release
D. Analgesia
E. None of the above

55. A 35-year-old man falls and sustains a fracture to the middle third of his clavicle. Which
vessel is at greatest risk of injury?
A. Subclavian vein
B. Subclavian artery
C. External carotid artery
D. Internal carotid artery
E. Vertebral artery

56. A 63 years old diabetic patient presented with late onset painless urine retention
following a total hip replacement surgery. What is the most likely cause of this condition?
A. Underactive bladder
B. Stone
C. Urethral stenosis
D. Bladder neck inactivity
E. None of the above

57. A 50 years old male with dysuria and tender swollen testicle. He is feverish with rising
TLC. What is the diagnosis?
A. Testicular torsion
B. Testicular hematocele

95
C. Epididymitis
D. Teratoma
E. Spermatocele

58. A woman suffered an arm injury. On examination, she is unable to catch the knife as her
hand flexes. There is loss of sensation over the badge area. What is the likely cause?
A. Radial nerve
B. Axillary nerve
C. Posterior cord injury
D. Medial cord injury
E. None of the above

59. What is the best investigation for supracondylar fracture?


A. CT
B. MRI
C. X ray
D. Bone scan
E. None of the above

60. Nerve supply of dorsal interosseous muscles is:


A. Ulnar nerve
B. Radial nerve
C. Median nerve
D. Anterior interosseus nerve
E. None of the above

61. Young patient with breast lump in lateral upper quadrant, mammogram found a
microcalcifications. What is the most possible diagnosis?
A. Duct ectasia
B. Duct papilloma
C. Fibrocystic disease
D. Duct carcinoma insitu
E. None of the above

62. A 62-year-old woman presents to her General Practitioner with a two-week history of
backpain. She has lost 8 cm in height over the last five years Investigations reveal Findings
Normal Adjusted (corrected) Calcium 2.78 mmol/L (2.15-2.55) mmol/L Phosphate 0.84
mmol/L( 0.8-1.4 )Estimated GFR 0.96ml/minute(normal) More than 90 PTH 8.9 pmol/L 0.95-
5.7 Which of the following is the most likely cause?
A. Hypoparathyroidism
B. Primary hyperparathyroidism
C. Pseudo hypoparathyroidism
D. Secondary hyperparathyroidism
E. Tertiary hyperparathyroidism

96
63. 9-year-old female presented with smooth fluctuant mass showing halo sign in
mammography aspiration was done there is no blood, no fluid refilling what is your
management?
A. discharge no follow-up required
B. surgical excision
C. true cut biopsy
D. FANC
E. None of the above

64. A 55-year-old Male Patient presented with general fatigue and weight loss. With positive
Benz jones protein in urine and passing and lytic bone lesion in Xray what is your diagnosis?
A. Amyloidosis
B. Multiple myeloma
C. Lymphoma
D. Ostomalecia
E. None of the above

65. What is the Reason to give steroid to brain tumor before surgery?
A. Decrease peri tumor edema
B. Decrease tumor vascularity
C. Decrease tumor size
D. Treatment of papilledema
E. Noe of the above

66. A 60-year-old lady presented with creamy nipple discharge. What is your possible
diagnosis?
A. Duct ectasia
B. Breast cancer
C. Ductal papilloma
D. Fibroadenoma
E. None of the above

67. What is the Type of amyloid in Renal failure?


A. AL
B. AA
C. β-amyloid
D. Transthyretin
E. Globulin

68. A 5-year-old boy presents with recurrent episodes of sinusitis. The casualty staff are
surprised to find his liver lying in the left upper quadrant of the abdomen. What is the
underlying diagnosis?
A. Von Recklinghausen's disease

97
B. Li Fraumeni syndrome
C. Pierre Robin syndrome
D. Kartagener's syndrome
E. MEN I

69. 52 years old fell from stairs, his wife believes he lost consciousness for 15 seconds only, he
feels totally recovered now, what to do?
A. Admit and observe
B. Immediate CT
C. CT after 1 Hour
D. CT after 2 hours
E. Discharge and advise on warning sings ??

70. Multiple myeloma can cause which type of amyloidosis?


A. AL
B. AA
C. β-amyloid
D. Transthyretin
E. Globulin

71. Which virus is associated with Kaposi's sarcoma?


A. Human herpes virus 8
B. Human papillomavirus 16
C. Human T-lymphotropic virus 1
D. Epstein-Barr virus
E. Human papillomavirus 18

72. Man presenting with head injury showing deterioration in GCS which becomes 3 with
unilateral unreactive pupil?
A. tentorial Herniating
B. hydrocephalus
C. papilledema
D. Subarachnoid hemorrhage
E. None of the above

73. 60-year-old man brought to E/R complaining of trouble with speech and inability to form
long sentences what is the artery responsible for this condition?
A. ACA
B. MCA
C. PICA
D. PCA
E. NONE of the above

98
74. A 79-year-old woman with hypochromic microcytic anemia and axillary and inguinal
lymphadenopathy what is your diagnosis?
A. Bronchial cancer
B. Non-Hodgkin lymphoma
C. Hodgkin lymphoma
D. Gastric cancer
E. None of the above

75. A 50 year old Patient came to ER assessment of consciousness level was done according to
GCS patient can localizes pain, open eye to pain and sounds what the GCS?
A. 9
B. 7
C. 12
D. 11
E. None of the above

76. A 35-yr-old lady presented with bloody nipple discharge. What is your possible diagnosis?
A. Duct ectasia
B. Breast cancer
C. Ductal papilloma
D. Fibroadenoma
E. None of the above

77. Pt. had experience of abdominal pain and diarrhea and vomiting his investigations
revealed hyperparathyroidism what is the next investigation should be done
A. Thyroid
B. Pituitary
C. Adrenals
D. Kidney
E. None of the above

78. A 21-year-oid man has been hit by a cricket ball on the side of the head. Initially he has a
Glasgow coma score (GCS) of 1EJE4, V5, M6). Four hours after arrival in hospital, while on the
observation ward, his GCS drops to 7(E1, V2, M4). His left pupil dilates, and he begins to
demonstrate extensor posturing of the limbs. What are the most likely changes to his vital
signs since injury?
Blood pressure Heart rate
A Decreased Decreased
B Decreased Increased
C Decreased Normal
D Increased Decreased
E Increased Increased

99
79. A 66-year-old female presented with Thyroid mass rapidly growing and has stridor with
change in her voice what is your possible diagnosis?
A. follicular Carcinoma
B. Papillary carcinoma
C. Anaplastic carcinoma
D. Medullary carcinoma
E. None of the above

80. A 55 year old female presented with Thyroid mass and she also complaining from
pulsating mass in her forehead what is your possible diagnosis?
A. follicular Carcinoma
B. Papillary carcinoma
C. Anaplastic carcinoma
D. Medullary carcinoma
E. None of the above

81. A 24-year-old woman presents with a swelling in the thyroid gland. She also has enlarged
lymph nodes in the left anterior triangle of the neck. Which of the following is the most likely
diagnosis?
A. Anaplastic carcinoma
B. Follicular adenoma
C. Follicular carcinoma
D. Medullary carcinoma
E. Papillary carcinoma

82. An 8-year-old boy presents with a lump in his neck. On examination, he is found to have a
lump in the midline of the neck immediately below the hyoid which moves on tongue
protrusion. What is the most likely diagnosis?
A. Benign lymphadenopathy
B. Malignant lymphadenopathy
C. Thyroglossal cyst
D. Branchial cyst
E. Dermoid cyst

83. A 19- year-old man fell down a flight of stairs. On presentation to the Emergency
Department his Glasgow Coma Score (GCS) was 14/15 with no local neurological signs. An
hour later he vomited, and his GCS deteriorated to 10. He developed a fixed and dilated left
pupil. Which of the following is likely to seen on the scan?
A. Left-sided acute subdural hematoma
B. Left-sided extradural hematoma
C. Left-sided midbrain hematoma
D. Right-sided acute subdural hematoma
E. Right-sided extradural hematoma

100
84. A 25-year-old lady presents with a swelling located at the anterior border of the
sternocleidomastoid muscle. The swelling is intermittent and on examination it is soft and
fluctuant. What is the most likely diagnosis?
A. Thyroglossal cyst
B. Branchial cyst
C. Carotid body tumor
D. Infection with Bartonella
E. Dental abscess

85. Esophageal biopsy shows the columnar epithelium containing goblet cells what is the type
of changes occur in cell?
A. aplasia
B. atrophy
C. hyperplasia
D. hypertrophy
E. metaplasia

86. A 20-year-old lady presents with a lump in the upper outer aspect of her right breast. On
examination, she has a firm mobile mass in the upper outer quadrant of her right breast.
What is the most likely underlying disease process?
A. Atypical ductal hyperplasia
B. Breast cyst
C. Duct ectasia
D. Fibroadenoma
E. Fat necrosis

87. Pt with cervical lymphadenopathy underwent excisional biopsy showed non necrotizing
granuloma with giant cell and fibrosis what is your possible diagnosis?
A. Sarcoidosis
B. TB
C. Lymphoma
D. Leprosy
E. Fungal infection

88. A 49-year-old female is due to undergo a renal transplant. Apart from ABO compatibility
which of the following is most important in matching donor and recipient organs?
A. HLA-DR
B. Rhesus
C. HLA- A
D. HLA-B
E. HLA-C

101
89. 60-year-old man presents with a short history of pain in the right cheek and right upper
teeth Maxillary sinus infection is diagnosed. This sinus is particularly prone to infection
because of?
A. absence of Celia on the epithelium lining the sinus
B. closeness of the sinus to the nasal cavity
C. poor blood supply
D. Position of the sinus ostium high on the medial wall
E. relationship of the front teeth to the floor of the sinus

90. A 33-year-old female presented with depression and her investigations reveled, low TSH
and increased Ca and she was on antacid what is your possible diagnosis of this condition?
A. milk alkali syndrome
B. vit d toxicity
C. hyperthyroidism
D. Hyperparathyroidism
E. Sarcoidosis

91. Pt with liver biopsy, showing birefringence apple green with Congo red stain, what is your
diagnosis of this condition?
A. Abnormal deposition of copper
B. Amyloidosis
C. Hemochromatosis
D. Hemosiderin deposition
E. None of the above

92. What is the defective gene in a patient with breast cancer ?


A. P53
B. BRCA I
C. APC
D. SIS
E. None one the above

93. Discomfort. He has never attended the hospital previously and is usually well. He has just
retired from full time employment as a machinist in a PVC factory. CT scanning shows a large
irregular tumor in the right lobe of his liver. Which of the following lesions is the most likely?
A. Liposarcoma
B. Angiosarcoma
C. Hamartoma
D. Hydatid liver disease
E. Benign angioma

94. A 4-year-old is brought to the general practitioner by her mother. She has been distressed
with ear pain for the past 14 hours. She is constantly touching and pulling at her ear. Whilst

102
she is sat in the waiting room her mother notices a discharge of foul-smelling fluid from the
ear, following which the pain resolves
A. Acoustic neuroma
B. Preauricular sinus
C. Acute suppurative otitis media
D. Cholesteatoma
E. Otitis externa

95. A previously fit young man is admitted after a road traffic accident. He is satisfactorily
intubated because of a head injury. He has a Glasgow coma score of 8 (E2, V4, M4) There are
no other external injuries and initial cardiorespiratory stability is achieved during the primary
survey. He suddenly becomes hypoxic after log- rolling to examine the back. What is the most
likely cause of his deterioration?
A. Cardiac tamponade
B. Ongoing major hemorrhage
C. Spinal injury
D. Tension pneumothorax
E. Tracheal tube displacement

96. 32 years old female presented with thyroid nodule and hyperthyroidism followed by
hypothyroidism in her thyroid function what is the most distinctive investigation to reach
diagnosis?
A. FNAC
B. Antibodies
C. MRI
D. Repeat thyroid function
E. None of the above

97. A 35-year-old women complained from sustained tibial fracture, her underwent IMN, few
hours later he started to have calf pain, you suspected the patient might be having
compartment syndrome, which of the following is the first management?
A. Analgesia with morphine
B. Four compartment decompression
C. Serial examination and follow up
D. Intercompartmental pressure measurement
E. 5-Nerve block

98. A 35-year-old male presented with severe chest pain, he is tall and slim with history of
inguinal hernia repair when he was 2 years old. His father died sudden at age of 47 years old
with sudden chest pain. What is the most accepted diagnosis?
A. Anginal pain
B. Pleural effusion
C. Aortic dissection
D. Lung abscess

103
E. Esophageal rupture

99. You have collected a data of 87 child’s urea and create levels before anesthesia to study
the effect of anesthesia on dehydration. You matched them with the normal levels. What is
the statistical test to use?
A. Paired T test
B. Un paired T test
C. Mann Whitney
D. Chi-square

100.A 25-year-old female patient presented to the ER with sudden onset Abdominal and Bp
90/60 with 110 pulse rates. By examination she has tenderness all over the abdomen
especially right iliac fossa. She confirms that she has no history of abdominal pain or similar
condition. What is the most accepted diagnosis?
A. Acute appendicitis
B. Rupture ectopic pregnancy
C. Salpingitis
D. Perforated duodenal ulcer.
E. PID

101. A 37-Female presented with ulcer on the medial malleolus with painless skin changes
around ankle. With a history of varicose vein repair. Her ABI is normal. What is the most
accepted cause of this ulcer?
A. Infected eczema
B. Venous ulcer
C. Ischemic ulcer
D. Malignant ulcer
E. Neuropathic ulcer

102. What is the scientific bases of application of tourniquet that used in operations to
control blood loss and give bloodless field?
A. after exsanguination Inflation of cuff above diastolic
B. Inflation of cuff above mean before exsanguination.
C. Inflation of cuff above mean before exsanguination.
D. Inflation of cuff above systolic before exsanguination
E. Inflation of cuff above systolic after exsanguination

103. A 20-year-old man presents with a 24-hour history of sever anal pain and fever, with a
history of one month of perianal discharge and diagnosed a perianal fistula. On examination
he has a peri anal abscess. What is the best treatment?
A. Fistulotomy
B. Incision and drainage of the abscess
C. Seton wire application
D. Multistage fistula repair

104
E. Lateral sphincterotomy

104. You notice a nurse was practicing a minor procedure in the ward which lacks evidence or
any reference what will you do?
A. Risk assessment
B. Medical Audit
C. Report to GMC
D. Health and safety
E. Incident report

105. A 26-week gestation suspected to have a perforated viscus what is the most accurate
investigation to diagnose the case?
A. Chest x-rays erect.
B. Abdominal US
C. CT abdomen
D. MRI abdomen
E. Cross table lateral Abdominal X-ray

106. What is the best long-term management for post-phlebitis syndrome?


A. Heparin
B. Long life warfarin
C. Lower limb stocking bandage
F. Vitamin C
D. Antibiotics

107. You are conducting a cross sectional study measuring BP in certain population, what is
the type of the data that you will obtain?
A. Ordinal
B. Nominal
C. Continuous
D. Discrete
E. E. None of the above

108. A 35 female patient underwent lap cholecystectomy and discharged to home in a good
general condition, one month later she return unremitting jaundice, pale stool and dark
urine. What is the most accepted diagnosis?
A. Cystic duct leak
B. CBD injury
C. Missed CBD stone
D. Cancer pancreas
E. HCC

109. A 43-year-old lady underwent lap cholecystectomy, and presented post-operative with
colicky abdominal pain and jaundice so she procced for ERCP after this maneuver she is

105
feverish and jaundice with severe abdominal pain in the right upper quadrant, what is the
most likely diagnosis?
A. Pancreatitis
B. Cholangitis
C. Missed stone.
D. Perforated Duodenal ulcer
E. Liver abscess

110. A 50-year-old male presented with a dry gangrene of fifth toe, investigation done and
revealed a long segment occlusion in the superficial femoral artery with only one patent
runoff in the posterior tibial artery. What is the most appropriate management for this
situation?
A. Amputation of 5th toe
B. Below knee amputation
C. Midfoot amputation
D. Femoro-distal bypass
E. Angiography with insertion of stent

111. A 55-year-old man presents with a history of progressive dysphagia over the past 4
weeks. For the past 5 years he had regularly attended his general practitioner with symptoms
of dyspepsia and reflux. What is the most accepted cause for dysphagia?
A. Benign esophageal stricture
B. Carcinoma of the esophagus
C. Achalasia
D. Hiatus hernia
E. Esophageal reflux

112. A known case of Ca Head of pancreases presented by jaundice painless unremitting


jaundice, PET scan of the body showed a liver Mets. what is the best intervention?
A. Cholecystectomy
B. ERCP and Stent
C. Hepaticojejunostomy
D. Whipple’s operation
E. Gastrojejunostomy

113. A case of intermittent claudication in the calf muscles with loss of pulse below knee only,
where is the most probable site of the arterial lesion?
A. SFA occlusion at the adductor canal
B. CFA before origin of profunda artery
C. External iliac artery
D. Anterior tibial artery
E. Internal iliac artery

106
114. An 8 months screaming child was brought by his mother who described that he passed a
red jelly stool this morning. What is the most likely underlying diagnosis?
A. Intussusception
B. Meckel diverticulum
C. Necrotizing enterocolitis
D. Juvenile polyp
E. anal fissure

115. A man done hyperhidrosis operation. On examination, he has a ptosis and miosis. Which
nerve roots you suggest involvement of?
A. C2 & C3 roots
B. C4 & C5 roots
C. C6 & C7 roots
D. C8 & T1 roots
E. T2 & T3 roots

116. Renin acts on which one of the following


A. angiotensin 1
B. angiotensin 2
C. Angiotensinogen
D. ADH
E. Aldosterone

117. A 65 year old man is admitted with a two week history of vomiting investigations
confirm a benign pyloric stenosis. What of this finding match with this patient?
K+; Na+; PH: Normal 3.5-4.9 mmol/L; 137-144 mmol/L; 7.3 : 7.4
A 2.6; 148; 7.1
B 3.4; 156; 7.48
C 3.2; 118; 7.47
D 6.0; 120; 7.1
E 2.8; 128; 7.2

118. 45 years old patient with severe head injury 14 days ago. He is now has slow recovery,
what type of feeding this patient will you need?
A. TPN
B. Nasogastric tube
C. Feeding jejunostomy
D. Enteral with special formula
E. Oral intake

119. Which of the following serves as the axis of gut rotation during development?
A. aorta
B. Superior mesenteric artery
C. Inferior mesenteric artery

107
D. Umbilical vessels
E. Vitelline artery

120. Inferior epigastric vein drains in?


A. Internal iliac vein
B. External iliac vein
C. Superior mesenteric vein
D. Inferior mesenteric vein
E. Femoral vein

121. A patient will do hernial repair and his Hemoglobin is 9 he put in a list after 6 weeks how
would you give him supplement
A. blood transfusion
B. plasma transfusion
C. IV iron
D. oral iron
E. folic acid

122. 50-year-old patient for elective hernia repair as the day case, he is diabetic type 1 and
has autonomic neuropathy, what is the main problem as a day case in this patient?
A. Reduced pulse rate variability
B. Increase urine output
C. Bleeding
D. Ischemia of the heart
E. None of the above

123. Abdominal free fluid will collect in the lowest part of the peritoneal cavity. At operation
with the patient supine, in which of the following will the fluid collect first?
A. Hepatorenal pouch
B. Left anterior subphrenic space
C. Lesser sac
D. Right paracolic gutter
E. Right anterior subphrenic space

124. Which of following investigations is most accurate for measuring the renal plasma flow?
A. Creatinine clearance
B. Glucose Clarence
C. Inulin Clarence
D. Para-amino-hippuric acid
E. Urea clearance

125. A 12-year-old child was admitted to the Emergency Department having inhaled a
peanut. Where will the peanut most likely to have lodged?
A. Left lower lobe bronchus

108
B. left upper lobe bronchus
C. Right lower lobe bronchus
D. Right middle lobe bronchus
E. Right upper lobe bronchus

126. Patient presented by tachycardia and hypertension which of the following hormone
decrease with warming
A. TSH
B. ACTH
C. ADH
D. Cortisol
E. Catecholamine

127. Clinical scenario about tooth extraction and the patient develop lower lip numbness,
what the nerve affected?
A. Lingual nerve
B. Inferior alveolar
C. Marginal mandibular branch of facial nerve
D. Facial nerve
E. Hypoglossal nerve

128. A 32-years old man presents with a painful torticollis there is no past medical history and
his only complaint is that he has been feeling rather tired over the last 4 nights. On
examination, he has large rubbery mass in the lateral aspect of his neck as well as a few
smaller masses along his internal jugular vein. You correctly assume that the muscular neck
spasm and the large mass are connected. The most likely cause of his torticollis is due to
pressure on?
A. Ansa cervicalis
B. Cervical plexus branch
C. Cranial accessory nerve
D. Spinal accessory nerve
E. Vagus nerve

129. Patient with serum potassium 6.5. What should be immediate step?
A. I.V. fluid
B. Insulin and glucose infusion
C. B agonist
D. Pamidronate
E. i.v. ca gluconate

130. A female presented by 20% burn over her chest, for which she underwent graft
treatment, 10 days later she came with sweeping discharge from the graft edges, fever and
chest pain, what medication you will give?
A. Flucloxacillin

109
B. Cephalosporin
C. Cephalosporin and Metronidazole
D. Ampicillin
E. None of the above

131. Patient injured his tip of index finger, what is the best type of anesthesia to be used in
ring block?
A. Lidocaine
B. Lidocaine with adrenaline
C. Bupivacaine
D. Bupivacaine with adrenaline
E. Prilocaine

132. Which cell is dependent on Cori cycle for energy:


A. RBC
B. Muscle cell
C. Hepatocyte
D. Fat cells
E. Nerve cells

133. Hypercalcemia is firstly treated by


A. Iv saline chloride .9%
B. IV ca chelating
C. K supplementation
D. Pamidronate
E. Zoledronate

134. Which of these I.V fluid have the highest sodium content
A. Saline glucose fluid
B. Normal saline
C. Hartman solution
D. Glucose 5%
E. None of the above

135. Calculate Cerebral perfusion pressure if Blood pressure =110/80 and ICP =18.
A. 72
B. 82
C. 90
D. 102
E. 115

136. An 82-year-old man has complete occlusion of his inferior mesenteric artery on
angiography but no symptoms or signs of colonic ischemia. Which of the following arteries is

110
the most likely additional source of blood supply to the territory of the inferior mesenteric
artery?
A. Left colic
B. Left gastroepiploic
C. Middle colic
D. Splenic
E. Superior rectal

137. All are content of the posterior mediastinum except:


A. Descending aorta
B. Vagus nerve
C. Thoracic duct
D. Azygous vein
E. Vertebral body

138. In fetal circulation, how will the blood passes from rt atrium to left atrium?
A. Through ductus venosus
B. Through ductus arteriosus
C. Through foramen ovale
D. Through pulmonary VSD
E. None of the above

139. A Child with kwashiorkor developed hepatomegaly. What is the deposit in liver?
A. Sarcoid
B. Melanin
C. Lipofuscin
D. Amyloid
E. fat

140. A child with patent ductus arteriosus and left sternum murmur heard on his chest, what
about this anomaly?
A. Shunt from right atrium
B. Shunt to pulmonary artery
C. Shunt to aorta
D. Shunt to left atrium
E. None of the above

141. After clamping the umbilical cord what will not give oxygenated blood to the embryo
any more?
A. Ductus venosus
B. Umbilical artery
C. Ductus arteriosus
D. Pulmonary artery
E. Pulmonary vein

111
142. An elderly patient with an Ulcer painless on his face and his biopsy show flat cells with
keratin, what is your diagnosis?
A. SCC
B. Keratoacanthoma
C. Herpes labialis
D. BCC
E. Metastatic

143. 4 Month child with failure to thrive with recurrent chest infection and dyspnea increase
with lying flat in the night and awake him from sleep
A. TOF
B. Congenital diaphragmatic hernia
C. Asthma
D. GERD
E. Intestinal atresia

144. Where is the major duodenal papillae open?


A. In the anteromedial part of Third part of duodenum
B. In the posteromedial part of Third part of duodenum
C. In the posteromedial part of 2nd part of duodenum
D. In the anteromedial part of 2nd part of duodenum
E. In the posteromedial part of First part of duodenum

145. What vitamin supports collagen or important in wound healing?


A. vitamin A
B. vitamin E
C. vitamin B6
D. vitamin C
E. vitamin K

146. Peripheral neuropathy in alcoholic patient is due to deficiency of which vitamin


A. Thiamine
B. Pellagra
C. folic acid
D. iron
E. vitamin B12

147. Vitamin K is required for normal blood clotting. Which one of the following statements is
true about the effects and availability of vitamin K?
A. Affects platelet function
B. Cause bleeding if taken in excess
C. Is provided only by fresh food
D. Is depleted by oral broad-spectrum antibiotics

112
E. Vitamin K absorption is affected by gastric resection

148. A 34 years old patient was in operation with tourniquet. after release of tourniquet,
what is the substance released causing VD?
A. Noradrenaline
B. Vasopressin
C. Histamine
D. Serotonin
E. Adrenaline

149. A 7 years old boy presents with abdominal pain with attack of bloody stool and
diagnosed as Meckel diverticulum, which of the following ectopic mucosa mostly found in
Meckel
A. pancreatic
B. ileal
C. gastric
D. duodenal
E. none of the above

150. The female pelvis is characterized by which of the following:


A. the female pelvis is deeper and narrow
B. the female inlet is apple in shape
C. pubic arch is wide
D. the bone is heavy
E. coccyx is pointed inward

151. The patient has microcytic hypochromic anemia due to iron deficiency and taken for
endoscopy, what is the site of biopsy will you take?
A. Esophagus
B. Gastric
C. Duodenum
D. Jejunum
E. Colon

152. 1-day neonate with history of polyhydramnios intrauterine and after birth, fluid coming
from mouth with dyspnea and cyanosis
A. congenital esophageal atresia with tracheoesophageal fistula.
B. congenital diaphragmatic hernia
C. duodenal atresia
D. Hirschsprung
E. None of the above

153. A 28-year-old man presents with an ischiorectal abscess. Where is this abscess cavity
likely to be sited?

113
A. Above the levator ani
B. Between external and internal anal sphincter
C. Lateral to the obturator internus
D. Medial to the internal anal sphincter
E. Medial to the pudendal canal

154. Post colectomy heel pressure sore with skin necrosis, what is management?
A. debridement and delayed primary closure
B. compression
C. Elevation
D. STSG
E. Conservative dressing

155. What is true about SA node anatomy and physiology


A. Main conducting system of the heart
B. present in the wall of left atrium
C. has somatic nerve supply
D. has vagal nerve supply
E. none of the above

156. Patient with TIA, he underwent left carotid endarterectomy and immediately post
procedure he developed dysarthria and tongue deviation to the left.
What is the most likely diagnosis?
A. Another TIA
B. Hypoglossal nerve neuropraxia
C. Middle cerebral artery occlusion
D. Anterior cerebral artery occlusion
E. None of the above

157. Foramen of skull in which glossopharyngeal nerve pass


A. Foramen ovale
B. Foramen rotundum
C. Jugular foramen
D. Foramen lacerum
E. Hypoglossal canal

158. Man had a brain tumor, supplied by a very large middle meningeal artery, which
foramen was enlarged or dilated?
A. Foramen of ovale
B. Foramen rotundum
C. Foramen spinosum
D. Foramen lacerum
E. Jugular foramen

114
159. 2 years old girl with burn over whole dorsum of forearm with blister and erythema, what
is the appropriate management?
A. Occlusive dressing
B. Split thickness graft
C. escharotomy
D. Leave uncovered
E. fasciotomy

160. 18 years girl with cystic swelling fluctuating at anterior border of sternomastoid border
A. Thyroglossal cyst
B. cystic hygroma
C. Branchial cyst
D. congenital epidermoid
E. acquired epidermoid

161. In cases of Unilateral cleft lip there is failure of fusion of?


A. Maxillary process and medial nasal process
B. Maxillary process and lateral nasal process
C. Palatine processes
D. Maxillary processes
E. None of the above

162. A 18 years old man after he ate canned food, he feels flaccid paralysis and diagnosed as
botulinum toxin , in which does this toxin act on?
A. Acetylcholine
B. Adrenaline
C. GABA
D. Glutamate
E. Serotonin

163. A surgeon is carrying out an elective splenectomy for congenital spherocytosis. Which
structure will be divided in order to mobilize the spleen from the posterior abdominal wall?
A. Gastrosplenic ligament
B. Lesser omentum
C. Lienorenal ligament
D. Phrenicocolic ligament
E. Short gastric vessels

164. A patient came with keloid scar after old trauma, what is Keloid scar best management
A. intralesional steroids
B. re-excision
C. laser therapy
D. oral corticosteroids
E. none of the above

115
165. A woman with increased TSH+ lower limit T4 , what is your diagnosis?
A. primary hypothyroidism
B. primary hyperthyroidism
C. Secondary hyperthyroidism
D. Secondary hypothyroidism
E. sick euthyroid

166. Overweight person and doing hip surgery . Which of the following is not considered a
measure to enhance rapid recovery?
A. oral carbohydrate 2 hours preoperatively
B. weight reduction before surgery
C. anti-emetic
D. diazepam
E. Epidural analgesia

167. Porcine skin graft


A. xenograft
B. Isograft
C. Autograft
D. Allograft
E. None of the above

168. A 28-year-old primigravida presents in the Emergency Department with history of


sudden chest pain. Her father died at 42 years of age of a heart attack. She is strikingly tall
and slim. On examination there are no abnormal physical signs other than a tachycardia of 90
beats minute in keeping with her anxiety.
A. Acute massive pulmonary embolism
B. Aortic dissection
C. Myocardial infarction
D. Puerperal sepsis
E. Pulmonary infarction

169. Bullet going through junction of linea semilunaris and costal margin on right side, which
structure will be injured?
A. GB
B. Liver
C. Right kidney
D. Cecum
E. Spleen

170. 72-year-old man is due to undergo an esophagectomy for malignancy. His BMI is 17.5.
What is the best feeding regime immediately following surgery?
A. Total parenteral nutrition.
B. Feeding jejunostomy.

116
C. Feeding duodenostomy.
D. Liquid diet orally.
E. Soft solids orally.

171. Newborn baby girl passed meconium via vagina and after examination show lower
vaginal what is embryological origin?
A. Vittelointestinal tract
B. Urachus
C. Improper division of Cloacal septum
D. Mesonephric duct
E. Metanephric duct

172. In a patient with Cleft palate, it represents defect in the embryological fusion of what
structures?
A. Frontonasal
B. Maxillary
C. palatal processes
D. lateral nasal
E. medial nasal

173. 50 years old woman presented by 3 cm solitary swelling in her right lobe of the thyroid
gland
A. Excisional biopsy with 2 mm safety margin
B. Excisional biopsy with 2 cm safety margin
C. Incisional biopsy
D. FNAB
E. True cut biopsy

174. A 68 years old man presented with an inguinal hernia. During repair it was found to be
direct hernia. Which of the following statements is correct regarding direct hernia?
A. inferior epigastric artery forms the lateral border
B. inferior epigastric artery forms the medial border
C. iliac artery forms the lateral border
D. iliac artery forms the medial border
E. none of the above

175. What is structure open during herniorrhaphy after skin and fascia
A. external oblique Aponeurosis
B. internal oblique muscle
C. conjoint tendon
D. fascia transversalis
E. peritoneum

117
176. A surgeon makes a Pfannenstiel incision for access to the pelvic organs. He incises the
abdominal wall down to and through the rectus sheath. He retracts the rectus abdominis
muscles laterally from the midline to expose the:
A. linea alba
B. peritoneum
C. posterior rectus sheath
D. transversalis fascia
E. transversus abdominis muscle

177. 65 years old man presented with an inguinoscrotal swelling in the right groin which is
non tender. A cough impulse is elicited, at operation, an indirect inguinal hernia is repaired.
The cremasteric muscle is derived from which of the following
A. External oblique aponeurosis
B. Internal oblique muscle
C. Rectus abdominal muscle
D. Rectus sheath
E. Transversalis fascia

178. A 19-year-old man was assaulted and sustained injuries to the right side of his head.
After two weeks he notices dial his right eye is dry and it could not produce tears. from which
ganglion is post synaptic fibers arise to supply lacrimal gland?
A. Geniculate ganglion
B. Inferior ganglion of the vagus nerve
C. Otic ganglion
D. Pterygopalatine ganglion
E. Superior cervical ganglion

179. What is the nerve supply for the 2nd brachial arch?
A. Mandibular of trigeminal
B. Facial
C. Glossopharyngeal
D. Superior laryngeal
E. Vagus

180. Pharyngeal diverticulum in pharynx occur between?


A. Between sup and middle constrictors
B. Between middle and inf constrictor
C. Between thyropharyngeal and cricopharyngeal muscle
D. Between false and true vocal cords
E. None of the above

181. Posterior relation of kidney includes which of the following?


A. Adrenal
B. Psoas muscle

118
C. Liver
D. Hepatic flexure
E. Pylorus

182. 1cm BCC on the nose of woman removed up to mucosa level and ala cartilage
A. full thickness graft
B. split thickness graft
C. local flap
D. heal by secondary intention
E. none of the above

183. Post thyroidectomy patient with low voice, what's nerve injury?
A. Superior laryngeal unilateral
B. Recurrent laryngeal unilateral
C. Glossopharyngeal nerve
D. Accessory
E. Phrenic nerve

184. A man with prostatism, On PR, posterolateral ridge on palpation. What structure?
A. Sacrospinous ligament
B. Puborectalis muscle
C. Ischiorectal fossa
D. Obturator internus muscle
E. None of the above

185. What is the structure that felt in PV examination anteriorly at level of cervix?
A. Dome of bladder
B. base of bladder
C. puborectalis
D. ischiorectal fossa
E. none of the above

186. A 50-year-old woman presents with a history of fecal incontinence over the past few
year she had a prolonged and difficult first stage of labor 20 years previously. Physical
examination reveals a relatively lax anal sphincter. Which nerve is likely to have been
damaged in labor?
A. Autonomic nerves to the rectum
B. Genitofemoral nerve
C. Lumbosacral trunk
D. Obturator nerve
E. Pudendal nerve

119
187. A 42-year-old multiparous woman is admitted to the Emergency Department due to
pelvic discomfort. The duty gynecologist diagnoses uterine prolapse. Which anatomical
structure gives significant direct support to the uterus?
A. Cervical ligaments
B. broad ligament
C. Round ligaments
D. Transverse perineal Muscles
E. None of the above

188. Closure of the tricuspid valve occurs at the onset of which phase of the cardiac cycle?
A. Isovolumic relaxation
B. Rapid filling
C. Reduced filling
D. Rapid and reduced ejection
E. Isovolumic contraction

189. Type of epithelium of the urinary bladder


A. Transitional epithelium
B. Non stratified Squamous epithelium
C. Columnar epithelium
D. Cuboidal epithelium
E. Stratified squamous epithelium

190. 31 years old presented by ureteric colic referred to inguinal region. Why this pain
referred to inguinal region according to dermatomal supply?
A. T8 – T 10
B. T10
C. T11 - L1
D. L2 – L 5
E. S2 – S5

191. Changes that happen in septic shock


COP SVR HR
A. High Low High
B. Low Low Low
C. Low High Low
D. Low High High
E. Normal Normal Normal

192.34 years old pregnant woman develop swollen leg. her mother and her maternal aunt
have the same problem during pregnancies. Which of the following test will be positive in the
blood?
A. Antinuclear antibodies
B. Antiphospholipid antibodies

120
C. Anti-thyroglobulin antibodies
D. Antimitochondrial antibodies
E. None of the above

193. In Hemophilia, which factor is likely to be affected?


A. V
B. VIII
C. X
D. VII
E. IX

194. The pleural space is negative with respect to atmospheric pressure, except for which of
the following:
A. During valsalva maneuver
B. During Muller maneuver
C. During expiration
D. During inspiration
E. None of the above

195. During inguinal hernial repair, we found transverse nerve when we open the external
oblique muscle during repair
A. Genital of Genitofemoral nerve
B. Femoral of genitofemoral nerve
C. Ilioinguinal nerve
D. Femoral nerve
E. Obturator nerve

196. What is the metabolic effect of insulin?


A. increase glycogen synthesis in muscle
B. increase potassium in blood
C. increase glycolysis in liver
D. increase Na in the blood
E. none of the above

197. Level of cones medullaris termination in 1 month Child?


A. L1- L2
B. L2-L3
C. L4-L5
D. S1- S2
E. S2 – S3

198. A woman is in the recovery room receives 15L oxygen by mask Blood Gas Shows PH (7.3)
, pO2 (11) , pCo2 (7): which receptor is most likely to be stimulated by this blood gas picture
A. Adrenergic receptors

121
B. Baroreceptors
C. Chemoreceptors
D. J receptors
E. Lung stretch-receptors

199. Location of middle cardiac vein in which site?


A. Anterior interventricular groove
B. Posterior interventricular groove
C. Right atrioventricular groove
D. Left atrioventricular groove
E. At margin of right ventricle

200. A patient undergone splenectomy after trauma. During operation the surgeon was due
to ligate the splenic artery. What is the structure most likely to be injured during this step?
A. Tail of pancreas
B. Fundus of stomach
C. Pancreatic duct
D. Phrenico-colic ligament
E. Small intestine

201. A 72-year-old man has just undergone an emergency repair for a ruptured abdominal
aortic aneurysm. Preoperatively he was taking aspirin, clopidogrel and warfarin. Intra
operatively he received 5000 units of unfractionated heparin prior to application of the aortic
cross clamp. His blood results on admission to the critical care unit are as follows: ull blood
count: Hb 8 g/dl, Platelets 40 * 109/l and WBC 7.1 * 109/l His fibrin degradation products are
measured and found to be markedly elevated. Which of the following accounts for these
results?
A. Anastomotic leak
B. Disseminated intravascular coagulation
C. Heparin induced thrombocytopenia
D. Adverse effect of warfarin
E. Dearranged platelet function as the drug not stopped

202. A 56-year-oId man presents with headache and vomiting with deterioration over the last
two weeks. On examination he has papilledema. A CT scan shows a space occupying lesion in
the right parietal lobe. He develops a sudden homonymous hemianopia. Which past of the
visual system to be affected?
A. Lens
B. Optic Chiasm
C. Optic radiation
D. Retina
E. Second cranial nerve

122
203. A 2-year-old child is referred because of an impalpable testis in the left scrotum. Which
of the following positions describes an ectopic testis?
A. At the deep inguinal ring
B. At the base of the penis
C. At the superficial inguinal ring
D. In the inguinal canal.
E. in the upper part of the scrotum

204. What is not reliable to measure oxygen on low tissue perfusion


A. Pulse oximeter
B. ABG
C. Capillary wedge pressure
D. Swan Ganz catheter
E. None of the above

205. What is the mechanism of action of Local lidocaine?


A. sodium channel block
B. sodium channel opening
C. k channel block
D. k channel open
E. none of the above

206. 28-year-old primigravida presents in the Emergency Department with history of sharp
pleuritic chest pain. Her father died at 67 years of age of stomach cancer. She coughed up
some blood at morning.
A. Acute massive pulmonary embolism
B. Aortic dissection
C. Myocardial infarction
D. Panic attack
E. Pulmonary infarction

207. 26-year-old primigravida presents in the Emergency Department extremely breathless


complaining of tightness in her chest. She is Cyanosed and the neck veins are distended Her
62-year-old father has recently died of myocardial infarction.
A. Acute massive pulmonary embolism
B. Aortic dissection
C. Myocardial infarction
D. Panic attack
E. Pulmonary infarction

208. Which single respiratory test parameter (FEVJ Forced expiratory volume in one second;
FVC -Forced vital capacity) is the best prediction for morbidity and mortality after surgery?
A. A. Confronted flow volume loop
B. FEV1 < 1L

123
C. FEV1/FVC > 90%.
D. tidal volume = 1500
E. increased residual volume

209. Target INR to patient on warfarin with superficial femoral superficial femoral thrombosis
A. 1 – 2
B. 2 – 3
C. 3 – 4
D. 4 – 5
E. 5 – 6

210. Man in RTA, complained of LL pain and lower abdominal pain, blood pressure 85/60,
heart rate 110 and conscious and talking GCS not mentioned, what is the initial management?
A. massive hemorrhagic protocol
B. morphine for pain
C. tracheal intubation
D. platelet transfusion
E. O blood transfusion

211. Bluish colored fluctuant swelling below the tongue due to Sublingual gland
A. Branchial cyst
B. Thyroglossal cyst
C. Epidermoid cyst
D. Ranula
E. Submandibular abscess

212. A 68-year-old farmer presents with a skin lesion on his forehead. It has been present for
the past 6 months and has grown slightly in size during that time. On examination he has an
ulcerated lesion with pearly white raised edges that measures 2cm in diameter.
A. Squamous cell carcinoma
B. Bowens disease
C. Actinic keratosis
D. Keratoacanthoma
E. Basal cell carcinoma

213. 54 years old presented by nocturia and thirst. The physician asked him to do FBF and
PPG – FBG was 5 then post prandial 2 hours was 7, what is your diagnosis?
A. Normoglycemia
B. impaired glucose tolerance test
C. DM
D. DI
E. None of the above

214. Recurrent pulmonary embolism despite of heparin use

124
A. LMWH
B. LMWH and stocking
C. vena cava filter
D. warfarin for life
E. warfarin for 6 months

215. What is true about Vertebral artery course?


A. closely related to vagus
B. enter skull through foramen ovale
C. pass through C6 transverse foramina
D. pass through C7 transverse foramina
E. none of the above

216. 10 y old child with persistent bloody diarrhea. Failure to thrive and anemia. Diagnosis?
A. AV malformation
B. Carcinoma of colon
C. Hurspuring disease
D. IBD
E. Juvenile polyp

217. Young athlete patient came with single rib fracture, has pain and was stable?
A. Oral analgesic
B. Admission, observation and analgesia
C. Infiltration of wound with .5% bupivacaine
D. chest drain insertion at 5th intercostal space
E. None of the above

218. Another patient elderly with chronic bronchitis came with rib fracture, x-ray done with
no hemothorax or pneumothorax, but the patient is in pain
A. Oral analgesic
B. Admission, observation and analgesia
C. Infiltration of wound with .5% bupivacaine
D. chest drain insertion at 5th intercostal space
E. none of the above

219. In postoperative period of thyroidectomy, the patient has symptoms of perioral tingling
and twitches
A. hypocalcemia
B. hypokalemia
C. hyponatremia
D. hypercalcemia
E. hyperkalemia

220. Massive infusion of Saline 0.9% will produce?

125
A. Hyperchloremic metabolic acidosis
B. Hyperchloremic metabolic alkalosis
C. Hyperchloremic respiratory acidosis
D. Hyperchloremic respiratory alkalosis
E. Hypochloremic metabolic alkalosis

221. 53 years old presented by crushing chest pain at rest. He was hypertensive and heavy
smoker, blood pressure 145/9 and pulse 135. What is the principal cause for decrease
coronary blood flow:
A. Decreased diastolic interval
B. Decreased systolic time
C. Increase diastolic time
D. Increase systolic time
E. None of the above

222. A 21-year-old man comes into the Emergency Department with a severe headache and
neck stiffness of recent onset. You suspect acute meningitis and decide immediately to treat
with antibiotics and perform a lumbar puncture. After your third attempt to obtain for
cerebrospinal fluid (CSF) you notice that the fluid obtained is stained red. However, during
the collection of CSF into three containers it is noted that the final bottle is now cleared of
any blood. Which anatomical structure is the most likely to be responsible for this bleeding?
A. Anterior spinal artery
B. Epidural artery
C. Subarachnoid vein
D. Veins within the erector spinae muscles
E. Vertebral venous plexus

223. Female with bilateral femoral and inguinal swelling or lymphadenopathy and spotting
blood in her underwar, what is the asking source of metastasis
A. cervix
B. vulva
C. body of the uterus
D. ovarian tumor
E. none of the above

224. Pain management of patient with renal cancer and has left humorous mets and pain
A. Paracetamol
B. Radiation
C. Non-steroids
D. Morphine
E. Nerve bock

225. 55 years old male smoker and alcoholic with painful bleeding ulcer from lateral side of
tongue, what is your diagnosis?

126
A. Aphthous ulcer
B. Squamous cell carcinoma
C. Benign ulcer
D. leukoplakia
E. none of the above

226. Kidney increase renal perfusion in hypovolemia by which of the following?


A. increase efferent diameter
B. decrease efferent diameter
C. increase afferent diameter
D. decrease afferent diameter
E. increase permeability

227. First response to hypothermia


A. peripheral vasoconstriction
B. decrease sweating
C. shivering
D. hypothalamus regulation
E. none of the above

228. A person has fracture sternum, which structures mostly to be damaged?


A. Left atrium
B. Left lobe of the liver
C. Left ventricle
D. Right atrium
E. Right ventricle

229. What is the Anterior relation of head of pancreas?


A. Bile duct
B. hepatic artery
C. second part of duodenum
D. Right kidney
E. pylorus of stomach

230. 11 years child with sudden acute testicular pain and left one is higher than the right one,
what is the diagnosis?
A. testicular torsion
B. hydrocele
C. orchitis
D. incarcerated hernia
E. none of the above

231. Female patients fall when she was upstairs presented by open triangular flap on the shin
of tibia 7 cm long and 5 cm wide but could not approximated

127
A. Immediate split thickness skin graft
B. Delayed split thickness skin graft
C. Primary closure
D. Delayed primary closure
E. Random free flap

232. Anterior surface gastric ulcer between the antrum and the body of stomach develops
massive bleeding, what is the most accepted artery to be affected?
A. short gastric a
B. right gastroepiploic a
C. left gastric a
D. gastroduodenal artery
E. none of the above

233. The pleural space is negative with respect to atmospheric pressure, except for which of
the following:
A. During a Valsalva maneuver
B. During Muller maneuver
C. During expiration
D. During inspiration
E. During forced inspiration

234. Esophageal varices means increase in pressure in which vessels


A. Short gastric
B. Left gastric
C. Left gastroepiploic
D. Right gastric
E. Right gastroepiploic

235. Male patient hypertensive and his parameters show hyperkalemia and hyponatremia
which of these drugs give this picture
A. Beta agonist
B. Angiotensin 2 blocker
C. alpha blocker
D. B blocker
E. none of the above

236. The "fight or flight", response produces a release of epinephrine (adrenaline). What is
the primary metabolic effect of epinephrine?
A. Alanine shunt activation
B. Cortisol release
C. Glycolysis
D. Tachycardia
E. Vasoconstriction

128
237. 34 years old female presented by calcular gall bladder with tenderness on right
hypochondrium. the site of cholecystitis tenderness is the meeting point between
A. Right linea semilunaris and midaxillary line
B. costal margin and midaxillary line
C. Right linea semilunaris and costal margin
D. linea alba and costal margin
E. none of the above

238. Thyroglossal cyst present in the neck between which muscles


A. cricothyroid and thyroglossus
B. thyropharyngeus and thyroglossus
C. omohyoid and sternomastoid
D. sternothyroid and sternohyoid
E. none of the above

239. 51 years old Patient with crohn disease and made an urgent operation and removing 50
cm from ileum inside the operation, what do you think the nutrition given postoperative
A. Start oral
B. start oral with special formula
C. nasogastric tube
D. TPN with peripheral line
E. TPN at home

240. T wave inversion in lead 3 in unstable hypoxic patient after 3 day from hip replacement
surgery
A. acute myocardial infarction
B. acute massive PE
C. cardiogenic shock
D. hypovolemic shock
E. none of the above

241.76 years old male had hemiarthroplasty, he receive enoxaparin and antibiotic , after 7
days he came hypotensive 85/65 and tachycardia 120 with pleural chest pain with deep
inhalation
A. thromboembolism
B. fat embolism
C. lobar pneumonia
D. tension pneumothorax
E. none of the above

242. Type 2 diabetes millets how to control perioperative


A. variable timing of insulin
B. fixed timing of blood sugar monitored regularly

129
C. blood sugar monitored by patient
D. place on insulin timing scale
E. none of the above

243. Absolute contraindication to contrast study


A. allergy to iodinated contrast
B. GFR 40
C. dehydration
D. sepsis
E. none of the above

244. Athlete runner after long marathon presented by generalized muscle weakness and pain
and abnormal color urine and has low urine volume
A. acute kidney injury
B. specific gravity less than 1.005
C. creatinine kinase more than 300
D. serum myoglobin normal
E. none of the above

130
September 2021 Recalls
1. A patient suffered sensory deficit over the anterior thigh with inability to extend the knee
after an open hernia repair. What is the affected nerve?
A. Obturator nerve
B. Sciatic nerve
C. Genitofemoral nerve
D. Femoral nerve
E. Ilioinguinal nerve

2. A man presented with complete clawing at little and ring, Sensation at the back of hand is
intact and wasting of hypothenar muscle, what is your diagnosis?
A. At the axilla
B. At the elbow
C. At the upper arm
D. At the wrist
E. Deep ulnar nerve injury

3. A 53 years old female presented with a painless dark black eschar affecting her hand. What
is the likely cause?
A. Streptococcus pyogenes
B. Anthrax
C. Campylobacter
D. Staphylococcus aureus
E. Clostridium perfringens

4. A woman had a routine gynecological check. What is palpated anterior to the cervix on PV?
A. base of bladder
B. dome of bladder
C. ischiorectal fossa
D. puborectalis muscle
E. none of the above

5. The male pelvis differs from the female one by all of the following except:
A. the female pelvis is wider and shallower than male
B. the female inlet is rounded or oval in shape while male is apple in shape
C. suprapubic angle is acute while male is large
D. the bone is lighter in female than in male
E. coccyx is straight in female while in male is curves

6. A patient diagnosed that he has a herpes zoster complaining of pain in the tip of the nose.
Which of the following at the same Dermatome of tip of nose?
A. Ear pinna

131
B. angle of mouth
C. jaw angle
D. cornea
E. None of the above

6. Anterior relation of head of pancreas include:


A. pylorus of the stomach
B. common bile duct
C. portal vein
D. IVC
E. Aorta

7. Bleeding is come from Anterior gastric ulcer between the antrum and the body of stomach,
what is the source of bleeding
A. short gastric
B. left gastroepiploic
C. right gastroepiploic
D. left gastric
E. none of the above

8. 55 years old complained of a stroke affecting Right lower extremity paralysis, what is the
arterial supply responsible for that paralysis
A. right anterior cerebral
B. left anterior cerebral
C. right middle cerebral
D. PICA.
E. anterior spinal artery

9. 55 years old suffered from Vitamin deficiency and the patient is alcoholic and has
manifestations of Wernicke's ataxia with cardiomyopathy
A. thiamine deficiency
B. pellagra
C. folic acid
D. iron
E. vitamin B12

10. Which hormone increases pancreatic secretion rich in bicarbonate?


A. Secretin
B. CCK
C. Gastrin
D. Histamine
E. Serotonin

11. Zinker’s diverticulum in pharynx occur between:

132
A. Between thyropharengeaous and cricothyroid muscle
B. Between middle and inf constrictor
C. Between thyropharengeaous and cricopharyngeal muscle
D. Between false and true vocal cords
E. None of the above

12. Target INR for warfarin therapy is:


A. 1:2
B. 2:3
C. 3:4
D. 4:5
E. 5:6

13. Which electrolyte disturbance causes flat p wave and inverted T wave?
A. Hypokalemia
B. Hyperkalemia
C. Hypocalcemia
D. Hypercalcemia
E. Hyponatremia

14. Which diuretic acts on Na/k channel in collecting duct and causes hyperkalemia?
A. Furosemide
B. Thiazide
C. Mannitol
D. spironolactone
E. None of the above

15. During DRE, what will you palpate 4 cm from anal verge posterolaterally?
A. ischiospinous ligament
B. puborectalis muscle
C. Sacro tuberous ligament
D. seminal vesicles
E. bladder

16. All are content of the posterior mediastinum except:


A. Descending aorta
B. Thoracic duct
C. Azygous vein
D. body of vertebrae
E. splanchnic nerves to heart

17. Male 35 years old has blood pressure is 110/80 and ICP is 18, what is cerebral perfusion
pressure
A. 52

133
B. 72
C. 92
D. 112
E. 132

18. Nor-adrenaline exert its inotropic effect through which receptor?


A. alpha1
B. d1
C. d2
D. alpha2
E. beta1

19. Which cell is dependent on Cori cycle for energy:


A. Erythrocyte
B. Muscle cell
C. Hepatocyte
D. Fat cells
E. All of the above

20. 35 years old patient after eating canned food presented by flaccid paralysis and diagnosed
as botulinum toxin affection. What is the mode of action of botulin toxin
A. Block release of acetyl choline from its vesicles
B. Block nicotinic cholinergic receptors
C. Block muscarinic cholinergic receptors
D. close ca release from sarcoplasmic reticulum
E. Block Na channel

21. Vertebral artery true is enters the foramen transversum at which level?
A. C3
B. C4
C. C5
D. C6
E. C7

22. Nerve of 2nd branchial arch?


A. II
B. V
C. VII
D. X
E. IX

23. A man complains of being thirsty and getting up in the middle of the night to go to the
toilet. His weight is 85.5 kg, height 1.65 in and blood pressure 167/94 mmHg. An oral glucose

134
tolerance test was performed and produced the following results: Fasting plasma glucose
11.3 mmol/L The most likely diagnosis:
A. Diabetes insipidus
B. Diabetes mellitus
C. Impaired fasting blood sugar
D. sepsis
E. DKA

24. Caucasian girl with splenomegaly, jaundice and increase bilirubin level. What is the
diagnosis?
A. Spherocytosis
B. Sickle cell anemia
C. Intermittent porphyria
D. ITP
E. none of the above

25. What is the embryological cause of vagino-rectal fistula?


A. Vaginal tube tract
B. Urachus
C. Cloaca
D. Mesonephric duct
E. Metanephric duct

26. After clamping the umbilical cord what will not give oxygenated blood to the embryo any
more
A. Ductus venosus
B. Umbilical artery
C. Ductus arteriosus
D. Pulmonary artery
E. Pulmonary vein

27. A 42-year-old multiparous woman is admitted to the Emergency Department due to


pelvic discomfort. The duty gynecologist diagnoses uterine prolapse. Which anatomical
structure gives significant direct support to the uterus?
A. Cervical ligaments
B. broad ligament
C. Round ligaments
D. Transverse perineal Muscles
E. None of the above

28. A 65 years old man presented with an inguinoscrotal swelling in the right groin which is
non tender. A cough impulse is elicited, at operation, an indirect inguinal hernia is repaired.
The cremasteric muscle is derived from which of the following
A. External oblique aponeurosis

135
B. Internal oblique muscle
C. Rectus abdominal muscle
D. Rectus sheath
E. Transversalis fascia

29. A Patient presented with fracture of the face involving maxilla and numbness over the
cheek. Which of the following nerve is most likely involved?
A. Mental
B. Infra-orbital
C. Superior alveolar
D. Auriculotemporal
E. Supratrochlear

30. Male patient 58 years old with diabetes and chronic AF and come with sudden loss left
lower limb pain, by examination he has no distal pulsation below knee, what is the most
accepted cause of ischemia?
A. Chronic thrombosis
B. Embolic cause
C. Peripheral hypoperfusion
D. Normal Sequalae in old age
E. None of the above

31. A woman sustained needle brick to her thigh. Several days later she noticed red streaks
around site of the brick. What is the most likely cause of this condition?
A. Streptococcus viridians
B. Streptococcus Pyrogens
C. Staphylococcus aureus
D. Streptococcus pneumonia
E. None of the above

32. A 23 years old patient was admitted after falling on his outstretched hand. His
examination revealed that he has lost sensation over the index and middle finger.
What is the most accepted cause of this injury?
A. Hook of hamate fracture
B. Distal radial fracture
C. lunate dislocation.
D. Scaphoid fracture
E. Montagia fracture

33. Biopsy of normal urinary bladder was done; which cell will be found?
A. Stratified Squamous epithelium non-keratinized
B. Columnar epithelium with goblet cell
C. Columnar epithelium without goblet cell
D. Transitional epithelium

136
E. Stratified Squamous epithelium keratinized

34. A surgeon made an incision from below the clavicle to the deltopectoral groove. Which of
the following structures will not be encountered?
A. Medial cord of brachial plexus
B. Axillary vein
C. Thoracoacromial artery
D. Pectoralis minor insertion
E. Middle trunk of Brachial plexus

35. A 54 years old man undergone hip replacement for fractured neck femur. During follow
up, the man walks with the Trendelenburg gait. what is the most likely cause?
A. Sciatic nerve injury
B. femoral nerve injury
C. Superior gluteal nerve injury
D. Gluteus maximus muscle paralysis
E. Inferior gluteal nerve injury

36. A child has Pruritis ani, stool analysis showed ova and cysts in stool. What is the most
appropriate treatment option?
A. Metronidazole
B. Mebendazole
C. Estradiol
D. Tinidazole
E. None of the above

37. A 23 years old Patient underwent ORIF for a lateral malleolus fracture. Below knee cast
was applied. The patient then developed inability to evert his foot and paranesthesia along
the dorsum of the foot. Which nerve injury would be the cause of this case?
A. Common peroneal nerve injury
B. Sural nerve injury
C. Superficial peroneal n injury
D. Deep peroneal nerve injury
E. Tibial nerve injury

38. A 28 years old man developed loss of sensation below the knee after a posterior hip
dislocation. This is associated with weak movements of the leg. What is the most likely
cause?
A. Tourniquet palsy.
B. Sciatic nerve injury.
C. Compartment syndrome.
D. Femoral nerve injury.
E. None of the above

137
39. Which of the following infections is associated with staghorn stones?
A. Pseudomonas
B. E.coli
C. Acinitobacter
D. Staphylococcus aureus
E. Proteus

40. A woman suffered an arm injury. On examination, she is unable to catch the knife as her
hand flexes. There is loss of sensation over the badge area. What is the likely cause?
A. Radial nerve
B. Axillary nerve
C. Posterior cord injury
D. Medial cord injury
E. None of the above

41. Which virus is associated with Kaposi's sarcoma?


A. Human herpes virus 8
B. Human papillomavirus 16
C. Human T-lymphotropic virus 1
D. Epstein-Barr virus
E. Human papillomavirus 18

42. Man presenting with head injury showing deterioration in GCS which becomes 3 with
unilateral unreactive pupil?
A. tentorial Herniating
B. hydrocephalus
C. papilledema
D. Subarachnoid hemorrhage
E. None of the above

43. A 35-yr-old lady presented with bloody nipple discharge. What is your possible diagnosis?
A. Duct ectasia
B. Breast cancer
C. Ductal papilloma
D. Fibroadenoma
E. None of the above

44. Esophageal biopsy shows the columnar epithelium containing goblet cells what is the type
of changes occur in cell?
A. aplasia
B. atrophy
C. hyperplasia
D. hypertrophy
E. metaplasia

138
45. Pt with cervical lymphadenopathy underwent excisional biopsy showed nonnecrotizing
granuloma with giant cell and fibrosis what is your possible diagnosis?
A. Sarcoidosis
B. TB
C. Lymphoma
D. Leprosy
E. Fungal infection

46. 60-year-old man presents with a short history of pain in the right cheek and right upper
teeth Maxillary sinus infection is diagnosed. This sinus is particularly prone to infection
because of?
A. absence of Celia on the epithelium lining the sinus
B. closeness of the sinus to the nasal cavity
C. poor blood supply
D. Position of the sinus ostium high on the medial wall
E. relationship of the front teeth to the floor of the sinus

47. A previously fit young man is admitted after a road traffic accident. He is satisfactorily
intubated because of a head injury. He has a Glasgow coma score of 8 (E2, V4, M4) There are
no other external injuries and initial cardiorespiratory stability is achieved during the primary
survey. He suddenly becomes hypoxic after log- rolling to examine the back. What is the most
likely cause of his deterioration?
A. Cardiac tamponade
B. Ongoing major hemorrhage
C. Spinal injury
D. Tension pneumothorax
E. Tracheal tube displacement

48. You have collected a data of 87 child’s urea and create levels before anesthesia to study
the effect of anesthesia on dehydration. You matched them with the normal levels. What is
the statistical test to use?
A. Paired T test
B. Un paired T test
C. Mann Whitney
D. Chi-square

49. A 65-year-old man is admitted with a two-week history of vomiting. He also had
succasion splash. What of this finding match with this patient?
K+ Na+ PH
A 2.6 mmol/L 148 mmol/L 7.1
B 3.4 mmol/L 156 mmol/L 7.48
C 3.2 mmol/L 118 mmol/L 7.47
D 6.0 mmol/L 120 mmol/L 7.1

139
E 2.8 mmol/L 128 mmol/L 7.2
Normal 3.5-4.9 137-144 7.3: 7.4.

50. Which of the following serves as the axis of gut rotation during development?
A. aorta
B. Superior mesenteric artery
C. Inferior mesenteric artery
D. Umbilical vessels
E. Vitelline artery

51. Abdominal free fluid will collect in the lowest part of the peritoneal cavity. At operation
with the patient supine, in which of the following will the fluid collect first?
A. Hepatorenal pouch
B. Left anterior subphrenic space
C. Lesser sac
D. Right paracolic gutter
E. Right anterior subphrenic space

52. Which of following investigations is most accurate for measuring the GFR?
A. Creatinine clearance
B. Glucose Clarence
C. Inulin Clarence
D. Para-amino-hippuric acid
E. Urea clearance

53. In OT a saline set was connected to bair hugger. Which of the following hormone will
decrease?
A. TSH
B. ACTH
C. ADH
D. Cortisol
E. Catecholamine

54. Which of these I.V fluid have the highest sodium content
A. Saline glucose fluid
B. Normal saline
C. Hartman solution
D. Glucose 5%
E. 4.8% Sodium-bicarbonate

55. All are content of the posterior mediastinum except:


A. Descending aorta
B. Vagus nerve
C. Thoracic duct

140
D. Azygous vein
E. Vertebral body

56. A Child with kwashiorkor developed hepatomegaly. What is the deposit in liver?
A. Sarcoid
B. Melanin
C. Lipofuscin
D. Amyloid
E. fat

57. A child with patent ductus arteriosus and left sternum murmur heard on his chest, what
about this anomaly?
A. Shunt from right atrium
B. Shunt to pulmonary artery
C. Shunt to aorta
D. Shunt to left atrium
E. None of the above

58. Where does the major duodenal papillae open?


A. In the anteromedial part of Third part of duodenum
B. In the posteromedial part of Third part of duodenum
C. In the posteromedial part of 2nd part of duodenum
D. In the anteromedial part of 2nd part of duodenum
E. In the posteromedial part of First part of duodenum

59. Vitamin K is required for normal blood clotting. Which one of the following statements is
true about the effects and availability of vitamin K?
A. Affects platelet function
B. Cause bleeding if taken in excess
C. Is provided only by fresh food
D. Is depleted by oral broad-spectrum antibiotics
E. Vitamin K absorption is affected by gastric resection

60. A 34 years old patient was in operation with tourniquet. After release of tourniquet, what
is the substance released causing VD?
A. Noradrenaline
B. Vasopressin
C. Histamine
D. Serotonin
E. Adrenaline

61. 1-day neonate with history of polyhydramnios intrauterine and after birth, fluid coming
from mouth with dyspnea and cyanosis
A. congenital esophageal atresia with tracheoesophageal fistula.

141
B. congenital diaphragmatic hernia
C. duodenal atresia
D. Hirschsprung
E. None of the above

62. A man has weakness of sternocleidomastoid mastoid muscle and trapezius muscle along
with loss of gag reflex and loss of sensation on palate, where is the lesion?
A. Foramen ovale
B. Foramen rotundum
C. Jugular foramen
D. Foramen lacerum
E. Hypoglossal canal

63. Man had a brain tumor, supplied by a very large middle meningeal artery, which foramen
was enlarged or dilated?
A. Foramen of ovale
B. Foramen rotundum
C. Foramen spinosum
D. Foramen lacerum
E. Jugular foramen

64. A surgeon is carrying out an elective splenectomy for congenital spherocytosis. Which
structure will be divided in order to mobilize the spleen from the posterior abdominal wall?
A. Gastrosplenic ligament
B. Lesser omentum
C. Lienorenal ligament
D. Phrenicocolic ligament
E. Short gastric vessels

65. A woman with increased TSH + lower limit T4 , what is your diagnosis?
A. primary hypothyroidism
B. primary hyperthyroidism
C. Secondary hyperthyroidism
D. Secondary hypothyroidism
E. sick euthyroid

66. In a patient with Cleft palate, it represents defect in the embryological fusion of what
structures?
A. Frontonasal
B. Maxillary
C. palatal processes
D. lateral nasal
E. medial nasal

142
67. A 19-year-old man was assaulted and sustained injuries to the right side of his head. After
two weeks he notices dial his right eye is dry and it could not produce tears. From which
ganglion is post synaptic fibers arise to supply lacrimal gland?
A. Geniculate ganglion
B. Inferior ganglion of the vagus nerve
C. Otic ganglion
D. Pterygopalatine ganglion
E. Superior cervical ganglion

68. Closure of the tricuspid valve occurs at the onset of which phase of the cardiac cycle?
A. Isovolumic relaxation
B. Rapid filling
C. Reduced filling
D. Rapid and reduced ejection
E. Isovolumic contraction

69. Changes happen in septic shock


COP SVR HR
A. High Low High
B. Low Low Low
C. Low High Low
D. Low High High
E. Normal Normal Normal

70. A 72-year-old man has just undergone an emergency repair for a ruptured abdominal
aortic aneurysm. Preoperatively he was taking aspirin, clopidogrel and warfarin. Intra
operatively he received 5000 units of unfractionated heparin prior to application of the aortic
cross clamp. His blood results on admission to the critical care unit are as follows: ull blood
count: Hb 8 g/dl, Platelets 40 * 109/l and WBC 7.1 * 109/l. His fibrin degradation products are
measured and found to be markedly elevated. Which of the following accounts for these
results?
A. Anastomotic leak
B. Disseminated intravascular coagulation
C. Heparin induced thrombocytopenia
D. Adverse effect of warfarin
E. Dearranged platelet function as the drug not stopped

71. What is not reliable to measure oxygen on low tissue perfusion


A. Pulse oximeter
B. ABG
C. Capillary wedge pressure
D. Swan Ganz catheter
E. None of the above

143
72. A person has fracture sternum, which structures mostly to be damaged?
A. Left atrium
B. Left lobe of the liver
C. Left ventricle
D. Right atrium
E. Right ventricle

73. Esophageal varices means increase in pressure in which vessels


A. Short gastric
B. Left gastric
C. Left gastroepiploic
D. Right gastric
E. Right gastroepiploic

74. A man develops septicemia following surgery for perforated acute appendicitis He is
hypotensive. Arterial blood gases reveal: PH= (7.26) PaCo2= (7.2KPA) Pa02= (75 kPa).
Bicarbonate= (17 mmol/L). What is the most likely explanation for these readings?
A. Compensated metabolic acidosis
B. Compensated respiratory acidosis
C. Mixed metabolic and respiratory acidosis
D. Uncompensated metabolic acidosis
E. Uncompensated respiratory acidosis

75. ABG of sepsis after perforated Appendicitis.

76. ABG of DKA

77. In obstetric surgery, Pfannenstiel's incision was made. Which structure is divided?
A. Rectus abdominis
B. arcuate ligament
C. external oblique
D. Rectus sheath
E. none of the above

78. Gastrin release is increased by:


A. Adrenaline
B. Glucagon
C. Calcitonin
D. Acetylcholine
E. None of the above

79. Unique features of axis vertebrae.


A. bifid spinous process
B. carotid tubercle(C6)

144
C. no transverse process
D. no spinous process
E. none of the above

80. A patient suffered a trauma to his thigh leaving a dirty wound with lacerations. Several
days later the wound becomes infected with crepitations. Which Organism?
A. Clostridium deficile
B. Staphylococcus aureus
C. Streptococcus pyogenes
D. Clostridium perfringens
E. Bacillus anthracis

81. A case of intermittent claudication in the calf muscles with loss of pulse below knee only,
where is the most probable site of the arterial lesion?
A. SFA occlusion at the adductor canal
B. CFA before origin of profunda artery
C. External iliac artery
D. Anterior tibial artery
E. Internal iliac artery

82. A 73-year-old man presents with pain in the right buttock. Clinical examination reveals
weakness of abduction of the right thigh. Which nerve is most likely affected?
A. Inferior gluteal nerve
B. Nerve to obturator internus
C. Nerve to quadratus femoris
D. Sciatic nerve
E. Superior gluteal nerve

83. Patient has ventricular hypertrophy, microscopy reveals deposition between muscles.
Cause of hypertrophy?
A. Metaplasia
B. Dysplasia
C. Neoplasia
D. Amyloidosis
E. Reactional hypertrophy

84. Patient has Pink fleshy lesion and diagnosed as a case of multiple myeloma, what is the
diagnosis?
A. Metaplasia
B. Dysplasia
C. AL amyloidosis
D. AA amyloidosis
E. Beta 2 micro amyloidosis

145
85. Lumber puncture is done in which space?
A. Sub Dural
B. Epidural
C. Sub arachnoid
D. Dural
E. Sub pial

86. Patient has Potassium 6mmol/l, which of these is a likely cause?


A. Frusemide
B. Thiazide
C. Acetazolamide
D. Aldosterone
E. Spironolactone

87. Aldosterone acts on?


A. Na/H pump
B. Na/K/2CL pump
C. Na/K pump
D. Na/Ca pump
E. Na/Cl pump

88. Patient presented with dental abscess, causative organism?


A. Staph. Aureus
B. Staph Viridians
C. Klebsiella
D. Enterobacter
E. Bacteroides

89. A boy presented with purpura and fever and lethargy but no palpable lymph node or
spleen, what is your diagnosis?
A. Leukemia
B. Lymphoma
C. HSP
D. ITP
E. Aplastic anemia

90. 2-Month old girl with bowel sound in thorax, what is your diagnosis?
A. Failure of development of right pleuroperitoneal membrane
B. Failure of development of left pleuroperitoneal membrane
C. Cloacal membrane developmental error
D. Mal rotation of gut
E. Ectopia cordis

146
91. Artery moves above the piriformis muscle and divide into two after originating from
external iliac artery. What is the artery?
A. Superior Gluteal
B. Pudendal
C. Inferior gluteal
D. Circumflex iliac
E. Femoral

92. Patient Presented in the AE with an Injury at the back of elbow joint. It was 4cm deep and
6cm above the elbow and there is loss of extension of forearm/elbow joint. Where is the
injury?
A. Brachioradialis
B. Radial nerve
C. Median nerve
D. Triceps
E. Biceps Brachi

93. A baby aspirated a peanut what is probable location of it?


A. Left Upper lobe
B. Right Upper Lobe
C. Left Lower Lobe
D. Left Lower Lobe
E. Lingula

94. Patient has diet-controlled diabetes. He will undergo colectomy. How diabetes is
managed?
A. Regular Observation of blood sugar level
B. Insulin
C. Sliding scale insulin
D. Oral Metformin
E. No intervention required.

95. Loss of pupillary reflex and accommodation, which cranial nerve pair is damaged?
A. 1 and 5
B. 2 and 3
C. 10 and 9
D. 1 and 3
E. 3 and 7

96. Patient has stridor after thyroid surgery which nerve is damaged?
A. Glossopharyngeal
B. Facial
C. Vagus
D. oculomotor

147
E. Stellate ganglia

97. A test was needed to be performed for 87 people regarding dehydration before and after
anesthesia. The values are within normal limit and data are paired. What is the best test?
A. Paired t test
B. Wilcoxon test
C. Mann Whitney U test
D. ANOVA test
E. Chi Square test

98. An error in prescription system causes havoc in the ward with potential damage to
patient, what can be done?
A. Research
B. Inform GMC
C. Medical Audit
D. Risk assessment
E. Incident report

99. Fluid accumulates when patient in supine position?


A. Paracollic gutter
B. Recto-vesical pouch
C. Hepato-Renal pouch
D. Pelvic region
E. Thorax

100. Which pathogen is responsible for pneumonia in sickle cell patient?


A. E. coli
B. Salmonella
C. Strep. Pneumoniae
D. Pseudomonas
E. Staph aureus

101. A 2-year old boy presented with fever, halitosis, hot potato voice, what is the causative
organism?
A. Staph. aureus
B. Strep. Pneumoniae
C. H. influenziae
D. E. coli
E. pseudomonas

102. What is responsible for central respiratory drive?


A. H ion concentration
B. Hco3 ion
C. CO2 in medulla

148
D. Hypoxia
E. None

103. A 45-year old man presented with dilated pupil, where is the probable lesion?
A. Optic nerve injury
B. Trochlear nerve injury
C. 6th Nerve injury
D. Tentorial herniation
E. Facial nerve injury

104. A 10- year old boy presented with scalp injury, what is the best local anesthetic for
primary closure?
A. bupivacaine with adrenaline
B. Lidocaine
C. Lidocaine with adrenaline
D. Levobupivacaine with adrenalin
E. Prilocaine

105. A patient has diminished coronary blood supply what is the responsible mechanism?
A. Increased preload
B. Decreased cardiac output
C. Decreased diastolic interval
D. Decreased systolic interval
E. Increased systolic interval

106. Paresthesia over check which nerve is damaged?


A. Infra alveolar
B. Zygomatic branch of facial
C. Infra trochlear
D. Supra Orbital
E. Infraorbital

107. Acute response in hypothermia?


A. Vasodilation
B. Shivering
C. Peripheral vasoconstriction
D. Sweating
E. Diminished blood supply to hypothalamus

108. Decreased gastric acid secretion By?


A. Histamine
B. Gastrin
C. PE2
D. NSAID

149
E. Vagal stimulation

109. Lymph Node metastasis from scrotal carcinoma?


A. Vertical group of inguinal
B. Iliac
C. Para aortic
D. Medial group of inguinal
E. Popliteal

110. S3 Dermatome Territory?


A. Perianal
B. Ischial Tuberosity
C. Anterior Scrotal
D. Perineal
E. Inguinal Region

111. Anal Carcinoma metastasis to?


A. Iliac LN
B. Popliteal LN
C. Superficial Inguinal LN
D. Deep Inguinal LN
E. Obturator lymph node

112. A Patient present in AE department with head injury, Blood pressure 90/70, passed urine
after one hour, what should be possible composition of urine
A. 30ml diluted urine
B. 30 ml concentrated urine
C. 50ml diluted urine
D. 300 ml diluted urine
E. 300 ml concentrated urine

113. A surgeon is incising hydrocele until the fluid comes and found testes is covered by?
A. Tunica vaginalis (parietal layer)
B. Tunica albuginea
C. Tunica vaginalis – visceral layer
D. Cremasteric Muscle
E. Dartos muscle

114. A 28-year-old girl with single episode of blood stained discharge from right nipple with
no palpable mass what is the diagnosis?
A. Duct ectasia
B. Duct Carcinoma
C. Duct Papilloma
D. Plasma cell mastitis

150
E. Lobular carcinoma

115. Which System activates after hemorrhage?


A. Bain Bridge
B. Respiratory
C. RAA system
D. Myogenic Contraction
E. None of the above

115. A 60-year-old patient had elective esophagectomy causing injury to thoracic duct, where
is it injured?
A. Diaphragmatic hiatus to abdomen
B. Crus of Diaphragm
C. Diaphragmatic hiatus to superior mediastinum
D. In middle mediastinum
E. Superior mediastinum in right.

116. Direct inguinal hernia is due to defect in?


A. Rectus abdominis
B. fascia transversalis
C. External oblique aponeurosis
D. Internal oblique
E. None of the above

117. A patient present with hypovolemia due to severe dehydration. How kidney preserve
fluid in this situation?
A. Efferent vasoconstriction
B. efferent vasodilation
C. Diminished renal blood flow
D. Diminished GFR
E. Increased flow in kidney

118. What form the posterior layer of inguinal canal?


A. External oblique aponeurosis
B. Conjoint Tendon
C. Lacunar ligament
D. Pectineal ligament
E. Rectus femoris muscle

119. Cutaneous supply of ear lobule?


A. vagus
B. Auriculo temporal
C. greater auricular
D. Nervous intermedius

151
E. Maxillary nerve

120. Isotonic Fluid?


A. 0.45% NACL
B. 3% NACL
C. 0.9% NACL
D. 0.25%NACL
E. None

121. Which fluid has more sodium content?


A. 0.9% NACL
B. Hartman solution
C. 5% DA
D. 4.8% sodium bicarbonate
E. 0.45% normal saline

122. What increases in hemorrhage?


A. Insulin
B. Plasma
C. Angiotensin II
D. Plasmin
E. Thrombin

123. A 30-year man present with abdominal pain and diarrhea. In barium follow through it
was found that barium moves from ileum to sigmoid colon, what is the diagnosis?
A. chrons’
B. Sigmoid diverticulosis
C. AVM
D. Colorectal carcinoma
E. IBS

124. A man experienced loss of taste sensation from anterior 2/3rd of tongue which ganglion
is affected?
A. Gasserian
B. Ottic
C. Geniculate
D. Pterygopalatine
E. Sub mandibular

125. A 30-year-old female has family history of breast cancer and her mother & aunt died of
Breast cancer, what she should do to avoid breast cancer?
A. Abstain from alcohol
B. Take OCP
C. NSAIDS

152
D. Avoid early gestation
E. None of the above

126. A hypothyroid person present with low MCV and Low iron content in blood film.
What is the main concern of surgeon during operation?
A. Hypovolemia
B. Hypothermia
C. Iron Deficiency Anemia
D. Heart contractility
E. Wound infection

127. A 80-year old man presented with weight loss and altered bowel habit, imaging was
done and found Cecum cancer, which artery to ligate?
A. MMA
B. IMA
C. Ileo-Colic
D. Middle rectal
E. Superior Rectal

128. A middle age man present with scrotal pain, no abnormality during supine, but during
standing he has cystic mass in cord with cough impulse, what is the diagnosis?
A. Hematocele
B. Hydrocele ??(encysted hydrocele of the cord)
C. varicocele
D. Carcinoma
E. Hernia

129. Vertebral artery enters transverse process of which vertebra?


A. C6
B. C5
C. C8
D. C4
E. C2

130. Medial 3 and a half toe has no sensation in planter aspect, what is the nerve being
injured?
A. Lateral plantar
B. Sural
C. Saphenous
D. Common peroneal
E. Medial planter

131. Parasympathetic causes?


A. Pupil dilation

153
B. Diminished blood flow to skeletal muscle
C. Increase Peristalsis
D. Decrease gastric emptying
E. Ejaculation

132. What diminishes gut motility?


A. Ach
B. Dopamine
C. Dobutamine
D. Nor adrenaline
E. ANP

133. Relation of the contents of the popliteal fossa?


A. artery superficial
B. artery between vein and nerve
C. Artery lies deep to nerve and vein
D. Artery lies medial to nerve in lower part
E. Artery lies lateral to nerve in upper part.

134. Diminished sensation on medial aspect of arm, injury in


A. Median nerve
B. Ulnar nerve
C. Radial nerve
D. Pan coastal tumor
E. Upper trunk of brachial plexus

135. Metabolic abnormality in metastatic carcinoma?


A. Hypokalemia
B. Hypercalcemia
C. Hypokalemia
D. Hypo Kalema
E. Hypo natremia

136. Pancreatitis with paresthesia and muscle weakness, Electrolyte abnormality?


A. Hypo natremia
B. Hypokalemia
C. Hypo-calcemia
D. Hypercalcemia
E. Hypo-thyroidism

137. A 45-year old female present with urinary retention and lower limb diminished
sensation which artery is involved?
A. Anterior spinal
B. Posterior Spinal

154
C. Artery of Adamckweiz Great Anterior Radiculmedullary artery / Arteria Radicularis Anterior Magnum
D. Vertebral
E. Median sacral

138. A patient has Bp 100/70 HR 115 and SPO2 99% after operation. He has diminished urine
output, what is the cause?
A. PE
B. MI
C. Hypovolemia
D. Septic shock
E. Overdose of analgesic

139. A-60-year-old has multiple adhesions, a gastroenterologist was performing


cholangiography, she cannot bear the pain and asked to stop the investigation, she has no
any other abnormality. What to do next?
A.MRI
B. CT colography
C. laparotomy
D. Biopsy
E. Laparoscopy

140. A patient has a laparotomy wound which is normally healing for 2 months.
Which healing process is occurring?
A. vasculogenesis
B. Angiogenesis
C. Neutrophil infiltration
D. Diminished blood supply
E. Granulation tissue

141. Destruction of bacteria is enhanced by?


A. Seminization
B. Neutralization
C. Opsonization
D. Pinocytosis
E. None of the above

142. Arteriolar dialation is caused by?


A. Serotonin
B. Adrenalin
C. Nor-adrenalin
D. Histamine
E. Angiotensin ii

155
143. A 23-year-old female present with breast pain; no palpable mass was found and
analgesic was given which subsided her symptoms. Now again present with breast pain, what
to do next?
A. Reasure
B. MRI
C. Biopsy
D. Mammogram
E. USG

144. Patient had mesenteric embolus what is the next investigation?


A. MRI
B. CT abdomen
C. CT angiography
D. USG
E. X-ray

145. Management of post gastrectomy megaloblastic anemia?


A. Methionine
B. Iron
C. B6
D. Folic acid
E. B12

146. A patient has supra clavicular lymph node with iron deficiency anemia, what is the
diagnosis?
A. Bronchial carcinoma
B. Renal carcinoma
C. Colonic carcinoma
D. Laryngeal carcinoma
E. Gastric carcinoma

147. Causes of cleft palate?


A. Unfused primary palate
B. Unfused palatine shelves
C. Unfused maxilla
D. Unfused nasal process and maxillary process
E. None

148. Farmer present with black eschar what is the diagnosis?


A. BCC
B. SCC
C. Clostridium histolyticum
D. Bacillus anthraces
E. Nocardia

156
149. A man presented with necrosis and crepitus what is the causative organism?
A. Staph aureus
B. Staph. Epidermidisae
C. Streptococcus pyogens
D. Clostridium perfringens
E. Bacillus anthraces

150. A patient present with lung cancer with cerebral metastasis what to give next?
A. Dexamethasonae
B. Cyclizine
C. Ondansetron
D. aspirin
E. Chemotherapy

151. Which nerve injury in loss of sensation over deltoid?


A. Musculo cutaneous
B. Median
C. Radial
D. Axillary
E. Radial.

152. Patient has injury in lower thigh with absence of planter flexion which nerve is
damaged?
A. Sural
B. tibial
C. Femoral
D. Common peroneal
E. Planter

153. Loss of flexion of wrist and loss of sensation over the ulnar side of forearm, nerve
affected?
A. Upper trunk
B. Lower trunk
C. Middle trunk
D. Median
E. Ulnar

154. hypokalemia Ecg?


A. ST elevation
B. Prolong QRS
C. Wide T
D. Tall T
E. Inverted T

157
155. Septic Shock Finding
A. Increased HR Increased SVR Decreased CO
B. Increased CO decreased SVR Increased HR
C. Increased BP Decreased HR Decreased SVR
D. Decreased HR Decreased SVR Decreased CO
E. none of the Above

156. Imaging for DVT


A. Compression Ultrasound
B. CT
C. Venography
D. Duplex
E. None

157. Determinant of cerebral blood flow in head injury


A. Hypocapnia
B. Hypoxia
C. ICP
D. Parasympathetic stimulation
E. Heart rate

158. What happen in ARDS?


A. Reduced diffusion
B. Co-officiant of diffusion
C. Embolism
D. Thrombosis
E. None

159. Bleeding after intercoastal intubation?


A. Damage to intercoastal vein
B. Damage to intercoastal artery
C. DCA
D. Musculophrenic artery
E. Mammary artery.

160. Enlarged lymph node with Non-necrotizing granuloma, cause?


A. Lymphoma
B. TB
C. Syphilis
D. Cat scratch disease
E. Multiple myeloma

161. A man presented with solitary thyroid nodule. What is the best investigation?

158
A. CT scan
B. Ultrasonography
C. FNAC
D. Radioisotope Scan
E. Functional Imaging

162. A woman 50 years old has exophthalmos, goiter. Previously diagnosed as Graves’
disease. Now presented with recurrent relapse. Best management option?
A. Propylthiouracil
B. Carbimazol
C. Thyroidectomy
D. Iodine ablation
E. Propranolol

163. 8-Month-old baby presented with upper neck cystic mass which is attached to hyoid
bone. What is the diagnosis?
A. Branchial Cyst
B. Congenital Dermoid cyst
C. Thyroglossal cyst
D. Epidermoid cyst
E. Cystic Hygroma

164. 7-year old boy was attacked by a dog. He has laceration on face. Skin over the nose was
lost resulting in exposure of alar cartilage. What should be the management?
A. Primary closure
B. Debridement and healed by secondary intention
C. Graft
D. Local flap
E. Pedicalated flap

165. A 65-year-old man has Basal cell carcinoma. Excision of the lesion kept 1cm gap in the
face. What is the management?
A. STSG
B. Full thickness graft
C. Local flap
D. Primary closure
E. Conservative dressing

166. A 75-year-old diabetic male presented with black small toe. What is the management?
A. Debridement and healed by secondary
B. Primary Closure
C. Graft
D. Flap
E. Conservative dressing

159
167. A biker fell of his bike and injured his leg. In AE department, he was examined and found
that there was laceration over the leg exposing Tibia. What is the management?
A. Conservative dressing
B. STSG
C. Debridement and closure
D. Local flap
E. Full Thickness graft

168. A motor biker fell on the highway and presented with a closed fracture of tibia at the
lower end. What is the best management?
A. IM nail
B. Screw and plate fixation
C. Delta Frame
D. Plaster Cast
E. Dynamic Hip Screw

169. 8-year-old girl presented with minimally displaced tibia fracture. What is the
management?
A. Cannulated screw
B. Below knee cast
C. Groin to toe cast
D. Hip spica
E. IM nail

170. A Chinese woman presented with epistaxis and conductive deafness with a mass in neck.
What is your diagnosis?
A. Maxillary carcinoma
B. Adenoid Cystic Carcinoma
C. Pleomorphic adenoma
D. Nasopharyngeal carcinoma
E. Ethmoidal polyp

171. A man had cholecystectomy few weeks back. Now he presented with intermittent
jaundice. What is your diagnosis?
A. Carcinoma Pancreatic head
B. Mirizzi syndrome
C. Residual stone in bile duct
D. Cholangitis
E. Fibrosis of bile duct

172. A 32-year-old man presented with fever, upper quadrant pain and pale stool and high
color urine. What is your diagnosis?
A. Acute Cholecystitis

160
B. Chronic Cholecystitis
C. Carcinoma head of the pancreas
D. Gilbert Syndrome
E. Cholangitis

173. A man presented with thymic mass and biopsy revealed glandular type of epithelium.
What is the diagnosis?
A. Squamous cell carcinoma
B. Thymoma
C. Basal cell carcinoma
D. Adenocarcinoma
E. Metaplasia

174. A 12 years old overweight child presented with obvious limping and hips moves into
external rotation when flexed. He denies any history of trauma. What is the most accurate
diagnosis?
A. Perthes disease
B. Slipped upper femoral epiphysis
C. Developmental dysplasia of the hip
D. Septic arthritis
E. Osteoarthritis

175. A female patient presented with sharp pain over the dorsum of the foot radiating to the
toes. On examination, there is tender swelling in the cleft between 3rd and 4th metatarsal
bones. What is the most likely cause?
A. March fracture
B. Freiburg disease
C. Morton’s neuroma
D. Osteoarthritis
E. None of the above

176. A 30 years old athlete sustained trauma to his right knee which became rigidly flexed at
30 degrees. Trials to flex or extend knee were unsuccessful and he mentioned that his knee
gives away. What is the most likely cause?
A. Anterior cruciate ligament injury
B. Loose body
C. Dislocated patella
D. Meniscal tear
E. Posterior cruciate ligament injury

177. During a game of football, Andrew experienced twisted injury. The next day emergency
house officer found a patellar tap. After 4 weeks he experienced same type of injury. What is
your diagnosis?
A. Anterior cruciate ligament injury

161
B. Knee Dislocation
C. Dislocated patella
D. Medial Meniscal tear
E. Lateral collateral ligament injury

178. A 53 years old female presented with loss of sensation over the lateral aspect of the leg
after a varicose vein surgery. What is the affected nerve?
A. Sural nerve
B. Saphenous nerve
C. Superficial peroneal nerve
D. Deep peroneal nerve
E. Tibial nerve

179. A man suffered a fracture neck of femur. ORIF was done through a posterior hip
approach. Postoperatively, he complained weakness in his left leg. What is the likely cause?
A. Tourniquet paralysis
B. Sciatic nerve injury
C. Femoral nerve injury
D. Compartmental syndrome
E. None of the above

180. A patient with Gouty arthritis undergone joint aspiration. What is the expected finding
under the microscope?
A. Needle shaped crystals and negative birefringence
B. Rhomboidal shaped crystals and negative birefringence
C. Needle shaped crystals and positive birefringence
D. Rhomboidal shaped crystals and positive birefringence
E. None of the above

181. After tibia fracture a man was managed with IM nail, he develops pain in the leg after
few days. What is the least reliable sign of compartment syndrome?
A. Increasing pain out of proportion
B. Altered sensation
C. Increased compartment pressure
D. Pain on passive stretch
E. Loss of arteria dorsalis pedis pulse

182. A man has trauma to his face affecting orbit with periorbital hematoma,
Subconjunctival hemorrhage and epistaxis. What is the possible fracture site?
A. Nasal bone
B. Naso-ethmoidal
C. Maxillary
D. Zygomatic
E. None of the above

162
183. 35 yrs. old female on the routine mamo has intermediate microcalcification and the
mammogram was negative 3 months ago with no lymph node. What is your Diagnosis?
A. Ductal carcinoma in situ
B. Lobular carcinoma in situ
C. Invasive ductal carcinoma
D. Lobular carcinoma
E. Paget's disease

184. A 28 years old patient stabbed on right side posterior to axillary line, X-ray showing
pneumothorax with fluid level. What is the best management?
A. Chest drains with suction
B. Needle Decompression
C. Thoracentesis
D. Chest drain
E. None of the above

185. A 25-year-old male pedestrian is hit by a van on a busy road. He is brought to the
Emergency Department by ambulance. On examination he is dyspneic, and hypoxic despite
administration of high flow 100% oxygen. His blood pressure is 100/80 and pulse rate is 110
bpm. The left side of his chest is hyperresonant on percussion and has decreased breath
sounds. The trachea is deviated to the right. What is your initial management?
A. Needle in 5th intercostal space at the mid axillary line
B. Between Xiphoidal process and left sternocostal margin
C. Between Xiphoidal process and right sternocostal margin
D. 3rd intercostal space
E. None of the above

186. A 50-year-old women presented with a neck mass. On examination, she also had a
fluctuant pulsatile mass on her fore head. What is the most likely underlying diagnosis?
A. Follicular thyroid cancer
B. Medullary thyroid cancer
C. Papillary thyroid cancer
D. Anaplastic thyroid cancer
E. Parathyroid gland cancer

187. A 28-year-old Patient came to ER assessment of consciousness level was done according
to GCS patient can localizes pain, open eye to pain and no verbal response what the GCS?
A. 10
B. 7
C. 12
D. 8
E. 13

163
188. A 30 years old patient brought to ER with rib fracture patient is intubated, then becomes
hypotensive and hypoxic what is your diagnosis?
A. Tension pneumothorax
B. Hemothorax
C. Hemopericardium
D. Hemopneumothorax
E. None of the above

189. Groin mass in a child appears during the time of bath and disappear during the night
what is the pathology?
A. Dartos muscle
B. Patent processes vaginalis
C. Testicular capsule
D. Tunica albuginea
E. Tunica vasculosa

190. Patient with congenital anti thrombin 3 deficiency came to do an operation, what will
you prescribe for her postoperative
A. LMWH s/c for 6 months
B. Oral anticoagulant for life
C. Stocking for 6 months
D. Injectable anticoagulant for life
E. None of the above

191. 4 Month child with failure to thrive with recurrent chest infection and dyspnea increase
with lying flat in the night and awake him from sleep
A. Tracheosophageal fistula
B. Congenital diaphragmatic hernia
C. Asthma
D. GERD
E. None of the above

192. 1-day neonate presented by bilious vomiting and scaphoid abdomen


A. Esophageal atresia
B. CHPS
C. Duodenal atresia
D. HSD
E. Volvulus

193. 10 months boy is screaming with passing dark blood, sausage shaped abdominal mass in
examination with free DRE
A. Intussusception
B. Meckel's diverticulum
C. juvenile polyp

164
D. anal fissure
E. anal hemorrhoid

194. A patient with Malignant melanoma. What is the best prognostic factor
A. Nodal Status
B. Completeness of excision
C. No lymph nodes involved
D. No distant spread
E. 0.6 mm thickness

195. A patient will do hemorrhoids and his HB is 9 he put in a list after 6 weeks how would
you give him supplement
A. blood transfusion
B. IV IRON
C. oral iron
D. vitamin B
E. folic acid

196. A child has got a Water burn (scald) covering whole dorsal forearm with erythema with
blisters, what is your management?
A. occlusive dressing
B. keep it open
C. split thickness skin graft
D. full thickness skin graft
E. flap

197. Woman has a gunshot to her back, which transect cord presents with features suggestive
of a Brown-Sequard syndrome. The clinical findings will include which of the following?
A. Ipsilateral sided weakness, Ipsilateral proprioception and vibration loss,
Contralateral sided loss of pain and temperature
B. Contralateral sided weakness, Ipsilateral sided proprioception and vibration
loss, Contralateral sided loss of pain and temperature
C. Ipsilateral sided weakness, Contralateral sided proprioception and vibration
loss, Contralateral sided loss of pain and temperature
D. Ipsilateral sided weakness, Contralateral sided proprioception and vibration
loss, Ipsilateral sided of loss of pain and temperature
E. None of the above

198. Tourniquet used in operation to control blood loss and give bloodless field is based on:
A. after exsanguination Inflation of cuff above diastolic
B. Inflation of cuff above mean before exsanguination
C. Inflation of cuff above mean before exsanguination
D. Inflation of cuff above systolic before exsanguination
E. Inflation of cuff above systolic after exsanguination

165
199. 35 years male presented by back pain and weakness of lower limbs due to central disc
prolapse with lower motor neuron lesion manifestation
A. Central cord syndrome
B. syringomyelia
C. brown Sequard syndrome
D. posterior cord syndrome
E. anterior cord syndrome

200. Male patient 39-year-old presented with sudden chest pain and hematemesis which
occurs after aggressive vomiting. By examination he has subcutaneous crepitus sensation
over the upper chest, what is the most accepted diagnosis?
A. Mallory Weiss syndrome
B. Borene syndrome
C. Rupture Esophagus
D. Rupture Zenker diverticulum
E. Rupture pulmonary plebs

201. A 67-year-old lady presents with jaundice and abdominal pain. Her investigations show a
dilated common bile duct, a carcinoma of the pancreatic head compressing the pancreatic
duct. Her liver contains bi-lobar metastasis.
A. Gastrojejunostomy
B. Pancreatoduodenectomy
C. Conservative management
D. MRI guided pancreatic stent
E. Endoscopic pancreatic stent

202. 2 years old child presented with Arnold chiari syndrome due to cerebellar tonsil
herniation and associated syringomyelia with affection of which tract
A. Dorsal column
B. Spinothalamic tract
C. Corticospinal tract
D. Extrapyramidal tract
E. None of the above

203. Management of false femoral aneurysm?


A. USG observation
B. Endovascular repair
C. Ultrasonographic compression
D. surgical repair
E. None of the above

204. A 25 years old athlete came with a single rib fracture. He is in pain, but he is vitally
stable. What is the most appropriate management?

166
A. Chest drain
B. Admission, observation and analgesia
C. Analgesia and discharge
D. Infiltration of wound with 5% bupivacaine
E. None of the above

205. A 59 years old COPD patient came with a rib fracture. X-ray was done with no hemo or
pneumothorax detected, but the patient is in pain. What is the most appropriate
management?
A. Chest drain
B. Admission, observation and analgesia
C. Analgesia and discharge
D. Infiltration of wound with 5% bupivacaine
E. None of the above

206. A 20 years old man with history of a pinned midtarsal fracture 7 years ago is now
presenting with tarsal pain at the site of fracture. X-ray would likely show:
A. Osteoarthritis
B. Rheumatoid arthritis
C. Malunion
D. Nonunion
E. Avascular necrosis

207. A 38 years old man sustained a tibial fracture for which he undergone closed reduction
and casting. Several hours later, he developed severe pain and numbness in his leg. What is
the most appropriate management?
A. Analgesia and follow up
B. Assess pulsation
C. Compartmental pressure management
D. 4 compartment decompression
E. None of the above

208. A 70-year-old man presents to his General Practitioner with back pain that has been
worsening over the last six weeks. He also complains of slow urinary flow. Digital rectal
examination reveals an irregular hard prostate. Prostate specific antigen is 300 ng/ml (normal
<4). What should the first line of treatment be:
A. Bladder neck incision
B. Hormonal manipulation
C. Radical prostatectomy
D. Transurethral resection of prostate
E. Watchful waiting

167
209. A boy developed acute scrotum after trauma with edematous tender left hemiscrotum.
On examination there’s a tender testis which is higher than the other one. The most likely
underlying cause is:
A. Testicular torsion
B. Testicular hematocele
C. Hydrocele
D. Teratoma
E. Spermatocele

210. A 50-year-old woman sustains a displaced distal radial fracture. This is manipulated
under anesthetic and treated in a cast for six weeks. After three months she returns to the
fracture clinic with painful and limited pronation and supination. Which of the following is
the most probable diagnoses?
A. Complex regional pain syndrome Type 1 (reflex sympathetic dystrophy)
B. Malunion
C. Non-union
D. Posterior interosseous nerve palsy
E. Radial nerve compression

211. A 62 years old man developed hematuria and weight loss over the last 2 months.
Abdominal U/S showed a 10 cm mass in the right kidney. What is the most appropriate
investigation?
A. MRI
B. Biopsy
C. CT scan
D. Ascending urethrogram
E. None of the above

212. A 63 years old diabetic patient presented with late onset painless urine retention
following a total hip replacement surgery. What is the most likely cause of this condition?
A. Underactive bladder
B. Stone
C. Urethral stenosis
D. Bladder neck inactivity
E. None of the above

213. A 79-year-old woman with hypochromic microcytic anemia and axillary and inguinal
lymphadenopathy what is your diagnosis?
A. Bronchial cancer
B. Non-Hodgkin lymphoma
C. Hodgkin lymphoma
D. Gastric cancer
E. None of the above

168
214. A 70-year-old female presented with thyroid mass rapidly growing over 3month, what is
your possible diagnosis?
A. follicular Carcinoma
B. Papillary carcinoma
C. Anaplastic carcinoma
D. Medullary carcinoma
E. None of the above

215. What is the defective gene in a patient with breast cancer?


A. P53
B. BRCA I
C. APC
D. SIS
E. None one the above

216. Discomfort. He has never attended the hospital previously and is usually well. He has
just retired from full time employment as a machinist in a PVC factory. CT scanning shows a
large irregular tumor in the right lobe of his liver.
Which of the following lesions is the most likely?
A. Liposarcoma
B. Angiosarcoma
C. Hamartoma
D. Hydatid liver disease
E. Benign angioma

217. A 35-year-old female, pregnant, presented with chest pain. Her examination was normal
apart from HR 90. Her father died sudden at age of 50 years old due to MI. What is the most
accepted diagnosis?
A. Anginal pain
B. Pleural effusion
C. Aortic dissection??
D. Lung abscess
E. Esophageal rupture

218. What is the best long-term management for post-phlebitis syndrome?


A. Heparin for 7 days
B. Long life warfarin
C. Lower limb stocking
F. Warfarin for 6 months
D. Antibiotics

219. 45 years old patient with severe head injury 14 days ago. He is now has slow recovery,
what type of feeding this patient will you need?
A. TPN

169
B. Nasogastric tube
C. Feeding jejunostomy
D. Enteral with special formula
E. Oral intake

220. A patient will do hernial repair and his Hemoglobin is 9 he put in a list after 6 weeks how
would you give him supplement
A. blood transfusion
B. plasma transfusion
C. IV iron
D. oral iron
E. folic acid

221. 50-year-old patient for elective hernia repair as the day case, he is diabetic type 1 and
has autonomic neuropathy, what is the main problem as a day case in this patient?
A. Reduced pulse rate variability
B. Increase urine output
C. Bleeding
D. Ischemia of the heart
E. Orthostatic hypotension

222. Female presented by 20% burn over her chest, for which she underwent graft treatment,
10 days later she came with sweeping discharge from the graft edges, fever and chest pain,
what medication you will give?
A. Flucloxacillin
B. Cephalosporin
C. Cephalosporin and Metronidazole
D. Ampicillin
E. None of the above

223. Patient injured his tip of index finger, what is the best type of anesthesia to be used in
ring block?
A. Lidocaine
B. Lidocaine with adrenaline
C. Bupivacaine
D. Bupivacaine with adrenaline
E. Prilocaine

224. Overweight person and doing hip surgery. Which of the following is not considered a
measure to enhance rapid recovery?
A. oral carbohydrate 2 hours preoperatively
B. weight reduction before surgery
C. anti-emetic
D. diazepam

170
E. Epidural analgesia

225. Porcine skin graft is a type of


A. xenograft
B. Isograft
C. Autograft
D. Allograft
E. None of the above

226. 72-year-old man is due to undergo an esophagectomy for malignancy. His BMI is 17.5.
What is the best feeding regime immediately following surgery?
A. Total parenteral nutrition.
B. Feeding jejunostomy.
C. Feeding duodenostomy.
D. Liquid diet orally.
E. Soft solids orally.

227. Man in RTA, complained of LL pain and lower abdominal pain, blood pressure 85/60,
heart rate 110 and conscious and talking GCS not mentioned, what is the initial management?
A. massive hemorrhagic protocol
B. morphine for pain
C. tracheal intubation
D. platelet transfusion
E. O blood transfusion

228. Recurrent pulmonary embolism despite of heparin use


A. LMWH
B. LMWH and stocking
C. vena cava filter
D. warfarin for life
E. warfarin for 6 months

229. 10 y old child with persistent bloody diarrhea. Failure to thrive and anemia. Diagnosis?
A. AV malformation
B. Carcinoma of colon
C. Hurspuring disease
D. IBD
E. Juvenile polyp

230. In postoperative period of thyroidectomy, the patient has symptoms of perioral tingling
and twitches
A. hypocalcemia
B. hypokalemia
C. hyponatremia

171
D. hypercalcemia
E. hyperkalemia

231. Massive infusion of Saline 0.9% will produce?


A. Hyperchloremic metabolic acidosis
B. Hyperchloremic metabolic alkalosis
C. Hyperchloremic respiratory acidosis
D. Hyperchloremic respiratory alkalosis
E. Hypochloremic metabolic alkalosis

232. Female with bilateral femoral and inguinal swelling or lymphadenopathy and spotting
blood in her underwear, what is the asking source of metastasis?
A. cervix
B. vulva
C. body of the uterus
D. ovarian tumor
E. none of the above

233. Pain management of patient with renal cancer and has left humorous metastasis and
pain
A. Paracetamol
B. Radiation
C. Non-steroids
D. Morphine
E. Nerve bock

234. 76 years old male had hemiarthroplasty, he receives enoxaparin and antibiotic, after 7
days he came hypotensive 85/65 and tachycardia 120 with pleural chest pain with deep
inhalation
A. thromboembolism??
B. fat embolism
C. lobar pneumonia
D. tension pneumothorax
E. none of the above

235. Type 2 diabetes mellitus how to control perioperative


A. variable timing of insulin
B. fixed timing of blood sugar monitored regularly
C. blood sugar monitored by patient
D. place on insulin timing scale
E. none of the above

236. Athlete runner after long marathon presented by generalized muscle weakness and pain
and abnormal color urine and has low urine volume

172
A. acute kidney injury
B. specific gravity less than 1.005
C. creatinine kinase more than 300
D. serum myoglobin normal
E. none of the above

237. A man underwent esophagogastrostomy for cancer esophagus. Post operative period he
lost significant weight. What should you do?
A. TPN
B. Nasogastric tube feeding
C. Intraoperative jejunostomy
D. Gastrostomy
E. None of the above

238. A 23-year-old patient present to E/R department with Head trauma, apart from multiple
episode of vomiting he was normal and conscious. What is your management?
A. Refer to neurosurgery
B. CT head within 1 hour
C. CT head within 8 hours
D. MRI brain
E. None of the above

239. Patient with weak pulse below knee but has pain in movement of hip, what is your
diagnosis
A. Osteoarthritis
B. Embolus
C. Thrombosis
D. Fat embolism
E. Aneurysm

240. 70-year-old man with carcinoma of the bronchus presents with blurring of vision,
headaches and nausea, particularly in the morning. Which of the following is the most
Appropriate treatment?
A. Carbamazepine
B. Dexamethasone
C. Morphine elixir
D. Paracetamol
E. Radiotherapy

241. Patient came with fever, pain on swallowing and Uvula deviated to one side, what is
your diagnosis?
A. Ludwig angina
B. Acute tonsillitis
C. Carcinoma

173
D. Quinsy
E. Oropharyngeal Carcinoma

242. Metabolic abnormality in Cushing Disease?


A. Low sodium
B. High potassium
C. Low calcium
D. Low potassium
E. High Phosphate

243. Patient has long standing rheumatoid arthritis. Now presented with renal failure, what
type of amyloidosis in renal failure?
A. AA
B. AL
C. Beta
D. BB
E. Micro

244. A 2-month-old boy needed to undergo lumber puncture. What is the best site for
puncture?
A. S1/S2
B. L2/L3
C. L4/L5
D. T12/L1
E. L1/L2

245. A 16-year boy with loin to groin pain with hematuria, no history of trauma, what is your
diagnosis?
A. Urethra rupture
B. Wilms tumor
C. Ureteric stone
D. Pyelonephritis
E. Bladder rupture

246. There was a collision of two boats, patient presented with bp 110/80 and HR 110 and
conscious, he complaints severe pain in leg and abdomen. What is the best initial
management?
A. Acute hemorrhagic protocol
B. IV morphine
C. Intubation
D. Transfer to neurosurgery
E. Antibiotic

174
247. 19 Year boy presented with mild jaundice and pale stool, which type of antibody can be
found?
A. Anti-Phospholipid
B. Anti- endomysial
C. Anti-mitochondrial
D. Anti-cardiolipin
E. ANCA

248. A 2-year-old toddler is presented with black stool in nappy, what is the diagnosis?
A. IBD
B. Intussusception
C. Meckel’s diverticulum
D. IBS
E. Juvenile polyp

249. A 3-year-old boy with painless bright red rectal blood. No other abnormality was found.
What is the diagnosis?
A. Anal fissure
B. Anal fistula
C. Rectal Polyp
D. AVM
E. Hemorrhoid

250. A patient presented with respiratory distress. He was in ITU for 8 days post-Op 8 months
back. What is the diagnosis?
A. Subglottic stenosis
B. Tracheomalacia
C. Laryngomalacia
D. Supra Glottic stenosis
E. Tracheal fistula

251. A man hailing from Zimbabwe presented with hematuria. What is the abnormality you
can find in cystoscopy and biopsy?
A. Transitional cell carcinoma
B. Bladder polyp
C. Dysplasia
D. Squamous metaplasia
E. Columnar metaplasia

252. A smoker presented with cough and hemoptysis. Imaging found cavitation lesion, what
is the diagnosis?
A. Metaplasia
B. SCC
C. BCC

175
D. Adenocarcinoma
E. Small cell carcinoma

254. A 10-year-old boy present with recurrent UTI, low potassium, high creatinine, imaging
revealed unequal and scarred small kidney. What is the diagnosis?
A. Acute Pyelonephritis
B. Chronic Pyelonephritis?
C. AGN
D. ADPKD
E. Urethral valve

255. Cause of recurrent maxillary sinusitis?


A. More spacious
B. Location of maxillary hiatus high up
C. Abscesses from tooth
D. Staph. aureus
E. Antibiotic use

256. What is the investigation of choice in barrets esophagus?


A. Esophagectomy and histopathology
B. Ultrasonography
C. CT scan
D. Endoscopy and biopsy
E. PET scan.

257. Pilon Fracture Management?


A. Cast
B. External fixation
C. Internal fixation
D. Amputation
E. IM nail

258. Patient has chest pain and dyspnea after hip replacement surgery, He was on
enoxaparin, saturation was 90%, Low BP and high HR. Cause?
A. Fat embolism
B. Thromboembolism
C. MI
D. TIA
E. Pulmonary Infraction

259. Absent pulse after atria fibrillation cause?


A. Left ventricular thrombus
B. Atherosclerosis
C. Small vessel vasculitis

176
D. Left atria myxoma
E. Paradoxical embolus

260. A patient present with Wrist drop, injury of nerve in?


A. Scapular #
B. Mid shaft Ulna #
C. Mid shaft Radius #
D. Fracture of thumb
E. Median nerve

261. 2-year-old baby present with fracture and X-ray shows multiple lytic lesion, what might
be the pathology?
A. Nephroblastoma
B. Medulloblastoma
C. Rhabdomyosarcoma
D. Neuroblastoma
E. Astrocytoma

262. Patient had stroke and 14 days after the incident he is recovering slowly, what is the
best feeding process?
A. Oro gastric tube
B. NG tube
C. PEG tube
D. Oral
E. TPN

263. Stain for amyloid?


A. Acid fuchsin
B. Patent blue
C. zeil nelson
D. Congo red
E. H&E

264. A patient present with cervical pain and loss of sensation along the C8 dermatome,
which is the best investigation?
A. X-ray
B. CT
C. MRI
D. PET CT
E. USG

265. A 25 years-old man has 8/9(multiple) episode of diarrhea with joint pain, diagnosis?
A. Rectal ulcer
B. Chrons disease

177
C. Ulcerative colitis
D. AVM
E. RA

266. Patient present with verbal aphasia which arterial territory is abnormal?
A. ACA
B. PCA
C. PICA
D. MCA
E. AICA

267. Patient is in ICU with this ECG. What to do next?

A. Atropine
B. Urgent pacing??
C. Clopidogrel
D. Aspirin
E. Streptokinase

268. An Old man present with Arnold Chiari malformation presented with syringomyelia.
What is the associated abnormality?
A. Lession in spinothalamic tract
B. Meningocele
C. Meningomyelocele
D. Hydrocephalus
E. Syringo-bulbia

178
109. A patient of chron’s disease present with low calcium and phosphate and high ALP. What
is the diagnosis?
A. Carcinoma metastatic
B. osteoporosis
C. Osteopetrosis
D. Osteomalacia
E. Paget’s disease

110. Recurrent emboli Management?


A. LMWH
B. UFH
C. warfarin
D.IVC filter
E. STK

111. A 78-year old woman present with frank hematuria and renal mass, clinically diagnosed
as renal carcinoma what is the best investigation next?
A. Biopsy
B. CT
C. MRI
D. USG??
E. X-ray

112. A 30-year-old patient present with peri anal fistula due to chrons disease, what to do
next?
A. Lay open
B. Cutting seton
C. Loose seton??
D. MRI
E. Fistulotomy

113. A footballer had head injury he was unconscious for a few minutes and spectate rest of
the game in the bench and found collapsed next day at the bed. What is the diagnosis?
A. Sub dural hemorrhage
B. Sub- arachnoid hemorrhage
C. Chronic SDH
D. EDH
E. TIA

114 A 13-year-old girl present with severe dyspnea and respiratory distress, known case of
asthma, what is the best measure in critical asthma patient?
A. IPPV
B. CPAP
C. Intubation

179
D. Steroid
E. Antibiotic

115. What to divide during tracheostomy?


A. Crico thyroid membrane
B. Thyro-hyoid membrane
C. Uvula
D. Arytenoids membrane
E. Thyroid isthmus

180

You might also like